2016 AIME I Math Jam
Go back to the Math Jam ArchiveAoPS instructors discuss all 15 problems of the 2016 AIME I.
Copyright © 2024 AoPS Incorporated. This page is copyrighted material. You can view and print this page for your own use, but you cannot share the contents of this file with others.
Facilitator: Jeremy Copeland
copeland
2016-03-05 18:23:58
Hello and welcome to the Math Jam. We'll begin a minute or two after the hour.
The classroom is moderated. That means the things you type won't appear directly in the classroom. Don't worry, we do see you!
Hello and welcome to the Math Jam. We'll begin a minute or two after the hour.
The classroom is moderated. That means the things you type won't appear directly in the classroom. Don't worry, we do see you!
eisirrational
2016-03-05 18:51:09
Hello!
Hello!
akaashp11
2016-03-05 18:51:09
Hello! How's your day been?
Hello! How's your day been?
copeland
2016-03-05 18:51:12
Hi. Rocking.
Hi. Rocking.
copeland
2016-03-05 18:52:21
Please do not ask about administrative aspects of the contests, and please do not ask me to speculate about the results. I have no idea what the index will be for the USAJMO or the USAMO.
Please do not ask about administrative aspects of the contests, and please do not ask me to speculate about the results. I have no idea what the index will be for the USAJMO or the USAMO.
epiclucario
2016-03-05 18:54:42
Will this math jam allow me to get a 16 on the AIME II?
Will this math jam allow me to get a 16 on the AIME II?
copeland
2016-03-05 18:54:43
No guarantees.
No guarantees.
ychen
2016-03-05 18:56:18
when will the results come out approximately?
when will the results come out approximately?
copeland
2016-03-05 18:56:20
Speaking of "no guarantees. . . "
Speaking of "no guarantees. . . "
SHARKYBOY
2016-03-05 18:56:55
Hi AoPS!
Hi AoPS!
SS00090
2016-03-05 18:57:16
How many problems are we gonna do?
How many problems are we gonna do?
copeland
2016-03-05 18:57:18
15!
15!
SmallKid
2016-03-05 18:57:27
Hi SHARKYBOY!
Hi SHARKYBOY!
chaoshadow37
2016-03-05 18:58:49
was there a problem with #9? if so how is it gonna affect the scoring?
was there a problem with #9? if so how is it gonna affect the scoring?
copeland
2016-03-05 18:58:50
Yes, and I am officially refusing to predict that they won't do anything.
Yes, and I am officially refusing to predict that they won't do anything.
lidada
2016-03-05 18:59:10
15 factorial is a lot of problems...
15 factorial is a lot of problems...
copeland
2016-03-05 18:59:11
Yes, did you bring snacks?
Yes, did you bring snacks?
v8599131275
2016-03-05 18:59:32
Wassup Mr. Copeland
Wassup Mr. Copeland
copeland
2016-03-05 18:59:33
Yo.
Yo.
ninjataco
2016-03-05 19:00:19
did you have a favorite problem?
did you have a favorite problem?
copeland
2016-03-05 19:00:24
I loved all but #7 equally.
I loved all but #7 equally.
nukelauncher
2016-03-05 19:00:30
will we do the problems in order? this is my first math jam
will we do the problems in order? this is my first math jam
copeland
2016-03-05 19:00:33
For sure.
For sure.
popcorn1
2016-03-05 19:00:45
1307674368000 is a LOT of problems, what was the time limit?
1307674368000 is a LOT of problems, what was the time limit?
SmallKid
2016-03-05 19:00:45
15! = $1307674368000$. Assuming we do one problem every 12 minutes (AIME speed), it will take us almost 30 million years.
15! = $1307674368000$. Assuming we do one problem every 12 minutes (AIME speed), it will take us almost 30 million years.
copeland
2016-03-05 19:00:56
We need to be faster than that.
We need to be faster than that.
jfurf
2016-03-05 19:01:17
Lets do a problem every millisecond!
Lets do a problem every millisecond!
copeland
2016-03-05 19:01:23
That'll definitely speed things up.
That'll definitely speed things up.
SmallKid
2016-03-05 19:01:43
It'll still take us around 41 years.
It'll still take us around 41 years.
copeland
2016-03-05 19:01:51
Getting better!
Getting better!
Locust
2016-03-05 19:01:57
Good, 30 million years, afterwards we can take the AIME again in the year 30002016
Good, 30 million years, afterwards we can take the AIME again in the year 30002016
copeland
2016-03-05 19:02:15
Alright, ready to start?
Alright, ready to start?
brian6liu
2016-03-05 19:02:41
yeah
yeah
SimonSun
2016-03-05 19:02:41
yessir
yessir
walnutwaldo20
2016-03-05 19:02:41
Yup
Yup
sohappy
2016-03-05 19:02:41
yes
yes
SHARKYBOY
2016-03-05 19:02:41
yup
yup
Darth_Math
2016-03-05 19:02:41
yes
yes
ychen
2016-03-05 19:02:41
YEAH!
YEAH!
blue8931
2016-03-05 19:02:41
yes
yes
LoneConquerorer
2016-03-05 19:02:41
yeah!
yeah!
math2fun
2016-03-05 19:02:41
oui
oui
akaashp11
2016-03-05 19:02:41
Yessir
Yessir
ompatel99
2016-03-05 19:02:41
Let's roll!
Let's roll!
andsun19
2016-03-05 19:02:41
Yup
Yup
jfurf
2016-03-05 19:02:41
YEAH!!
YEAH!!
hamup1
2016-03-05 19:02:41
Yes!
Yes!
copeland
2016-03-05 19:02:57
Welcome to the 2016 AIME I Math Jam!
Welcome to the 2016 AIME I Math Jam!
copeland
2016-03-05 19:02:57
I'm Jeremy Copeland, and I'll be leading our discussion tonight.
I'm Jeremy Copeland, and I'll be leading our discussion tonight.
copeland
2016-03-05 19:03:03
I'm the school director here at AoPS.
I'm the school director here at AoPS.
copeland
2016-03-05 19:03:06
I like math.
I like math.
andsun19
2016-03-05 19:03:17
I like math too!
I like math too!
sohappy
2016-03-05 19:03:17
I also like math
I also like math
SmallKid
2016-03-05 19:03:17
Me too!
Me too!
kikipet
2016-03-05 19:03:17
me too!
me too!
copeland
2016-03-05 19:03:19
Great!
Great!
copeland
2016-03-05 19:03:36
Um. . . bio. . .
Um. . . bio. . .
copeland
2016-03-05 19:03:44
I have a Ph.D. from UChicago.
I have a Ph.D. from UChicago.
copeland
2016-03-05 19:03:48
and, uh. . .
and, uh. . .
copeland
2016-03-05 19:03:51
I taught at MIT for a while. . .
I taught at MIT for a while. . .
copeland
2016-03-05 19:04:02
and, uh, I work at AoPS.
and, uh, I work at AoPS.
copeland
2016-03-05 19:04:04
Great!
Great!
copeland
2016-03-05 19:04:06
Before we get started I would like to take a moment to explain our virtual classroom to those who have not previously participated in a Math Jam or one of our online classes.
Before we get started I would like to take a moment to explain our virtual classroom to those who have not previously participated in a Math Jam or one of our online classes.
copeland
2016-03-05 19:04:08
The classroom is moderated, meaning that students can type into the classroom, but these comments will not go directly into the room. These comments go to the instructors, who may choose to share your comments with the room.
The classroom is moderated, meaning that students can type into the classroom, but these comments will not go directly into the room. These comments go to the instructors, who may choose to share your comments with the room.
copeland
2016-03-05 19:04:15
This helps keep the class organized and on track. This also means that only well-written comments will be dropped into the classroom, so please take time writing responses that are complete and easy to read.
This helps keep the class organized and on track. This also means that only well-written comments will be dropped into the classroom, so please take time writing responses that are complete and easy to read.
copeland
2016-03-05 19:04:18
There are a lot of students here! As I said, only a relatively small fraction of the well-written comments will be passed to the entire group. Please do not take it personally if your responses do not get posted, and please do not complain about it. I expect this Math Jam to be much larger than our typical class, so please be patient with me---there are quite a few of you here tonight!!
There are a lot of students here! As I said, only a relatively small fraction of the well-written comments will be passed to the entire group. Please do not take it personally if your responses do not get posted, and please do not complain about it. I expect this Math Jam to be much larger than our typical class, so please be patient with me---there are quite a few of you here tonight!!
copeland
2016-03-05 19:04:32
Also, we won't be going through the math quite as thoroughly as we do in our classes -- I can't teach all the necessary material for every problem as we go. Another difference between tonight and our regular online classes is that it is very unlikely that we'll be able to answer every single question you ask. We always to try do so in our regular online classes, but we have a large number of students tonight! So, please go ahead and ask questions, but also please understand if we aren't able to answer them all!
Also, we won't be going through the math quite as thoroughly as we do in our classes -- I can't teach all the necessary material for every problem as we go. Another difference between tonight and our regular online classes is that it is very unlikely that we'll be able to answer every single question you ask. We always to try do so in our regular online classes, but we have a large number of students tonight! So, please go ahead and ask questions, but also please understand if we aren't able to answer them all!
budu
2016-03-05 19:04:35
Jeremy joined AoPS in 2009. He earned his Ph.D. in mathematics from the University of Chicago in 2006 and was on the math faculty at MIT from 2006 to 2009. He is currently the Online School Director at AoPS. He once specialized in turning hard problems in geometry, algebra, and mathematical physics into easy problems in combinatorics and graph theory. Now he specializes in helping to redistribute mathematics from brilliant teachers to brilliant students. He gets his signs right 50 percent of the
Jeremy joined AoPS in 2009. He earned his Ph.D. in mathematics from the University of Chicago in 2006 and was on the math faculty at MIT from 2006 to 2009. He is currently the Online School Director at AoPS. He once specialized in turning hard problems in geometry, algebra, and mathematical physics into easy problems in combinatorics and graph theory. Now he specializes in helping to redistribute mathematics from brilliant teachers to brilliant students. He gets his signs right 50 percent of the
copeland
2016-03-05 19:04:38
Ooh!
Ooh!
copeland
2016-03-05 19:04:41
Someone did his homework.
Someone did his homework.
copeland
2016-03-05 19:04:51
We plan to have two teaching assistants with us tonight to help answer your questions.
We plan to have two teaching assistants with us tonight to help answer your questions.
copeland
2016-03-05 19:04:59
We sent one to the wrong room, unfortunately.
We sent one to the wrong room, unfortunately.
copeland
2016-03-05 19:05:06
The other is really good, though:
The other is really good, though:
copeland
2016-03-05 19:05:09
Nicholas Yang (nackster): Nick is currently a sophomore at Princeton University. He has enjoyed competition math since elementary school, and has been particularly active in Florida's Mu Alpha Theta math organization. He has had success in math by qualifying for National MATHCOUNTS and the USA Math Olympiad. Nick loves to help others learn, whether it is in math or other subjects, and he's very excited to help out with AoPS classes!
Nicholas Yang (nackster): Nick is currently a sophomore at Princeton University. He has enjoyed competition math since elementary school, and has been particularly active in Florida's Mu Alpha Theta math organization. He has had success in math by qualifying for National MATHCOUNTS and the USA Math Olympiad. Nick loves to help others learn, whether it is in math or other subjects, and he's very excited to help out with AoPS classes!
nackster12
2016-03-05 19:05:15
Hi everyone!
Hi everyone!
popcorn1
2016-03-05 19:05:37
Time for # 1!
Time for # 1!
v8599131275
2016-03-05 19:05:37
Shout out to my man Mr. Yang
Shout out to my man Mr. Yang
sohappy
2016-03-05 19:05:37
hi
hi
SHARKYBOY
2016-03-05 19:05:37
Hi Nick!
Hi Nick!
Locust
2016-03-05 19:05:37
Hello!!!
Hello!!!
kikipet
2016-03-05 19:05:37
Hi!
Hi!
popcorn1
2016-03-05 19:05:37
Hello!
Hello!
ThorJames
2016-03-05 19:05:37
Hi
Hi
ychen
2016-03-05 19:05:37
hi nackster12!
hi nackster12!
SmallKid
2016-03-05 19:05:37
Hi there!
Hi there!
SS00090
2016-03-05 19:05:37
hi
hi
sxu
2016-03-05 19:05:37
hello
hello
Aragorn66
2016-03-05 19:05:37
Hello
Hello
ryanyz10
2016-03-05 19:05:37
hi!
hi!
copeland
2016-03-05 19:05:43
He can answer questions by whispering to you or by opening a window with you to chat 1-on-1. However, due to the large size of the session tonight, they may not be able to get to you right away (or at all). Repeating your question over and over is more likely to annoy us than to get it answered faster, so please, just ask your question once and be patient, and please understand that we may not be able to answer all the questions tonight.
He can answer questions by whispering to you or by opening a window with you to chat 1-on-1. However, due to the large size of the session tonight, they may not be able to get to you right away (or at all). Repeating your question over and over is more likely to annoy us than to get it answered faster, so please, just ask your question once and be patient, and please understand that we may not be able to answer all the questions tonight.
copeland
2016-03-05 19:05:55
Please also remember that the purpose of this Math Jam is to work through the solutions to AIME problems, and not to merely present the answers. "Working through the solutions" includes discussing problem-solving tactics. Also on occasion we may stop to prove things that you wouldn't necessary need to prove while doing the contest. So please, when a question is posted, do not simply respond with the final answer. That's not why we're here. We're going to work through the problems step-by-step, and comments that skip key steps or jump ahead in the problem, without providing explanation or motivation, won't be acknowledged.
Please also remember that the purpose of this Math Jam is to work through the solutions to AIME problems, and not to merely present the answers. "Working through the solutions" includes discussing problem-solving tactics. Also on occasion we may stop to prove things that you wouldn't necessary need to prove while doing the contest. So please, when a question is posted, do not simply respond with the final answer. That's not why we're here. We're going to work through the problems step-by-step, and comments that skip key steps or jump ahead in the problem, without providing explanation or motivation, won't be acknowledged.
copeland
2016-03-05 19:05:58
Before we get started, I have a question for those of you who took the test: What was your favorite question on the test?
Before we get started, I have a question for those of you who took the test: What was your favorite question on the test?
weilunsun28
2016-03-05 19:06:35
#1!
#1!
memc38123
2016-03-05 19:06:35
Number 1
Number 1
SHARKYBOY
2016-03-05 19:06:35
Question #6
Question #6
SmallKid
2016-03-05 19:06:35
15
15
ChickenOnRage
2016-03-05 19:06:35
8
8
budu
2016-03-05 19:06:35
number 10
number 10
popcorn1
2016-03-05 19:06:35
#2
#2
SS00090
2016-03-05 19:06:35
#2 the easist
#2 the easist
mewtwomew
2016-03-05 19:06:35
1
1
simon1221
2016-03-05 19:06:35
14
14
ThorJames
2016-03-05 19:06:35
3
3
exhonourated
2016-03-05 19:06:35
#7
#7
ucap
2016-03-05 19:06:35
6
6
jwlw2014
2016-03-05 19:06:35
6
6
TheStrangeCharm
2016-03-05 19:06:35
3
3
jrl_ct
2016-03-05 19:06:35
5
5
kikipet
2016-03-05 19:06:35
not #7
not #7
math0127
2016-03-05 19:06:35
#14
#14
exhonourated
2016-03-05 19:06:35
7 was littt
7 was littt
rcheng66
2016-03-05 19:06:35
NUMBER 1!!!!!!
NUMBER 1!!!!!!
copeland
2016-03-05 19:06:41
Really? 7?
Really? 7?
copeland
2016-03-05 19:06:55
OK, let's rock.
OK, let's rock.
copeland
2016-03-05 19:07:02
We're going to work through all 15 problems from the 2015 AIME I, in order.
We're going to work through all 15 problems from the 2015 AIME I, in order.
copeland
2016-03-05 19:07:06
1. For $-1 < r < 1$, let $S(r)$ denote the sum of the geometric series \[12 + 12r + 12r^2 + 12r^3 + \cdots.\] Let $a$ between -1 and 1 satisfy $S(a)S(-a) = 2016$. Find $S(a) + S(-a)$.
1. For $-1 < r < 1$, let $S(r)$ denote the sum of the geometric series \[12 + 12r + 12r^2 + 12r^3 + \cdots.\] Let $a$ between -1 and 1 satisfy $S(a)S(-a) = 2016$. Find $S(a) + S(-a)$.
copeland
2016-03-05 19:07:14
An infinite geometric series... What can we do with that?
An infinite geometric series... What can we do with that?
calculus_riju
2016-03-05 19:07:31
I think u meant 2016?
I think u meant 2016?
copeland
2016-03-05 19:07:38
Let's get started! We're going to work through all 15 problems from the 2016 AIME I, in order.
Let's get started! We're going to work through all 15 problems from the 2016 AIME I, in order.
copeland
2016-03-05 19:07:41
1. For $-1 < r < 1$, let $S(r)$ denote the sum of the geometric series \[12 + 12r + 12r^2 + 12r^3 + \cdots.\] Let $a$ between -1 and 1 satisfy $S(a)S(-a) = 2016$. Find $S(a) + S(-a)$.
1. For $-1 < r < 1$, let $S(r)$ denote the sum of the geometric series \[12 + 12r + 12r^2 + 12r^3 + \cdots.\] Let $a$ between -1 and 1 satisfy $S(a)S(-a) = 2016$. Find $S(a) + S(-a)$.
copeland
2016-03-05 19:07:45
That's what I said. . .
That's what I said. . .
copeland
2016-03-05 19:07:55
Now, here's an infinite geometric series... What can we do with that?
Now, here's an infinite geometric series... What can we do with that?
popcorn1
2016-03-05 19:08:11
a / 1-r
a / 1-r
lidada
2016-03-05 19:08:11
12/(1-r)
12/(1-r)
SHARKYBOY
2016-03-05 19:08:11
the sum is 12/(1-r)
the sum is 12/(1-r)
memc38123
2016-03-05 19:08:11
Use a/(1-r)
Use a/(1-r)
jfurf
2016-03-05 19:08:11
Write in closed form! $\frac{12}{1-r}$
Write in closed form! $\frac{12}{1-r}$
MikoTennisPro
2016-03-05 19:08:11
use the formula a/(1-r) to express S(r)
use the formula a/(1-r) to express S(r)
blue8931
2016-03-05 19:08:11
write the sum as $\frac{12}{1-a}$
write the sum as $\frac{12}{1-a}$
xwang1
2016-03-05 19:08:11
$\frac{a}{1-r}$
$\frac{a}{1-r}$
copeland
2016-03-05 19:08:14
We can write its sum in a closed form! \[12 + 12r + 12r^2 + \cdots = \frac{12}{1-r}.\]
We can write its sum in a closed form! \[12 + 12r + 12r^2 + \cdots = \frac{12}{1-r}.\]
copeland
2016-03-05 19:08:15
So what is $S(a) \cdot S(-a)$?
So what is $S(a) \cdot S(-a)$?
brian6liu
2016-03-05 19:08:49
144/(1-r^2)
144/(1-r^2)
SHARKYBOY
2016-03-05 19:08:49
144/(1-r^2)
144/(1-r^2)
hamup1
2016-03-05 19:08:49
$\dfrac{144}{1-a^2}.$
$\dfrac{144}{1-a^2}.$
ChickenOnRage
2016-03-05 19:08:49
$144/(1 - r^2)$
$144/(1 - r^2)$
math0127
2016-03-05 19:08:49
144/(1-r^2)
144/(1-r^2)
SimonSun
2016-03-05 19:08:49
$144/(1-r^2)$
$144/(1-r^2)$
ninjataco
2016-03-05 19:08:49
144/(1-r^2)
144/(1-r^2)
tennis1729
2016-03-05 19:08:49
144/(1-a^2)
144/(1-a^2)
copeland
2016-03-05 19:08:51
We have $S(a) = \dfrac{12}{1-a}$ and $S(-a) = \dfrac{12}{1+a}$. So, \[S(a) \cdot S(-a) = \frac{12}{1-a} \cdot \frac{12}{1+a} = \frac{144}{1 - a^2}.\]
We have $S(a) = \dfrac{12}{1-a}$ and $S(-a) = \dfrac{12}{1+a}$. So, \[S(a) \cdot S(-a) = \frac{12}{1-a} \cdot \frac{12}{1+a} = \frac{144}{1 - a^2}.\]
copeland
2016-03-05 19:08:53
We know $S(a)S(-a) = 2016$. So we could use this to explicitly solve for $a$ right now if we wanted to. But, let's take a closer look at the quantity we're asked for first.
We know $S(a)S(-a) = 2016$. So we could use this to explicitly solve for $a$ right now if we wanted to. But, let's take a closer look at the quantity we're asked for first.
copeland
2016-03-05 19:08:54
What is $S(a) + S(-a)$?
What is $S(a) + S(-a)$?
ryanyz10
2016-03-05 19:09:43
24/(1-r^2)
24/(1-r^2)
ompatel99
2016-03-05 19:09:43
24/(1-a^2)
24/(1-a^2)
ninjataco
2016-03-05 19:09:43
24/(1-a^2)
24/(1-a^2)
hamup1
2016-03-05 19:09:43
$\dfrac{24}{1-a^2}$
$\dfrac{24}{1-a^2}$
andsun19
2016-03-05 19:09:43
24/(1-a)/(1+a)
24/(1-a)/(1+a)
thequantumguy
2016-03-05 19:09:43
$24/(1-a^2)$
$24/(1-a^2)$
ChickenOnRage
2016-03-05 19:09:43
$24/(1-a^2)$
$24/(1-a^2)$
alexli2014
2016-03-05 19:09:43
24/(1-a^2)
24/(1-a^2)
blue8931
2016-03-05 19:09:43
$\frac{24}{1-r^2}$
$\frac{24}{1-r^2}$
SimonSun
2016-03-05 19:09:43
$12/(1-a)+12/(1+a)$
$12/(1-a)+12/(1+a)$
mathmaniatoo
2016-03-05 19:09:43
24/1-a^2
24/1-a^2
copeland
2016-03-05 19:09:46
That's equal to \[S(a) + S(-a) = \frac{12}{1-a} + \frac{12}{1+a}.\]
That's equal to \[S(a) + S(-a) = \frac{12}{1-a} + \frac{12}{1+a}.\]
copeland
2016-03-05 19:09:48
Putting everything over a common denominator, \[ S(a) + S(-a) = \frac{12(1+a)}{1-a^2} + \frac{12(1-a)}{1-a^2} = \frac{24}{1 - a^2}. \]
Putting everything over a common denominator, \[ S(a) + S(-a) = \frac{12(1+a)}{1-a^2} + \frac{12(1-a)}{1-a^2} = \frac{24}{1 - a^2}. \]
copeland
2016-03-05 19:10:05
That's way simpler than solving for $a.$ What's the sum?
That's way simpler than solving for $a.$ What's the sum?
copeland
2016-03-05 19:10:38
That is, what is $S(a)+S(-a)?$
That is, what is $S(a)+S(-a)?$
blue8931
2016-03-05 19:10:43
oh... just divide 2016 by 6 to get the answer of $\boxed{336}$
oh... just divide 2016 by 6 to get the answer of $\boxed{336}$
sxu
2016-03-05 19:10:43
336
336
blue8931
2016-03-05 19:10:43
2016/6 = 336
2016/6 = 336
calculus_riju
2016-03-05 19:10:43
336
336
tennis1729
2016-03-05 19:10:43
just divide 2016 by 6 to get 336
just divide 2016 by 6 to get 336
ChickenOnRage
2016-03-05 19:10:43
2016 / (144/24) = 2016/6 = 336
2016 / (144/24) = 2016/6 = 336
SimonSun
2016-03-05 19:10:43
2016/6=336
2016/6=336
goodbear
2016-03-05 19:10:43
336
336
walnutwaldo20
2016-03-05 19:10:43
336
336
nosaj
2016-03-05 19:10:43
336!
336!
copeland
2016-03-05 19:10:48
We see that $S(a) + S(-a)$ is just $\dfrac{24}{144}S(a) \cdot S(-a) = \dfrac16S(a) \cdot S(-a) = \dfrac16 \cdot 2016 =\boxed{336}.$
We see that $S(a) + S(-a)$ is just $\dfrac{24}{144}S(a) \cdot S(-a) = \dfrac16S(a) \cdot S(-a) = \dfrac16 \cdot 2016 =\boxed{336}.$
copeland
2016-03-05 19:10:54
OK, 1 down. 15 to go.
OK, 1 down. 15 to go.
memc38123
2016-03-05 19:11:10
14 to go
14 to go
ryanyz10
2016-03-05 19:11:10
14
14
brian6liu
2016-03-05 19:11:10
do you mean 14?
do you mean 14?
ThorJames
2016-03-05 19:11:10
14 to go
14 to go
lidada
2016-03-05 19:11:10
14 to go
14 to go
copeland
2016-03-05 19:11:19
OK, we'll only do 14 more, then.
OK, we'll only do 14 more, then.
copeland
2016-03-05 19:11:21
2. Two dice appear to be standard dice with their faces numbered from 1 to 6, but each die is weighted so that the probability of rolling the number $k$ is directly proportional to $k$. The probability of rolling a 7 with this pair of dice is $\dfrac mn$, where $m$ and $n$ are relatively prime positive integers. Find $m+n$.
2. Two dice appear to be standard dice with their faces numbered from 1 to 6, but each die is weighted so that the probability of rolling the number $k$ is directly proportional to $k$. The probability of rolling a 7 with this pair of dice is $\dfrac mn$, where $m$ and $n$ are relatively prime positive integers. Find $m+n$.
goodbear
2016-03-05 19:11:33
#15 is still left
#15 is still left
copeland
2016-03-05 19:11:39
Thanks for the save.
Thanks for the save.
copeland
2016-03-05 19:11:41
The probability of rolling a $k$ is directly proportional to $k$... Let's call the constant of proportionality $p$.
The probability of rolling a $k$ is directly proportional to $k$... Let's call the constant of proportionality $p$.
copeland
2016-03-05 19:11:43
Just to make sure we understand the question, what is the probability of rolling a 5?
Just to make sure we understand the question, what is the probability of rolling a 5?
brian6liu
2016-03-05 19:12:12
5p
5p
calculus_riju
2016-03-05 19:12:12
5p
5p
stan23456
2016-03-05 19:12:12
5p
5p
blue8931
2016-03-05 19:12:12
5p
5p
copeland
2016-03-05 19:12:15
The probability of rolling a 5 is $5p$. Similarly, the probability of rolling a 1 is $1p$, the probability of rolling a 2 is $2p$, etc.
The probability of rolling a 5 is $5p$. Similarly, the probability of rolling a 1 is $1p$, the probability of rolling a 2 is $2p$, etc.
copeland
2016-03-05 19:12:29
And what is $p?$
And what is $p?$
kikipet
2016-03-05 19:12:54
1/21
1/21
Locust
2016-03-05 19:12:54
1/21
1/21
ychen
2016-03-05 19:12:54
1/21
1/21
hamup1
2016-03-05 19:12:54
$\dfrac{1}{21}$
$\dfrac{1}{21}$
SimonSun
2016-03-05 19:12:54
1/21
1/21
simon1221
2016-03-05 19:12:54
1/21
1/21
idkanymath
2016-03-05 19:12:54
1/21
1/21
stan23456
2016-03-05 19:12:54
1/21
1/21
SmallKid
2016-03-05 19:12:54
Sum of all coefficients of p is 21, so 21p is 1. p has to be 1/21.
Sum of all coefficients of p is 21, so 21p is 1. p has to be 1/21.
Aragorn66
2016-03-05 19:12:54
1/21, because 1+2+3+4+5+6=21
1/21, because 1+2+3+4+5+6=21
Prof.iHen
2016-03-05 19:12:54
1/21
1/21
TheStrangeCharm
2016-03-05 19:12:54
$\frac{1}{21}$
$\frac{1}{21}$
fishy15
2016-03-05 19:12:54
1/21
1/21
copeland
2016-03-05 19:12:58
If we roll a die, it's guaranteed to show one of the numbers 1,2,3,4,5,6, so \[ 1p + 2p + 3p + 4p + 5p + 6p = 1.\]
If we roll a die, it's guaranteed to show one of the numbers 1,2,3,4,5,6, so \[ 1p + 2p + 3p + 4p + 5p + 6p = 1.\]
copeland
2016-03-05 19:13:00
Since $1 + 2 + 3 + 4 + 5+6 = 21$, we get $p = \dfrac 1{21}$.
Since $1 + 2 + 3 + 4 + 5+6 = 21$, we get $p = \dfrac 1{21}$.
copeland
2016-03-05 19:13:02
So the probability of rolling a 1 is $\dfrac{1}{21}$, the probability of rolling a 2 is $\dfrac{2}{21}$, etc.
So the probability of rolling a 1 is $\dfrac{1}{21}$, the probability of rolling a 2 is $\dfrac{2}{21}$, etc.
copeland
2016-03-05 19:13:04
Now that we have the probability of rolling each number, let's think about the probability of rolling numbers that sum to 7.
Now that we have the probability of rolling each number, let's think about the probability of rolling numbers that sum to 7.
copeland
2016-03-05 19:13:19
One of my favorite tools on the AIME. . .
One of my favorite tools on the AIME. . .
fishy15
2016-03-05 19:13:46
chart
chart
ThorJames
2016-03-05 19:13:46
chart.
chart.
TheStrangeCharm
2016-03-05 19:13:46
draw a chart
draw a chart
SK200
2016-03-05 19:13:46
table?
table?
copeland
2016-03-05 19:13:49
A table! Let's start making a table to organize our counting.
A table! Let's start making a table to organize our counting.
copeland
2016-03-05 19:13:51
\[\begin{array}{c|c|c}
\text{First Die}& \text{Second Die} & \text{Probability} \\
\hline
1 && \\
2 && \\
3 && \\
4 && \\
5 && \\
6 && \\
\end{array}\]
\[\begin{array}{c|c|c}
\text{First Die}& \text{Second Die} & \text{Probability} \\
\hline
1 && \\
2 && \\
3 && \\
4 && \\
5 && \\
6 && \\
\end{array}\]
copeland
2016-03-05 19:13:55
What can we roll on the second die to make a sum of 7 if we roll a 1 on the first die?
What can we roll on the second die to make a sum of 7 if we roll a 1 on the first die?
person754
2016-03-05 19:14:14
6
6
IQMathlete
2016-03-05 19:14:14
6\
6\
Peggy
2016-03-05 19:14:14
6
6
jrl_ct
2016-03-05 19:14:14
6
6
mewtwomew
2016-03-05 19:14:14
6
6
alexli2014
2016-03-05 19:14:14
6
6
SmallKid
2016-03-05 19:14:14
6.
6.
rcheng66
2016-03-05 19:14:14
6
6
akaashp11
2016-03-05 19:14:14
6
6
mewtwomew
2016-03-05 19:14:14
6.
6.
copeland
2016-03-05 19:14:17
We need to roll a 6. What is the probability of rolling a 1 on the first die and 6 on the second die?
We need to roll a 6. What is the probability of rolling a 1 on the first die and 6 on the second die?
Math1331Math
2016-03-05 19:14:41
6/441
6/441
SimonSun
2016-03-05 19:14:41
6/441
6/441
MikoTennisPro
2016-03-05 19:14:41
6/441
6/441
goodbear
2016-03-05 19:14:41
6/21^2
6/21^2
ninjataco
2016-03-05 19:14:41
6/441
6/441
blue8931
2016-03-05 19:14:41
1/21 * 6/21 = 6/441
1/21 * 6/21 = 6/441
brian6liu
2016-03-05 19:14:41
6/21^2
6/21^2
jfurf
2016-03-05 19:14:41
a $6$! This occurs with probability $\frac{1}{21} \cdot \frac{6}{21} = \frac{6}{441} $.
a $6$! This occurs with probability $\frac{1}{21} \cdot \frac{6}{21} = \frac{6}{441} $.
goodbear
2016-03-05 19:14:41
6/441
6/441
copeland
2016-03-05 19:14:42
The probability of rolling a 1 is $\dfrac{1}{21}$ and the probability of rolling a 6 is $\dfrac{6}{21}$. So the probability is $\dfrac{6}{21^2}$.
The probability of rolling a 1 is $\dfrac{1}{21}$ and the probability of rolling a 6 is $\dfrac{6}{21}$. So the probability is $\dfrac{6}{21^2}$.
copeland
2016-03-05 19:14:45
\[\begin{array}{c|c|c}
\text{First Die}& \text{Second Die} & \text{Probability} \\
\hline
1 & 6& \frac{6}{21^2} \\
2 && \\
3 && \\
4 && \\
5 && \\
6 && \\
\end{array}\]
\[\begin{array}{c|c|c}
\text{First Die}& \text{Second Die} & \text{Probability} \\
\hline
1 & 6& \frac{6}{21^2} \\
2 && \\
3 && \\
4 && \\
5 && \\
6 && \\
\end{array}\]
copeland
2016-03-05 19:14:46
If we roll a 2 on the first die?
If we roll a 2 on the first die?
andsun19
2016-03-05 19:15:09
5
5
goodbear
2016-03-05 19:15:09
5
5
rcheng66
2016-03-05 19:15:09
5
5
ompatel99
2016-03-05 19:15:09
5 on second die
5 on second die
calculus_riju
2016-03-05 19:15:09
5
5
person754
2016-03-05 19:15:09
5
5
SimonSun
2016-03-05 19:15:09
5 on second die
5 on second die
sxu
2016-03-05 19:15:18
10/441
10/441
kikipet
2016-03-05 19:15:18
10/441
10/441
Aragorn66
2016-03-05 19:15:18
10/21^2
10/21^2
SimonSun
2016-03-05 19:15:18
10/441
10/441
goodbear
2016-03-05 19:15:18
10/441
10/441
jrl_ct
2016-03-05 19:15:18
10/441
10/441
blue8931
2016-03-05 19:15:18
you need a 5; this occurs with probability $\frac{10}{441}$
you need a 5; this occurs with probability $\frac{10}{441}$
MikoTennisPro
2016-03-05 19:15:18
10/441
10/441
copeland
2016-03-05 19:15:20
Then we need a 5 on the second die. This happens with probability $\dfrac{2}{21} \cdot \dfrac{5}{21} = \dfrac{10}{21^2}.$
Then we need a 5 on the second die. This happens with probability $\dfrac{2}{21} \cdot \dfrac{5}{21} = \dfrac{10}{21^2}.$
copeland
2016-03-05 19:15:20
\[\begin{array}{c|c|c}
\text{First Die}& \text{Second Die} & \text{Probability} \\
\hline
1 & 6& \frac{6}{21^2} \\
2 &5& \frac{10}{21^2} \\
3 && \\
4 && \\
5 && \\
6 && \\
\end{array}\]
\[\begin{array}{c|c|c}
\text{First Die}& \text{Second Die} & \text{Probability} \\
\hline
1 & 6& \frac{6}{21^2} \\
2 &5& \frac{10}{21^2} \\
3 && \\
4 && \\
5 && \\
6 && \\
\end{array}\]
copeland
2016-03-05 19:15:21
A 3?
A 3?
Prof.iHen
2016-03-05 19:15:34
4
4
MikoTennisPro
2016-03-05 19:15:34
4 on second die
4 on second die
mewtwomew
2016-03-05 19:15:46
12/144
12/144
mewtwomew
2016-03-05 19:15:46
12/441
12/441
MikoTennisPro
2016-03-05 19:15:46
12/441
12/441
blue8931
2016-03-05 19:15:46
4; $\frac{12}{441}$
4; $\frac{12}{441}$
kikipet
2016-03-05 19:15:46
need a 4, P = 12/441
need a 4, P = 12/441
brian6liu
2016-03-05 19:15:46
you need a 4, so 12/21^2
you need a 4, so 12/21^2
fishy15
2016-03-05 19:15:46
4 so 12/441
4 so 12/441
idomath12345
2016-03-05 19:15:46
4 12/441
4 12/441
Math1331Math
2016-03-05 19:15:46
12/441
12/441
goodbear
2016-03-05 19:15:46
12/441
12/441
ompatel99
2016-03-05 19:15:50
You need a 4 on the second die with prob of 3/21*4/21=12/(21^2)
You need a 4 on the second die with prob of 3/21*4/21=12/(21^2)
SmallKid
2016-03-05 19:15:50
We need a 4, 3/4 combo has 3/21 * 4/21 probability = 12/441 = 4/147
We need a 4, 3/4 combo has 3/21 * 4/21 probability = 12/441 = 4/147
copeland
2016-03-05 19:15:52
Then we need a 4 on the second die. This happens with probability $\dfrac{3}{21} \cdot \dfrac{4}{21} = \dfrac{12}{21^2}.$
Then we need a 4 on the second die. This happens with probability $\dfrac{3}{21} \cdot \dfrac{4}{21} = \dfrac{12}{21^2}.$
copeland
2016-03-05 19:15:53
\[\begin{array}{c|c|c}
\text{First Die}& \text{Second Die} & \text{Probability} \\
\hline
1 & 6& \frac{6}{21^2} \\
2 &5& \frac{10}{21^2} \\
3 &4& \frac{12}{21^2} \\
4 && \\
5 && \\
6 && \\
\end{array}\]
\[\begin{array}{c|c|c}
\text{First Die}& \text{Second Die} & \text{Probability} \\
\hline
1 & 6& \frac{6}{21^2} \\
2 &5& \frac{10}{21^2} \\
3 &4& \frac{12}{21^2} \\
4 && \\
5 && \\
6 && \\
\end{array}\]
copeland
2016-03-05 19:15:58
Can we quickly fill up the table now?
Can we quickly fill up the table now?
akaashp11
2016-03-05 19:16:12
Symmetric after this point
Symmetric after this point
ompatel99
2016-03-05 19:16:12
Now we just fill it in backwads
Now we just fill it in backwads
andsun19
2016-03-05 19:16:12
And... symmetry!
And... symmetry!
memc38123
2016-03-05 19:16:12
Now just use their reverses
Now just use their reverses
ChickenOnRage
2016-03-05 19:16:12
yes, symmetry
yes, symmetry
brian6liu
2016-03-05 19:16:12
the second half is symmetric
the second half is symmetric
jfurf
2016-03-05 19:16:12
Yes. It is symmetrical.
Yes. It is symmetrical.
Math1331Math
2016-03-05 19:16:12
yes because of symmetry
yes because of symmetry
mathmaniatoo
2016-03-05 19:16:12
yes, just double
yes, just double
copeland
2016-03-05 19:16:14
The next three rows are symmetric with the first three.
The next three rows are symmetric with the first three.
copeland
2016-03-05 19:16:15
\[\begin{array}{c|c|c}
\text{First Die}& \text{Second Die} & \text{Probability} \\
\hline
1 & 6& \frac{6}{21^2} \\
2 &5& \frac{10}{21^2} \\
3 &4& \frac{12}{21^2} \\
4 & 3& \frac{12}{21^2} \\
5 & 2&\frac{10}{21^2} \\
6 &1&\frac{6}{21^2} \\
\end{array}\]
\[\begin{array}{c|c|c}
\text{First Die}& \text{Second Die} & \text{Probability} \\
\hline
1 & 6& \frac{6}{21^2} \\
2 &5& \frac{10}{21^2} \\
3 &4& \frac{12}{21^2} \\
4 & 3& \frac{12}{21^2} \\
5 & 2&\frac{10}{21^2} \\
6 &1&\frac{6}{21^2} \\
\end{array}\]
copeland
2016-03-05 19:16:16
So what is our desired probability?
So what is our desired probability?
Math1331Math
2016-03-05 19:16:54
8/63
8/63
lidada
2016-03-05 19:16:54
8/63
8/63
memc38123
2016-03-05 19:16:54
8/63
8/63
idkanymath
2016-03-05 19:16:54
8/63
8/63
goseahawks
2016-03-05 19:16:54
8/63
8/63
calculus_riju
2016-03-05 19:16:54
$\frac{8}{63}$
$\frac{8}{63}$
Aragorn66
2016-03-05 19:16:54
=8/63
=8/63
Locust
2016-03-05 19:16:54
weilunsun28
2016-03-05 19:16:54
8/63
8/63
copeland
2016-03-05 19:16:59
Since $56 = 8 \cdot 7$, this simplifies to $\dfrac{8}{7 \cdot 9} = \dfrac{8}{63}$. So what is our answer?
Since $56 = 8 \cdot 7$, this simplifies to $\dfrac{8}{7 \cdot 9} = \dfrac{8}{63}$. So what is our answer?
SmallKid
2016-03-05 19:17:14
But since we're adding m and n, our final answer is 71
But since we're adding m and n, our final answer is 71
andsun19
2016-03-05 19:17:14
71
71
weilunsun28
2016-03-05 19:17:14
71
71
ChickenOnRage
2016-03-05 19:17:14
071
071
fishy15
2016-03-05 19:17:14
71 yay
71 yay
ninjataco
2016-03-05 19:17:14
071
071
sxu
2016-03-05 19:17:14
71
71
memc38123
2016-03-05 19:17:14
71
71
mewtwomew
2016-03-05 19:17:14
071
071
kikipet
2016-03-05 19:17:14
071
071
Math1331Math
2016-03-05 19:17:14
71
71
SimonSun
2016-03-05 19:17:14
071
071
idkanymath
2016-03-05 19:17:14
71
71
person754
2016-03-05 19:17:14
71
71
rocket13jg
2016-03-05 19:17:14
071
071
stan23456
2016-03-05 19:17:14
071
071
Aragorn66
2016-03-05 19:17:14
71
71
calculus_riju
2016-03-05 19:17:14
71
71
Peggy
2016-03-05 19:17:14
71
71
alexli2014
2016-03-05 19:17:14
071
071
nosaj
2016-03-05 19:17:14
071
071
Math1331Math
2016-03-05 19:17:14
8+63=71
8+63=71
thequantumguy
2016-03-05 19:17:14
71
71
ychen
2016-03-05 19:17:18
071, can't miss the zero guuys
071, can't miss the zero guuys
copeland
2016-03-05 19:17:25
Mmm. Bubbles.
Mmm. Bubbles.
copeland
2016-03-05 19:17:30
3. A regular icosahedron is a 20-faced solid where each face is an equilateral triangle and five triangles meet at every vertex. The regular icosahedron shown below has one vertex at the top, one vertex at the bottom, an upper pentagon of five vertices all adjacent to the top vertex and all in the same horizontal plane, and a lower pentagon of five vertices all adjacent to the bottom vertex and all in another horizontal plane. Find the number of paths from the top vertex to the bottom vertex such that each part of a path goes downward or horizontally along an edge of the icosahedron, and no vertex is repeated.
3. A regular icosahedron is a 20-faced solid where each face is an equilateral triangle and five triangles meet at every vertex. The regular icosahedron shown below has one vertex at the top, one vertex at the bottom, an upper pentagon of five vertices all adjacent to the top vertex and all in the same horizontal plane, and a lower pentagon of five vertices all adjacent to the bottom vertex and all in another horizontal plane. Find the number of paths from the top vertex to the bottom vertex such that each part of a path goes downward or horizontally along an edge of the icosahedron, and no vertex is repeated.
copeland
2016-03-05 19:17:35
copeland
2016-03-05 19:17:46
(Deven stole the diagrams for this problem from MSTang.)
(Deven stole the diagrams for this problem from MSTang.)
nosaj
2016-03-05 19:17:50
Is that MSTang's diagram?
Is that MSTang's diagram?
copeland
2016-03-05 19:17:52
Yes it is, thank you MSTang!
Yes it is, thank you MSTang!
copeland
2016-03-05 19:18:08
Any ideas about this problem?
Any ideas about this problem?
andsun19
2016-03-05 19:18:48
Break it into steps!
Break it into steps!
ninjataco
2016-03-05 19:18:48
constructive counting
constructive counting
weilunsun28
2016-03-05 19:18:48
calculate each step then multiply together
calculate each step then multiply together
blue8931
2016-03-05 19:18:48
constructive counting
constructive counting
thequantumguy
2016-03-05 19:18:48
we can count the ways to move for each plane
we can count the ways to move for each plane
Aragorn66
2016-03-05 19:18:48
start from the top, how many ways can we moved down, etc
start from the top, how many ways can we moved down, etc
theeagle
2016-03-05 19:18:48
divide into layers
divide into layers
nosaj
2016-03-05 19:18:48
Let's think constructively about how to build a path to the bottom.
Let's think constructively about how to build a path to the bottom.
brian6liu
2016-03-05 19:18:48
constructive counting
constructive counting
copeland
2016-03-05 19:18:52
We can try constructive counting! We'll just start constructing the path step-by-step.
We can try constructive counting! We'll just start constructing the path step-by-step.
copeland
2016-03-05 19:19:10
Note: this is not "casework" here.
Note: this is not "casework" here.
copeland
2016-03-05 19:19:12
Right now we're at the red vertex up-top. What can we do?
Right now we're at the red vertex up-top. What can we do?
copeland
2016-03-05 19:19:13
memc38123
2016-03-05 19:19:44
5 ways downward
5 ways downward
bestwillcui1
2016-03-05 19:19:44
five coihces
five coihces
sxu
2016-03-05 19:19:44
5 ways to go down
5 ways to go down
Aragorn66
2016-03-05 19:19:44
we cna move down in 5 different ways
we cna move down in 5 different ways
SK200
2016-03-05 19:19:44
go in 5 ways
go in 5 ways
MikoTennisPro
2016-03-05 19:19:44
choose any of the 5 paths to go down
choose any of the 5 paths to go down
fishy15
2016-03-05 19:19:44
go down 5 ways
go down 5 ways
goodbear
2016-03-05 19:19:44
first step has 5 choices
first step has 5 choices
SimonSun
2016-03-05 19:19:44
we move to one of the 5 vertices
we move to one of the 5 vertices
ChickenOnRage
2016-03-05 19:19:44
move down to 1 of 5 verticies
move down to 1 of 5 verticies
thequantumguy
2016-03-05 19:19:44
their are 5 ways to move to the next plane
their are 5 ways to move to the next plane
ThorJames
2016-03-05 19:19:44
5 verticies
5 verticies
copeland
2016-03-05 19:19:46
We can move down to the pentagon (in blue below) in 5 different ways.
We can move down to the pentagon (in blue below) in 5 different ways.
copeland
2016-03-05 19:19:47
copeland
2016-03-05 19:19:48
What can we do now that we're on the blue pentagon?
What can we do now that we're on the blue pentagon?
kikipet
2016-03-05 19:20:20
go to another vertex on the pentagon or go down
go to another vertex on the pentagon or go down
weilunsun28
2016-03-05 19:20:20
we can move around the pentagon or go down
we can move around the pentagon or go down
SS00090
2016-03-05 19:20:20
Sideways or down
Sideways or down
mewtwomew
2016-03-05 19:20:20
either go down or move horizontally
either go down or move horizontally
person754
2016-03-05 19:20:20
move horizontally or go down
move horizontally or go down
walnutwaldo20
2016-03-05 19:20:56
stay at that vertex or got to any of the other 4
stay at that vertex or got to any of the other 4
alexli2014
2016-03-05 19:20:56
stay or move right or left
stay or move right or left
SimonSun
2016-03-05 19:20:56
we can move to any of the 5 points with 9 ways
we can move to any of the 5 points with 9 ways
ninjataco
2016-03-05 19:20:56
stay in the same spot, or move horizontally
stay in the same spot, or move horizontally
goodbear
2016-03-05 19:20:56
we can move to either one of the blue vertices in 9 ways
we can move to either one of the blue vertices in 9 ways
goodbear
2016-03-05 19:20:56
we can move to either one of the blue vertices in 2*5-1=9 ways
we can move to either one of the blue vertices in 2*5-1=9 ways
edwardneo
2016-03-05 19:20:56
there are 9 ways to move around the pentagon
there are 9 ways to move around the pentagon
calculus_riju
2016-03-05 19:20:56
we can stay there or move upto 4 vertices in clock or anticlock manner......9 ways
we can stay there or move upto 4 vertices in clock or anticlock manner......9 ways
copeland
2016-03-05 19:21:17
If we go down, we're forced to the next pentagon. Before that we should wander around.
If we go down, we're forced to the next pentagon. Before that we should wander around.
copeland
2016-03-05 19:21:30
We can move around the blue pentagon by either:
(a) Staying still
(b) Moving around counter-clockwise by up to 4 steps
(c) Moving around clockwise by up to 4 steps
We can move around the blue pentagon by either:
(a) Staying still
(b) Moving around counter-clockwise by up to 4 steps
(c) Moving around clockwise by up to 4 steps
copeland
2016-03-05 19:21:33
So we have 1+4+4=9 ways to move around the pentagon. Then how many ways are there to leave the blue pentagon?
So we have 1+4+4=9 ways to move around the pentagon. Then how many ways are there to leave the blue pentagon?
ryanyz10
2016-03-05 19:21:56
2
2
blue8931
2016-03-05 19:21:56
2
2
math0127
2016-03-05 19:21:56
two ways
two ways
goodbear
2016-03-05 19:21:56
2
2
rcheng66
2016-03-05 19:21:56
2
2
hliu70
2016-03-05 19:21:56
2
2
person754
2016-03-05 19:21:56
2
2
hliu70
2016-03-05 19:21:56
2 for each way
2 for each way
Aragorn66
2016-03-05 19:21:56
2
2
jwlw2014
2016-03-05 19:21:56
2 from each vertex
2 from each vertex
copeland
2016-03-05 19:21:58
No matter where we land, there are two different ways. For example, if we are at the red vertex below, we can travel to either of the vertices in green along the blue edges:
No matter where we land, there are two different ways. For example, if we are at the red vertex below, we can travel to either of the vertices in green along the blue edges:
copeland
2016-03-05 19:21:59
copeland
2016-03-05 19:22:01
Once we're at the lower pentagon, what can we do?
Once we're at the lower pentagon, what can we do?
andsun19
2016-03-05 19:22:28
Same thing! 9 ways
Same thing! 9 ways
sxu
2016-03-05 19:22:28
wander 9 ways
wander 9 ways
ompatel99
2016-03-05 19:22:28
Now another 9 ways because it's another pentagon
Now another 9 ways because it's another pentagon
blue8931
2016-03-05 19:22:28
same as upper pentagon; 9 options
same as upper pentagon; 9 options
akaashp11
2016-03-05 19:22:28
9 ways horizontally
9 ways horizontally
ryanyz10
2016-03-05 19:22:28
9 ways as well
9 ways as well
Peggy
2016-03-05 19:22:28
move on the pentagon, 9 ways again
move on the pentagon, 9 ways again
edwardneo
2016-03-05 19:22:28
there are 9 ways to move around the pentagon
there are 9 ways to move around the pentagon
rcheng66
2016-03-05 19:22:28
same as before wander around or go straight down in 9 ways
same as before wander around or go straight down in 9 ways
ninjataco
2016-03-05 19:22:28
9 ways to move around again
9 ways to move around again
mathmaniatoo
2016-03-05 19:22:28
rotate around 9 different ways
rotate around 9 different ways
copeland
2016-03-05 19:22:30
Again, we can move around the pentagon. We can either
(a) Stay still
(b) Move around counter-clockwise by up to 4 steps
(c) Move around clockwise by up to 4 steps
Again, we can move around the pentagon. We can either
(a) Stay still
(b) Move around counter-clockwise by up to 4 steps
(c) Move around clockwise by up to 4 steps
copeland
2016-03-05 19:22:31
So we have 1+4+4=9 ways to move around this pentagon as well. In how many ways can we leave this pentagon?
So we have 1+4+4=9 ways to move around this pentagon as well. In how many ways can we leave this pentagon?
sxu
2016-03-05 19:22:59
1
1
SimonSun
2016-03-05 19:22:59
1
1
rcheng66
2016-03-05 19:22:59
1
1
andsun19
2016-03-05 19:22:59
1
1
MikoTennisPro
2016-03-05 19:22:59
1
1
ychen
2016-03-05 19:22:59
1
1
blue8931
2016-03-05 19:22:59
only 1
only 1
Peggy
2016-03-05 19:22:59
1
1
ompatel99
2016-03-05 19:22:59
Once we move down, we're done-zo with only 1 way
Once we move down, we're done-zo with only 1 way
SK200
2016-03-05 19:22:59
1
1
fishy15
2016-03-05 19:22:59
1
1
alexli2014
2016-03-05 19:22:59
just 1
just 1
kikipet
2016-03-05 19:22:59
1 from each vertex
1 from each vertex
copeland
2016-03-05 19:23:02
There's only one vertex left for us to go to -- the bottom one -- so we have 1 way to do that.
There's only one vertex left for us to go to -- the bottom one -- so we have 1 way to do that.
copeland
2016-03-05 19:23:03
Let's consolidate all of our counting. How many ways are there to form such a path?
Let's consolidate all of our counting. How many ways are there to form such a path?
memc38123
2016-03-05 19:23:52
810
810
SimonSun
2016-03-05 19:23:52
$5*9*2*9=810$
$5*9*2*9=810$
rocket13jg
2016-03-05 19:23:52
810
810
MikoTennisPro
2016-03-05 19:23:52
5*9*2*9 = 810
5*9*2*9 = 810
blue8931
2016-03-05 19:23:52
5 * 9 * 2 * 9 * 1 = $\boxed{810}$
5 * 9 * 2 * 9 * 1 = $\boxed{810}$
alexli2014
2016-03-05 19:23:52
5*9*2*9*1=810
5*9*2*9*1=810
Peggy
2016-03-05 19:23:52
5*9*2*9=810
5*9*2*9=810
jrl_ct
2016-03-05 19:23:52
5*9*9*2 = 810
5*9*9*2 = 810
SmallKid
2016-03-05 19:23:52
5*9*9*2=810
5*9*9*2=810
hliu70
2016-03-05 19:23:52
5*9*2*9=810
5*9*2*9=810
ompatel99
2016-03-05 19:23:52
5*9*2*9=810
5*9*2*9=810
Aragorn66
2016-03-05 19:23:52
810
810
copeland
2016-03-05 19:23:54
We have 5 options for leaving the top, then 9 options for the navigating the first pentagon, 2 options for traveling between the pentagons, 9 options for navigating the second pentagon, and one way to leave the second pentagon. For a total of \[5\cdot9\cdot2\cdot9 = \boxed{810}\text{ paths}.\]
We have 5 options for leaving the top, then 9 options for the navigating the first pentagon, 2 options for traveling between the pentagons, 9 options for navigating the second pentagon, and one way to leave the second pentagon. For a total of \[5\cdot9\cdot2\cdot9 = \boxed{810}\text{ paths}.\]
copeland
2016-03-05 19:24:14
We finally managed to get our assistant into the right place.
We finally managed to get our assistant into the right place.
copeland
2016-03-05 19:24:17
Dillon Liu (dtliu): Dillon graduated from Columbia University in 2013 with a BS in Applied Physics and is now working on a DPhil at Oxford in Theoretical Condensed Matter Physics on a Marshall Scholarship. At Columbia, Dillon was a teaching assistant for physics and mathematics courses, including quantum mechanics and introductory cryptography. He has also been a teaching assistant at the Summer Science Program. At Oxford, he teaches for the fourth-year theory option covering quantum field theory, statistical mechanics, and other advanced topics.
Dillon Liu (dtliu): Dillon graduated from Columbia University in 2013 with a BS in Applied Physics and is now working on a DPhil at Oxford in Theoretical Condensed Matter Physics on a Marshall Scholarship. At Columbia, Dillon was a teaching assistant for physics and mathematics courses, including quantum mechanics and introductory cryptography. He has also been a teaching assistant at the Summer Science Program. At Oxford, he teaches for the fourth-year theory option covering quantum field theory, statistical mechanics, and other advanced topics.
copeland
2016-03-05 19:24:24
Sorry, Dillon. Welcome!
Sorry, Dillon. Welcome!
dtliu
2016-03-05 19:24:25
Hey all, I'm happy to be here!
Hey all, I'm happy to be here!
Locust
2016-03-05 19:24:37
Hi!!!!
Hi!!!!
Aragorn66
2016-03-05 19:24:37
dtliu hi long time no see
dtliu hi long time no see
blue8931
2016-03-05 19:24:37
hello!
hello!
kikipet
2016-03-05 19:24:37
hi
hi
edwardneo
2016-03-05 19:24:37
hi!
hi!
ychen
2016-03-05 19:24:41
hi mr dillon
hi mr dillon
Darth_Math
2016-03-05 19:24:41
Hi
Hi
sxu
2016-03-05 19:24:41
hi
hi
copeland
2016-03-05 19:24:49
4. A right prism with height $h$ has bases that are regular hexagons with sides of length 12. A vertex $A$ of the prism and its three adjacent vertices are the vertices of a triangular pyramid. The dihedral angle (the angle between the two planes) formed by the face of the pyramid that lies in a base of the prism and the face of the pyramid that does not contain $A$ measures $60^\circ.$ Find $h^2.$
4. A right prism with height $h$ has bases that are regular hexagons with sides of length 12. A vertex $A$ of the prism and its three adjacent vertices are the vertices of a triangular pyramid. The dihedral angle (the angle between the two planes) formed by the face of the pyramid that lies in a base of the prism and the face of the pyramid that does not contain $A$ measures $60^\circ.$ Find $h^2.$
copeland
2016-03-05 19:24:53
We start with. . .
We start with. . .
popcorn1
2016-03-05 19:25:28
picture!!!
picture!!!
Prof.iHen
2016-03-05 19:25:28
diagram
diagram
alexli2014
2016-03-05 19:25:28
diagram
diagram
calculus_riju
2016-03-05 19:25:28
diagram?!
diagram?!
LighteningAB
2016-03-05 19:25:28
A DIAGRAM
A DIAGRAM
illogical_21
2016-03-05 19:25:28
diagram
diagram
brian6liu
2016-03-05 19:25:28
a diagram
a diagram
simon1221
2016-03-05 19:25:28
diagram!
diagram!
copeland
2016-03-05 19:25:30
. . . a nice diagram:
. . . a nice diagram:
ompatel99
2016-03-05 19:25:33
Another stolen picture?
Another stolen picture?
copeland
2016-03-05 19:25:38
I made this one.
I made this one.
copeland
2016-03-05 19:25:40
copeland
2016-03-05 19:25:54
You can click any picture to see how pro I am at Asymptote.
You can click any picture to see how pro I am at Asymptote.
copeland
2016-03-05 19:26:01
You can also double-click it to make it stick around.
You can also double-click it to make it stick around.
copeland
2016-03-05 19:26:03
And here is our plane:
And here is our plane:
copeland
2016-03-05 19:26:04
copeland
2016-03-05 19:26:23
Now what should we draw?
Now what should we draw?
sxu
2016-03-05 19:26:51
the dihedral angle
the dihedral angle
copeland
2016-03-05 19:26:54
Sure, how?
Sure, how?
ChickenOnRage
2016-03-05 19:27:31
an altitude from A
an altitude from A
blue8931
2016-03-05 19:27:31
30-60-90 triangle
30-60-90 triangle
qwerty733
2016-03-05 19:27:31
The dihedral angle arms at the midpoint
The dihedral angle arms at the midpoint
memc38123
2016-03-05 19:27:31
A 30-60-90 triangle
A 30-60-90 triangle
hamup1
2016-03-05 19:27:31
foot from $A$ to that blue side
foot from $A$ to that blue side
andsun19
2016-03-05 19:27:31
Draw the line connecting the midpoint and the vertex
Draw the line connecting the midpoint and the vertex
andsun19
2016-03-05 19:27:31
Draw the line connecting the midpoint of the diagonal and A
Draw the line connecting the midpoint of the diagonal and A
TheStrangeCharm
2016-03-05 19:27:31
perpendicular from $A$ to the blue line in the bottom plane.
perpendicular from $A$ to the blue line in the bottom plane.
copeland
2016-03-05 19:27:34
We have a 30-60-90 triangle with right angle at $A,$ one vertex above $A$ and one vertex on the midpoint of the base edge:
We have a 30-60-90 triangle with right angle at $A,$ one vertex above $A$ and one vertex on the midpoint of the base edge:
copeland
2016-03-05 19:27:35
copeland
2016-03-05 19:27:38
Incidentally, to measure the dihedral angle between two planes, you first draw the intersection line ($CB$ in our diagram) and then you pick a point ($M$) on that line. From that point you draw lines in each of the two planes that are perpendicular to the original line. Here we drew $MA$ and $MV$, which are both perpendicular to $CB$. The angle between those lines is the dihedral angle between the planes - here it is $\angle VMA$.
Incidentally, to measure the dihedral angle between two planes, you first draw the intersection line ($CB$ in our diagram) and then you pick a point ($M$) on that line. From that point you draw lines in each of the two planes that are perpendicular to the original line. Here we drew $MA$ and $MV$, which are both perpendicular to $CB$. The angle between those lines is the dihedral angle between the planes - here it is $\angle VMA$.
copeland
2016-03-05 19:27:56
The height of $\triangle VAM$ is $h,$ which we're trying to find.
The height of $\triangle VAM$ is $h,$ which we're trying to find.
copeland
2016-03-05 19:27:57
copeland
2016-03-05 19:27:58
What else can we discover about $\triangle VAM?$
What else can we discover about $\triangle VAM?$
andsun19
2016-03-05 19:28:37
Find AM by looking at triangle ABC
Find AM by looking at triangle ABC
SuperMaltese
2016-03-05 19:28:37
we can find AM
we can find AM
andsun19
2016-03-05 19:28:37
We can find AM
We can find AM
memc38123
2016-03-05 19:28:37
AM is part of another 30-60-90
AM is part of another 30-60-90
Locust
2016-03-05 19:28:37
The length of AM
The length of AM
hliu70
2016-03-05 19:28:37
we can calculate AM
we can calculate AM
copeland
2016-03-05 19:28:40
The base is the height of an isosceles triangle with vertex angle $120^\circ$:
The base is the height of an isosceles triangle with vertex angle $120^\circ$:
copeland
2016-03-05 19:28:40
copeland
2016-03-05 19:28:41
And the red length?
And the red length?
blue8931
2016-03-05 19:28:56
30-60-90 triangles and AM= 6
30-60-90 triangles and AM= 6
walnutwaldo20
2016-03-05 19:28:56
AM = 6
AM = 6
sophiazhi
2016-03-05 19:28:56
AM = AC/2 = 6
AM = AC/2 = 6
SHARKYBOY
2016-03-05 19:28:56
AM = 6
AM = 6
MikoTennisPro
2016-03-05 19:28:56
AM = 6
AM = 6
person754
2016-03-05 19:28:56
6
6
walnutwaldo20
2016-03-05 19:28:56
12/2 = 6
12/2 = 6
alexli2014
2016-03-05 19:28:56
6
6
ThorJames
2016-03-05 19:28:56
6
6
blizzard10
2016-03-05 19:28:56
6
6
copeland
2016-03-05 19:28:59
The red length is definitely 6.
The red length is definitely 6.
copeland
2016-03-05 19:29:00
So what is $h^2?$
So what is $h^2?$
rcheng66
2016-03-05 19:29:33
108
108
alexli2014
2016-03-05 19:29:33
108
108
memc38123
2016-03-05 19:29:33
108
108
person754
2016-03-05 19:29:33
108
108
goodbear
2016-03-05 19:29:33
108
108
Peggy
2016-03-05 19:29:33
108
108
SuperMaltese
2016-03-05 19:29:33
(6sqrt3)^2 = 108
(6sqrt3)^2 = 108
walnutwaldo20
2016-03-05 19:29:33
(6* sqrt(3))^2 = 36*3 = 108
(6* sqrt(3))^2 = 36*3 = 108
Locust
2016-03-05 19:29:33
Niuniu01
2016-03-05 19:29:33
108
108
blue8931
2016-03-05 19:29:33
$(6\sqrt{3})^2=\boxed{108}$
$(6\sqrt{3})^2=\boxed{108}$
ninjataco
2016-03-05 19:29:33
(6sqrt 3)^2 = 108
(6sqrt 3)^2 = 108
copeland
2016-03-05 19:29:37
copeland
2016-03-05 19:29:38
This is a 30-60-90 triangle so $h=6\sqrt{3}$ and $h^2=\boxed{108}.$
This is a 30-60-90 triangle so $h=6\sqrt{3}$ and $h^2=\boxed{108}.$
copeland
2016-03-05 19:29:54
Ready for more?
Ready for more?
memc38123
2016-03-05 19:30:02
YEAH
YEAH
blue8931
2016-03-05 19:30:02
yes
yes
sxu
2016-03-05 19:30:02
yup
yup
SuperMaltese
2016-03-05 19:30:02
YES
YES
Aragorn66
2016-03-05 19:30:02
yes!
yes!
weilunsun28
2016-03-05 19:30:02
yesss
yesss
brian6liu
2016-03-05 19:30:02
yeah
yeah
Locust
2016-03-05 19:30:02
Yeah!!!
Yeah!!!
Niuniu01
2016-03-05 19:30:02
yeah!
yeah!
jrl_ct
2016-03-05 19:30:02
Of course!
Of course!
spicyray
2016-03-05 19:30:02
yeah
yeah
ninjataco
2016-03-05 19:30:02
yes
yes
ThorJames
2016-03-05 19:30:02
yes
yes
lleB_ocaT
2016-03-05 19:30:02
Ok
Ok
copeland
2016-03-05 19:30:05
Me too.
Me too.
copeland
2016-03-05 19:30:06
5. Anh read a book. On the first day she read $n$ pages in $t$ minutes, where $n$ and $t$ are positive integers. On the second day Anh read $n+1$ pages in $t+1$ minutes. Each day thereafter Ahn read one more page than she read on the previous day, and it took her one more minute than on the previous day until she completely read the 374 page book. It took her a total of 319 minutes to read the book. Find $n + t$.
5. Anh read a book. On the first day she read $n$ pages in $t$ minutes, where $n$ and $t$ are positive integers. On the second day Anh read $n+1$ pages in $t+1$ minutes. Each day thereafter Ahn read one more page than she read on the previous day, and it took her one more minute than on the previous day until she completely read the 374 page book. It took her a total of 319 minutes to read the book. Find $n + t$.
copeland
2016-03-05 19:30:10
Let's start with small numbers and find a pattern.
Let's start with small numbers and find a pattern.
copeland
2016-03-05 19:30:11
How many pages has Anh read in total after two days?
How many pages has Anh read in total after two days?
Niuniu01
2016-03-05 19:30:34
2n+1
2n+1
mewtwomew
2016-03-05 19:30:34
2n+1
2n+1
nosaj
2016-03-05 19:30:34
n + n + 1 = 2n+1
n + n + 1 = 2n+1
walnutwaldo20
2016-03-05 19:30:34
2n + 1
2n + 1
fishy15
2016-03-05 19:30:34
2n+1
2n+1
SuperMaltese
2016-03-05 19:30:34
2n+1
2n+1
ompatel99
2016-03-05 19:30:34
2n+1 pages
2n+1 pages
kikipet
2016-03-05 19:30:34
2n+1
2n+1
Locust
2016-03-05 19:30:34
2n+1
2n+1
rcheng66
2016-03-05 19:30:34
2n+1
2n+1
person754
2016-03-05 19:30:34
2n+1
2n+1
SimonSun
2016-03-05 19:30:34
2n+1
2n+1
copeland
2016-03-05 19:30:37
After two days, Anh has read $n + (n+1)$, or $2n + 1$ pages.
After two days, Anh has read $n + (n+1)$, or $2n + 1$ pages.
copeland
2016-03-05 19:30:37
After three days?
After three days?
SmallKid
2016-03-05 19:30:53
3n+3
3n+3
person754
2016-03-05 19:30:53
3n+3
3n+3
sxu
2016-03-05 19:30:53
3n+3
3n+3
brian6liu
2016-03-05 19:30:53
3n+3
3n+3
space_space
2016-03-05 19:30:53
3n+2
3n+2
Locust
2016-03-05 19:30:53
3n+3
3n+3
LighteningAB
2016-03-05 19:30:53
3n+3
3n+3
ninjataco
2016-03-05 19:30:53
3n+3
3n+3
fishy15
2016-03-05 19:30:53
3n+3
3n+3
hliu70
2016-03-05 19:30:53
3n+3
3n+3
copeland
2016-03-05 19:30:54
After three days, Anh has read $2n + 1 + (n+2) = 3n + (1 + 2)$ pages.
After three days, Anh has read $2n + 1 + (n+2) = 3n + (1 + 2)$ pages.
copeland
2016-03-05 19:30:55
Anyone seeing a pattern?
Anyone seeing a pattern?
ompatel99
2016-03-05 19:31:41
Triangular numbers
Triangular numbers
memc38123
2016-03-05 19:31:41
Number of days * n + the number of days -1th triangular number
Number of days * n + the number of days -1th triangular number
space_space
2016-03-05 19:31:41
1,3,6,10,15
1,3,6,10,15
SimonSun
2016-03-05 19:31:41
trianglular numbes
trianglular numbes
ompatel99
2016-03-05 19:31:41
kn+(k*(k-1))/2
kn+(k*(k-1))/2
goodbear
2016-03-05 19:31:41
kn+k(k-1)/2
kn+k(k-1)/2
Locust
2016-03-05 19:31:41
The amount of extra pages besides the n term are triangular numbers
The amount of extra pages besides the n term are triangular numbers
blizzard10
2016-03-05 19:31:41
After m days, Anh has read $mn + \dfrac{(m-1)(m)}{2}$ pages.
After m days, Anh has read $mn + \dfrac{(m-1)(m)}{2}$ pages.
kikipet
2016-03-05 19:31:41
after x days, total # of pages is xn+x(x-1)/2
after x days, total # of pages is xn+x(x-1)/2
ryanyz10
2016-03-05 19:31:41
kn + k(k-1)/2
kn + k(k-1)/2
brian6liu
2016-03-05 19:31:41
kn+k(k-1)/2, where k is the number of days
kn+k(k-1)/2, where k is the number of days
copeland
2016-03-05 19:31:43
After $k$ days, Anh has read $kn + T_{k-1}$ pages. Where $T_{k-1}$ is the $(k-1)^{\text{st}}$ triangle number.
After $k$ days, Anh has read $kn + T_{k-1}$ pages. Where $T_{k-1}$ is the $(k-1)^{\text{st}}$ triangle number.
copeland
2016-03-05 19:31:48
We know that $T_{k-1} = \dfrac{(k-1)k}{2}$. So, after $k$ days, Anh has read \[ kn + \frac{(k-1)k}{2} \] pages.
We know that $T_{k-1} = \dfrac{(k-1)k}{2}$. So, after $k$ days, Anh has read \[ kn + \frac{(k-1)k}{2} \] pages.
copeland
2016-03-05 19:31:50
How many minutes has Anh read for after $k$ days?
How many minutes has Anh read for after $k$ days?
brian6liu
2016-03-05 19:32:25
kt+k(k-1)/2
kt+k(k-1)/2
SmallKid
2016-03-05 19:32:25
Same thing but replace n with t
Same thing but replace n with t
LighteningAB
2016-03-05 19:32:25
same thing for n but for t
same thing for n but for t
ninjataco
2016-03-05 19:32:25
kt + (k-1)k/2
kt + (k-1)k/2
goodbear
2016-03-05 19:32:25
kt+k(k-1)/2
kt+k(k-1)/2
nosaj
2016-03-05 19:32:25
*same thing except with $t$
*same thing except with $t$
andsun19
2016-03-05 19:32:25
kt+(k-1)k/2
kt+(k-1)k/2
kikipet
2016-03-05 19:32:25
$kt + \frac{(t - 1)t}{2}$
$kt + \frac{(t - 1)t}{2}$
copeland
2016-03-05 19:32:27
Using the same argument, Anh has read for $kt + \dfrac{(k-1)k}{2}$ minutes.
Using the same argument, Anh has read for $kt + \dfrac{(k-1)k}{2}$ minutes.
copeland
2016-03-05 19:32:36
Now we get equations from the problem.
Now we get equations from the problem.
copeland
2016-03-05 19:32:37
If Anh read for $k$ days total, we have
\begin{align}
kn + \frac{(k-1)k}{2} &= 374 \\
kt + \frac{(k-1)k}{2} & = 319.
\end{align}
If Anh read for $k$ days total, we have
\begin{align}
kn + \frac{(k-1)k}{2} &= 374 \\
kt + \frac{(k-1)k}{2} & = 319.
\end{align}
copeland
2016-03-05 19:32:40
What can we do with these two equations?
What can we do with these two equations?
person754
2016-03-05 19:33:11
subtract
subtract
brian6liu
2016-03-05 19:33:11
subtract them
subtract them
SimonSun
2016-03-05 19:33:11
subtract
subtract
jrl_ct
2016-03-05 19:33:11
subtraction!
subtraction!
goodbear
2016-03-05 19:33:11
subtract
subtract
sophiazhi
2016-03-05 19:33:11
subtract
subtract
jfurf
2016-03-05 19:33:11
Subtract them
Subtract them
weilunsun28
2016-03-05 19:33:11
subtract
subtract
space_space
2016-03-05 19:33:11
subtract them
subtract them
Peggy
2016-03-05 19:33:11
subtract
subtract
ThorJames
2016-03-05 19:33:11
subtract?
subtract?
copeland
2016-03-05 19:33:18
Since they both contain a $(k-1)k/2$ term, it makes sense to try subtracting them. That gives us \[k(n-t) = 55.\]
Since they both contain a $(k-1)k/2$ term, it makes sense to try subtracting them. That gives us \[k(n-t) = 55.\]
copeland
2016-03-05 19:33:19
Does that tell us anything interesting about $k$?
Does that tell us anything interesting about $k$?
SmallKid
2016-03-05 19:33:49
k is a factor of 55
k is a factor of 55
andsun19
2016-03-05 19:33:49
factor of 55
factor of 55
ninjataco
2016-03-05 19:33:49
k is a divisor of 55
k is a divisor of 55
blue8931
2016-03-05 19:33:49
divisor of 55
divisor of 55
brian6liu
2016-03-05 19:33:49
k divides 55
k divides 55
sophiazhi
2016-03-05 19:33:49
factor of 55
factor of 55
kikipet
2016-03-05 19:33:49
k is a factor of 55
k is a factor of 55
Peggy
2016-03-05 19:33:49
k is 1, 5, 11, 55
k is 1, 5, 11, 55
weilunsun28
2016-03-05 19:33:49
its a factor of 55
its a factor of 55
lidada
2016-03-05 19:33:49
k is a factor of 55
k is a factor of 55
MikoTennisPro
2016-03-05 19:33:49
must be a factor of 55?
must be a factor of 55?
hliu70
2016-03-05 19:33:49
factor of 55
factor of 55
copeland
2016-03-05 19:33:53
This tells us that $k$ must be a divisor of 55. So Anh read for 1, 5, 11, or 55 days.
This tells us that $k$ must be a divisor of 55. So Anh read for 1, 5, 11, or 55 days.
copeland
2016-03-05 19:33:54
Are there any of these we can eliminate immediately?
Are there any of these we can eliminate immediately?
popcorn1
2016-03-05 19:34:32
1.
1.
Peggy
2016-03-05 19:34:32
1
1
mewtwomew
2016-03-05 19:34:32
1
1
walnutwaldo20
2016-03-05 19:34:32
1
1
valmat01
2016-03-05 19:34:32
1
1
goseahawks
2016-03-05 19:34:32
1 day
1 day
math2fun
2016-03-05 19:34:32
1
1
space_space
2016-03-05 19:34:32
1
1
Prof.iHen
2016-03-05 19:34:32
1
1
qwerty733
2016-03-05 19:34:32
1
1
hliu70
2016-03-05 19:34:32
1 because the problem also says the next day, so this is impossible
1 because the problem also says the next day, so this is impossible
blizzard10
2016-03-05 19:34:32
We know it took her more than one day. It says so in the problem.
We know it took her more than one day. It says so in the problem.
illogical_21
2016-03-05 19:34:32
1
1
copeland
2016-03-05 19:34:36
The problem implicitly said that Anh read for a "second day", so we know $k\ne 1$.
The problem implicitly said that Anh read for a "second day", so we know $k\ne 1$.
copeland
2016-03-05 19:34:37
(I didn't like this step much.)
(I didn't like this step much.)
copeland
2016-03-05 19:34:39
Is there anything else we can do with these equations that might help?
Is there anything else we can do with these equations that might help?
ompatel99
2016-03-05 19:35:10
Add them
Add them
ninjataco
2016-03-05 19:35:10
add them
add them
kikipet
2016-03-05 19:35:10
add them
add them
sxu
2016-03-05 19:35:10
add
add
memc38123
2016-03-05 19:35:10
add them
add them
Locust
2016-03-05 19:35:10
Add to get the n+t term
Add to get the n+t term
copeland
2016-03-05 19:35:13
We could also try adding them. That gives us \[k(n+t) + k(k-1) = 693.\]
We could also try adding them. That gives us \[k(n+t) + k(k-1) = 693.\]
copeland
2016-03-05 19:35:15
What's nice about that?
What's nice about that?
MikoTennisPro
2016-03-05 19:35:33
can factor out k again?
can factor out k again?
kikipet
2016-03-05 19:35:33
factor out k
factor out k
ryanyz10
2016-03-05 19:35:33
factor k
factor k
brian6liu
2016-03-05 19:35:40
k divides 693 too
k divides 693 too
hamup1
2016-03-05 19:35:40
k divides 693
k divides 693
sxu
2016-03-05 19:35:40
k is a factor of 693
k is a factor of 693
calculus_riju
2016-03-05 19:35:40
k is a factor of 693
k is a factor of 693
Ericaops
2016-03-05 19:35:40
k is a factor of 693
k is a factor of 693
copeland
2016-03-05 19:35:43
So?
So?
mewtwomew
2016-03-05 19:36:09
11 is the only factor of 693 in common with 55
11 is the only factor of 693 in common with 55
kikipet
2016-03-05 19:36:09
k is 11
k is 11
SimonSun
2016-03-05 19:36:09
k=11
k=11
ompatel99
2016-03-05 19:36:09
k is 11
k is 11
Locust
2016-03-05 19:36:09
k=11
k=11
space_space
2016-03-05 19:36:09
k is 11
k is 11
math2fun
2016-03-05 19:36:09
k = 11
k = 11
popcorn1
2016-03-05 19:36:09
not 11
not 11
ThorJames
2016-03-05 19:36:09
11
11
hamup1
2016-03-05 19:36:09
k must equal 11
k must equal 11
blue8931
2016-03-05 19:36:09
k must be 11
k must be 11
TheStrangeCharm
2016-03-05 19:36:09
k = 11
k = 11
Aragorn66
2016-03-05 19:36:09
k is a factor of both 693 and 55, so must be 11
k is a factor of both 693 and 55, so must be 11
copeland
2016-03-05 19:36:11
5 and 55 clearly don't divide 693. So we must have $k = 11$. What do we do now?
5 and 55 clearly don't divide 693. So we must have $k = 11$. What do we do now?
goodbear
2016-03-05 19:36:51
plug in
plug in
19bobhu
2016-03-05 19:36:51
sub it in
sub it in
idomath12345
2016-03-05 19:36:51
sub in.
sub in.
blizzard10
2016-03-05 19:36:51
Plug in to our expressions from earlier
Plug in to our expressions from earlier
space_space
2016-03-05 19:36:51
plug it into the equation
plug it into the equation
illogical_21
2016-03-05 19:36:51
plug it back in
plug it back in
copeland
2016-03-05 19:36:57
Plug in where?
Plug in where?
ompatel99
2016-03-05 19:37:27
Plug it in and solve for n+t without solving for n and t
Plug it in and solve for n+t without solving for n and t
Locust
2016-03-05 19:37:27
Plug back into the determined equation to solve n+t
Plug back into the determined equation to solve n+t
jfurf
2016-03-05 19:37:27
We can find $n+t$ quickly! Substitute in for $k$ in that equation.
We can find $n+t$ quickly! Substitute in for $k$ in that equation.
Locust
2016-03-05 19:37:27
The equation with the term (n+t)
The equation with the term (n+t)
fishy15
2016-03-05 19:37:27
our added equation
our added equation
idomath12345
2016-03-05 19:37:27
the sum eq.
the sum eq.
sxu
2016-03-05 19:37:29
the second *new* equation
the second *new* equation
copeland
2016-03-05 19:37:32
We can just plug that in to this equation!
We can just plug that in to this equation!
copeland
2016-03-05 19:37:35
We have \[11(n+t) + 11\cdot 10 = 693.\] So what is $n+t$?
We have \[11(n+t) + 11\cdot 10 = 693.\] So what is $n+t$?
jfurf
2016-03-05 19:38:11
$n+t=53$ Bubble in $\boxed{053}$ :)
$n+t=53$ Bubble in $\boxed{053}$ :)
calculus_riju
2016-03-05 19:38:11
53
53
kikipet
2016-03-05 19:38:11
053
053
ninjataco
2016-03-05 19:38:11
053
053
SimonSun
2016-03-05 19:38:11
53
53
goodbear
2016-03-05 19:38:11
53
53
ychen
2016-03-05 19:38:11
053
053
Prof.iHen
2016-03-05 19:38:11
53
53
math2fun
2016-03-05 19:38:11
53
53
fz2012
2016-03-05 19:38:11
53
53
SuperMaltese
2016-03-05 19:38:11
053
053
space_space
2016-03-05 19:38:11
53
53
tennis1729
2016-03-05 19:38:11
53
53
person754
2016-03-05 19:38:11
053
053
goodbear
2016-03-05 19:38:11
053
053
copeland
2016-03-05 19:38:22
We get $n+t = \boxed{53}$.
We get $n+t = \boxed{53}$.
copeland
2016-03-05 19:38:25
Great!
Great!
copeland
2016-03-05 19:38:34
1/3 of the way done.
1/3 of the way done.
fishy15
2016-03-05 19:38:49
2/3 left
2/3 left
copeland
2016-03-05 19:38:51
copeland
2016-03-05 19:38:54
6. In $\triangle ABC$ let $I$ be the center of the inscribed circle, and let the bisector of $\angle ACB$ intersect $\overline{AB}$ at $L$. The line through $C$ and $L$ intersects the cirumscribed circle of $\triangle ABC$ at the two points $C$ and $D$. If $LI = 2$ and $LD = 3$, then $IC = \frac pq$, where $p$ and $q$ are relatively prime positive integers. Find $p+q$.
6. In $\triangle ABC$ let $I$ be the center of the inscribed circle, and let the bisector of $\angle ACB$ intersect $\overline{AB}$ at $L$. The line through $C$ and $L$ intersects the cirumscribed circle of $\triangle ABC$ at the two points $C$ and $D$. If $LI = 2$ and $LD = 3$, then $IC = \frac pq$, where $p$ and $q$ are relatively prime positive integers. Find $p+q$.
Quinn
2016-03-05 19:39:16
DIAGRAM!!!
DIAGRAM!!!
blizzard10
2016-03-05 19:39:16
Diagram!
Diagram!
Aragorn66
2016-03-05 19:39:16
diagram
diagram
weilunsun28
2016-03-05 19:39:16
diagram
diagram
fishy15
2016-03-05 19:39:16
i think we need a diagram
i think we need a diagram
space_space
2016-03-05 19:39:16
diagram plz
diagram plz
idomath12345
2016-03-05 19:39:16
DIAGRAM!
DIAGRAM!
popcorn1
2016-03-05 19:39:16
Draw a picture
Draw a picture
copeland
2016-03-05 19:39:17
Let's start with a diagram:
Let's start with a diagram:
copeland
2016-03-05 19:39:18
andsun19
2016-03-05 19:39:26
I didn't get this one
I didn't get this one
nosaj
2016-03-05 19:39:26
the first really hard one
the first really hard one
copeland
2016-03-05 19:39:53
Yeah, this problem was harder for me than it should have been. I think it's because it needed a more sophisticated theorem than 6 usually does.
Yeah, this problem was harder for me than it should have been. I think it's because it needed a more sophisticated theorem than 6 usually does.
copeland
2016-03-05 19:40:02
I've drawn $IC$ in red since that's what we're looking for.
I've drawn $IC$ in red since that's what we're looking for.
copeland
2016-03-05 19:40:07
What else do we know about this diagram?
What else do we know about this diagram?
ychen
2016-03-05 19:40:37
IL = 2, LD = 3
IL = 2, LD = 3
nosaj
2016-03-05 19:40:37
IL = 2, LD = 3
IL = 2, LD = 3
mathmaniatoo
2016-03-05 19:40:37
LI=2, DL=3
LI=2, DL=3
person754
2016-03-05 19:40:37
li=2 ld-3
li=2 ld-3
rcheng66
2016-03-05 19:40:37
LI and LD
LI and LD
goodbear
2016-03-05 19:40:37
2,3
2,3
sxu
2016-03-05 19:40:37
IL and LD
IL and LD
copeland
2016-03-05 19:40:39
Well, we know $IL = 2$ and $LD = 3$, we can add that in.
Well, we know $IL = 2$ and $LD = 3$, we can add that in.
copeland
2016-03-05 19:40:40
copeland
2016-03-05 19:40:41
What else do we know?
What else do we know?
hamup1
2016-03-05 19:41:13
D is the midpoint of arc BC b/c angle bisector
D is the midpoint of arc BC b/c angle bisector
Locust
2016-03-05 19:41:13
Arcs AD and DB are congruent?
Arcs AD and DB are congruent?
hamup1
2016-03-05 19:41:13
d is the midpoint of arc AB
d is the midpoint of arc AB
ninjataco
2016-03-05 19:41:13
AD = BD
AD = BD
copeland
2016-03-05 19:41:15
We know that $D$ is the midpoint of arc $AB$. So $DA = DB$. I'll draw those in in blue so we remember they're the same.
We know that $D$ is the midpoint of arc $AB$. So $DA = DB$. I'll draw those in in blue so we remember they're the same.
copeland
2016-03-05 19:41:16
copeland
2016-03-05 19:41:19
Are there any other lines we might want to add to our diagram to help us figure out more?
Are there any other lines we might want to add to our diagram to help us figure out more?
copeland
2016-03-05 19:42:16
(That last bit is true since $\angle CAD=\angle BAD$ by the problem statement.)
(That last bit is true since $\angle CAD=\angle BAD$ by the problem statement.)
ompatel99
2016-03-05 19:42:24
IA, IB
IA, IB
MikoTennisPro
2016-03-05 19:42:24
IA and IB?
IA and IB?
r_b
2016-03-05 19:42:24
AI
AI
hliu70
2016-03-05 19:42:24
IA and IB maybe?
IA and IB maybe?
nosaj
2016-03-05 19:42:24
AI, BI
AI, BI
Quinn
2016-03-05 19:42:24
AI and BI
AI and BI
copeland
2016-03-05 19:42:31
It might make sense to add the angle bisectors from $A$ or $B$, since we know those pass through $I$ as well. I'm going to add the bisector from $A$ into our diagram.
It might make sense to add the angle bisectors from $A$ or $B$, since we know those pass through $I$ as well. I'm going to add the bisector from $A$ into our diagram.
copeland
2016-03-05 19:42:34
copeland
2016-03-05 19:42:36
We have a lot of stuff in our diagram now. Anyone notice anything that looks interesting?
We have a lot of stuff in our diagram now. Anyone notice anything that looks interesting?
idomath12345
2016-03-05 19:43:07
Isosceles stuff?
Isosceles stuff?
MikoTennisPro
2016-03-05 19:43:07
AID is isoceles!
AID is isoceles!
fishy15
2016-03-05 19:43:07
is AID isoceles?
is AID isoceles?
nosaj
2016-03-05 19:43:07
ISOSCELES!
ISOSCELES!
ompatel99
2016-03-05 19:43:07
IAD is a special triangle? isosceles?
IAD is a special triangle? isosceles?
copeland
2016-03-05 19:43:11
It looks a lot like $ADI$ is isosceles. Let's see if we can prove it. What would we want to show to prove that?
It looks a lot like $ADI$ is isosceles. Let's see if we can prove it. What would we want to show to prove that?
nosaj
2016-03-05 19:44:23
base angles equal
base angles equal
sxu
2016-03-05 19:44:23
congruent angles
congruent angles
ninjataco
2016-03-05 19:44:23
angle AID = angle DAI
angle AID = angle DAI
Locust
2016-03-05 19:44:23
The bases angles are congruent
The bases angles are congruent
goodbear
2016-03-05 19:44:23
DAI=DIA
DAI=DIA
hamup1
2016-03-05 19:44:23
$\angle{AID}=\angle{IAD}$
$\angle{AID}=\angle{IAD}$
calculus_riju
2016-03-05 19:44:23
the angles must be equal
the angles must be equal
Aragorn66
2016-03-05 19:44:23
angle IAD=angle AID
angle IAD=angle AID
copeland
2016-03-05 19:44:33
We'll want to prove $\angle DAI = \angle DIA$ since we want to use angles to learn about lengths.
We'll want to prove $\angle DAI = \angle DIA$ since we want to use angles to learn about lengths.
copeland
2016-03-05 19:44:37
How can we write $\angle DAI$ in a nicer way?
How can we write $\angle DAI$ in a nicer way?
weilunsun28
2016-03-05 19:45:28
<DAB+<LAI
<DAB+<LAI
blue8931
2016-03-05 19:45:28
DAL + LAI
DAL + LAI
brian6liu
2016-03-05 19:45:28
<BAD+<BAI
<BAD+<BAI
goodbear
2016-03-05 19:45:28
BAI+BAD
BAI+BAD
spicyray
2016-03-05 19:45:28
BAD + LAI?
BAD + LAI?
copeland
2016-03-05 19:45:30
We can split it up into $\angle DAB$ and $\angle BAI$. So $\angle DAI = \angle DAB + \angle BAI$.
We can split it up into $\angle DAB$ and $\angle BAI$. So $\angle DAI = \angle DAB + \angle BAI$.
copeland
2016-03-05 19:45:33
Can we express either of those angles more nicely?
Can we express either of those angles more nicely?
ychen
2016-03-05 19:46:38
BAD = BCD by inscirbed arc
BAD = BCD by inscirbed arc
ompatel99
2016-03-05 19:46:38
DAB=DCB by subtended arc
DAB=DCB by subtended arc
copeland
2016-03-05 19:46:41
Yes, $\angle DAB = \angle DCB$, since they both open to arc $DB$.
Yes, $\angle DAB = \angle DCB$, since they both open to arc $DB$.
copeland
2016-03-05 19:46:43
So $\angle DAI = \angle DCB + \angle BAI$.
So $\angle DAI = \angle DCB + \angle BAI$.
copeland
2016-03-05 19:46:50
What's nice about both of those angles?
What's nice about both of those angles?
stan23456
2016-03-05 19:47:57
They are the result of bisected angles
They are the result of bisected angles
19bobhu
2016-03-05 19:47:57
half of angles A and C
half of angles A and C
MikoTennisPro
2016-03-05 19:47:57
they both are created by angle bisectors
they both are created by angle bisectors
ychen
2016-03-05 19:47:57
they are both half of the angles on the original triangle
they are both half of the angles on the original triangle
idomath12345
2016-03-05 19:47:57
they are 1/2 of other angles.
they are 1/2 of other angles.
nosaj
2016-03-05 19:47:57
They're part of a bisected angle.
They're part of a bisected angle.
copeland
2016-03-05 19:48:00
They're both half of angles in our triangles because of the angle bisectors. $\angle DAI = \dfrac 12 \angle C + \dfrac 12 \angle A$.
They're both half of angles in our triangles because of the angle bisectors. $\angle DAI = \dfrac 12 \angle C + \dfrac 12 \angle A$.
copeland
2016-03-05 19:48:01
Can we show that this expression is also equal to $\angle AID$?
Can we show that this expression is also equal to $\angle AID$?
copeland
2016-03-05 19:48:15
Is there anything we know about $\angle AID$?
Is there anything we know about $\angle AID$?
hamup1
2016-03-05 19:48:53
exterior angles
exterior angles
MikoTennisPro
2016-03-05 19:48:53
AID is exterior angle of CIA
AID is exterior angle of CIA
sxu
2016-03-05 19:48:53
yah exterior angle
yah exterior angle
mewtwomew
2016-03-05 19:48:53
exterior angle of triangle cia
exterior angle of triangle cia
Ericaops
2016-03-05 19:48:53
exterior angle
exterior angle
copeland
2016-03-05 19:48:58
$\angle AID$ is external to triangle $AIC$. So $\angle AID = \angle ICA + \angle IAC$. Again, since $AI$ and $CI$ are angle bisectors, that is $\dfrac 12 C + \dfrac 12 A$.
$\angle AID$ is external to triangle $AIC$. So $\angle AID = \angle ICA + \angle IAC$. Again, since $AI$ and $CI$ are angle bisectors, that is $\dfrac 12 C + \dfrac 12 A$.
copeland
2016-03-05 19:48:59
So $ADI$ is isosceles.
So $ADI$ is isosceles.
goodbear
2016-03-05 19:49:07
AID=180-(180-(A/2+C/2))=A/2+C/2
AID=180-(180-(A/2+C/2))=A/2+C/2
copeland
2016-03-05 19:49:11
By the way, the fact we just prove is really really well known. It's part of what's known colloquially as "Fact 5". So it really helps to know your common geometry configurations -- knowing that fact would have saved us quite a bit of time here.
By the way, the fact we just prove is really really well known. It's part of what's known colloquially as "Fact 5". So it really helps to know your common geometry configurations -- knowing that fact would have saved us quite a bit of time here.
copeland
2016-03-05 19:49:21
Let's simplify our diagram a bit now that we have that.
Let's simplify our diagram a bit now that we have that.
copeland
2016-03-05 19:49:22
copeland
2016-03-05 19:49:40
Cool. Now we have a lot more known lengths. We also have a lot of equal angles from the equal arcs. What can you find?
Cool. Now we have a lot more known lengths. We also have a lot of equal angles from the equal arcs. What can you find?
MikoTennisPro
2016-03-05 19:50:44
similar triangles!!!!
similar triangles!!!!
nosaj
2016-03-05 19:50:44
also, in cyclic quadrilateral, there are a lot of similar triangles!
also, in cyclic quadrilateral, there are a lot of similar triangles!
eswa2000
2016-03-05 19:50:44
similar triangles
similar triangles
LighteningAB
2016-03-05 19:50:44
similar triangle lengths which match up
similar triangle lengths which match up
copeland
2016-03-05 19:51:07
Great! Do you see any equal angles near our known lengths?
Great! Do you see any equal angles near our known lengths?
brian6liu
2016-03-05 19:53:34
<ACI=<LAD
<ACI=<LAD
Aragorn66
2016-03-05 19:53:34
CBA, CDA
CBA, CDA
hliu70
2016-03-05 19:53:34
<CDA and <CBA
<CDA and <CBA
ThorJames
2016-03-05 19:53:34
<bcd = <bad
<bcd = <bad
SimonSun
2016-03-05 19:53:34
ADI AND ABC
ADI AND ABC
idomath12345
2016-03-05 19:53:34
ABD and ACD.
ABD and ACD.
copeland
2016-03-05 19:53:50
So there are a lot and that is 6 different approaches to this argument.
So there are a lot and that is 6 different approaches to this argument.
copeland
2016-03-05 19:53:59
Here is one:
Here is one:
copeland
2016-03-05 19:54:01
We know that $\angle DBA = \angle DCB$ since arc $AD$ equals arc $BD$.
We know that $\angle DBA = \angle DCB$ since arc $AD$ equals arc $BD$.
copeland
2016-03-05 19:54:02
copeland
2016-03-05 19:54:06
What does that tell us?
What does that tell us?
Locust
2016-03-05 19:55:44
Triangles LBD and BCD are similar
Triangles LBD and BCD are similar
jfurf
2016-03-05 19:55:44
$\triangle{CDB}\sim\triangle{BDL}$
$\triangle{CDB}\sim\triangle{BDL}$
brian22
2016-03-05 19:55:44
$\triangle BLD \sim \triangle CBD$
$\triangle BLD \sim \triangle CBD$
sxu
2016-03-05 19:55:44
DCB similar to DBL
DCB similar to DBL
brian6liu
2016-03-05 19:55:44
BLD~CBD
BLD~CBD
SimonSun
2016-03-05 19:55:44
BDL similar CDB
BDL similar CDB
copeland
2016-03-05 19:55:48
This tells us that $\triangle DBL \sim \triangle DCB$ since they share the angle at $D$.
This tells us that $\triangle DBL \sim \triangle DCB$ since they share the angle at $D$.
copeland
2016-03-05 19:55:51
So?
So?
nosaj
2016-03-05 19:57:04
Now we can get $CD$
Now we can get $CD$
Locust
2016-03-05 19:57:04
brian6liu
2016-03-05 19:57:04
CD=25/3
CD=25/3
MikoTennisPro
2016-03-05 19:57:04
3/5 = 5/(5+x)
3/5 = 5/(5+x)
kikipet
2016-03-05 19:57:04
(IC + 5)/5 = 5/3
(IC + 5)/5 = 5/3
ompatel99
2016-03-05 19:57:04
DL/DB=3/5=DB/DC=5/(5+?)
DL/DB=3/5=DB/DC=5/(5+?)
goodbear
2016-03-05 19:57:04
CD=25/3
CD=25/3
brian22
2016-03-05 19:57:04
$\frac{3}{5}=\frac{5}{5+CI}$
$\frac{3}{5}=\frac{5}{5+CI}$
atmchallenge
2016-03-05 19:57:04
$\frac{BD}{DL} = \frac{CD}{BD} \implies 25 = 3 CD \implies CD=\frac{25}{3}$
$\frac{BD}{DL} = \frac{CD}{BD} \implies 25 = 3 CD \implies CD=\frac{25}{3}$
blizzard10
2016-03-05 19:57:04
Therefore CD = $BD \cdot\frac{5}{3}$
Therefore CD = $BD \cdot\frac{5}{3}$
memc38123
2016-03-05 19:57:04
5/(IC + 5) = 3/5
5/(IC + 5) = 3/5
copeland
2016-03-05 19:57:07
That means that $\dfrac{DC}{DB} = \dfrac{DB}{DL}$. But we already know some of those...
That means that $\dfrac{DC}{DB} = \dfrac{DB}{DL}$. But we already know some of those...
copeland
2016-03-05 19:57:11
We have $\dfrac{5 + IC}{5} = \dfrac{5}{3}$.
We have $\dfrac{5 + IC}{5} = \dfrac{5}{3}$.
copeland
2016-03-05 19:57:16
So what is $IC$?
So what is $IC$?
blizzard10
2016-03-05 19:57:39
Which equals $\frac{10}{3}$.
Which equals $\frac{10}{3}$.
brian6liu
2016-03-05 19:57:39
10/3
10/3
ompatel99
2016-03-05 19:57:39
10/3
10/3
SmallKid
2016-03-05 19:57:39
10/3?
10/3?
tennis1729
2016-03-05 19:57:39
10/3
10/3
SimonSun
2016-03-05 19:57:39
10/3
10/3
jfurf
2016-03-05 19:57:39
$\frac{10}{3}$
$\frac{10}{3}$
idomath12345
2016-03-05 19:57:39
10/3
10/3
Ericaops
2016-03-05 19:57:39
10/3
10/3
ychen
2016-03-05 19:57:39
10/3
10/3
copeland
2016-03-05 19:57:41
We get $IC = \dfrac{10}{3}$.
We get $IC = \dfrac{10}{3}$.
copeland
2016-03-05 19:57:46
So what is our answer?
So what is our answer?
Aragorn66
2016-03-05 19:58:06
013
013
goodbear
2016-03-05 19:58:06
10/3,so 013
10/3,so 013
walnutwaldo20
2016-03-05 19:58:06
013
013
blue8931
2016-03-05 19:58:06
13
13
ychen
2016-03-05 19:58:06
013
013
popcorn1
2016-03-05 19:58:06
$013$
$013$
epiphany
2016-03-05 19:58:06
013
013
weilunsun28
2016-03-05 19:58:06
013
013
andsun19
2016-03-05 19:58:06
13
13
SK200
2016-03-05 19:58:06
013
013
copeland
2016-03-05 19:58:15
Our answer is $10+3=\boxed{13}.$
Our answer is $10+3=\boxed{13}.$
brian22
2016-03-05 19:58:59
Wait darn how did you add 10 and 3 so fast
Wait darn how did you add 10 and 3 so fast
copeland
2016-03-05 19:59:03
I converted to 1010+11 base 2 since the digits are smaller and easier to handle.
I converted to 1010+11 base 2 since the digits are smaller and easier to handle.
idomath12345
2016-03-05 19:59:20
Oh!!!!
Oh!!!!
walnutwaldo20
2016-03-05 19:59:20
Ah that makes sense
Ah that makes sense
LighteningAB
2016-03-05 19:59:24
YAY ONTO SEVEN
YAY ONTO SEVEN
popcorn1
2016-03-05 19:59:24
Time for #$7$
Time for #$7$
copeland
2016-03-05 19:59:27
7. For integers $a$ and $b$ consider the complex number \[ \frac{\sqrt{ab + 2016}}{ab + 100} - \left( \frac{\sqrt{|a + b|}}{ab + 100}\right) i. \] Find the number of ordered pairs of integers $(a,b)$ such that this complex number is a real number.
7. For integers $a$ and $b$ consider the complex number \[ \frac{\sqrt{ab + 2016}}{ab + 100} - \left( \frac{\sqrt{|a + b|}}{ab + 100}\right) i. \] Find the number of ordered pairs of integers $(a,b)$ such that this complex number is a real number.
copeland
2016-03-05 19:59:39
How can we make this number real?
How can we make this number real?
copeland
2016-03-05 19:59:59
Is it enough to have $\left( \dfrac{\sqrt{|a + b|}}{ab + 100}\right) i = 0$?
Is it enough to have $\left( \dfrac{\sqrt{|a + b|}}{ab + 100}\right) i = 0$?
qwerty733
2016-03-05 20:00:45
imaginary part is 0, real part is real
imaginary part is 0, real part is real
blizzard10
2016-03-05 20:00:45
There are two options: the second part equals 0, or both are imaginary and they cancel out.
There are two options: the second part equals 0, or both are imaginary and they cancel out.
memc38123
2016-03-05 20:00:45
nooooo! You can have the imaginary parts cancel
nooooo! You can have the imaginary parts cancel
brian6liu
2016-03-05 20:00:45
or the first term can equal the second
or the first term can equal the second
sxu
2016-03-05 20:00:45
no the first part must not be imaginary
no the first part must not be imaginary
Peggy
2016-03-05 20:00:45
no, because the thing inside the sq root has to be positive
no, because the thing inside the sq root has to be positive
walnutwaldo20
2016-03-05 20:00:45
No, left has to be real
No, left has to be real
qwerty733
2016-03-05 20:00:45
no "real" part must be real (i.e., sqrt geq 0)
no "real" part must be real (i.e., sqrt geq 0)
copeland
2016-03-05 20:00:48
No, we also need $\sqrt{ab + 2016}$ to be real in that case.
No, we also need $\sqrt{ab + 2016}$ to be real in that case.
copeland
2016-03-05 20:00:58
Apparently the real part is allowed to be non-real here. . .
Apparently the real part is allowed to be non-real here. . .
copeland
2016-03-05 20:01:10
First things first.
First things first.
copeland
2016-03-05 20:01:12
If $\left( \dfrac{\sqrt{|a + b|}}{ab + 100}\right) i = 0$, what can we say about $a$ and $b$?
If $\left( \dfrac{\sqrt{|a + b|}}{ab + 100}\right) i = 0$, what can we say about $a$ and $b$?
blizzard10
2016-03-05 20:01:49
$a + b = 0$
$a + b = 0$
sxu
2016-03-05 20:01:49
theyr opposites
theyr opposites
ryanyz10
2016-03-05 20:01:49
a = -b
a = -b
algebra_star1234
2016-03-05 20:01:49
a=-b
a=-b
fishy15
2016-03-05 20:01:49
a=-b
a=-b
memc38123
2016-03-05 20:01:49
a = -b
a = -b
qwerty733
2016-03-05 20:01:49
a=-b
a=-b
bestwillcui1
2016-03-05 20:01:49
they are negativs of each other
they are negativs of each other
SuperMaltese
2016-03-05 20:01:49
a = -b
a = -b
ChickenOnRage
2016-03-05 20:01:49
a = -b
a = -b
copeland
2016-03-05 20:01:52
We must have $a = - b$. So our other radical becomes $\sqrt{2016 - a^2}$. When is that real?
We must have $a = - b$. So our other radical becomes $\sqrt{2016 - a^2}$. When is that real?
memc38123
2016-03-05 20:02:19
when a^2 < 2016
when a^2 < 2016
sxu
2016-03-05 20:02:19
when a^2<2016
when a^2<2016
person754
2016-03-05 20:02:19
a^2<2016
a^2<2016
epiphany
2016-03-05 20:02:19
when a^2 is less than 2016
when a^2 is less than 2016
ChickenOnRage
2016-03-05 20:02:19
when $a^2 <= 2016$
when $a^2 <= 2016$
walnutwaldo20
2016-03-05 20:02:19
when a^2 < 2016
when a^2 < 2016
copeland
2016-03-05 20:02:21
That's real if $a^2 \le 2016$. How many $a$ are there like that?
That's real if $a^2 \le 2016$. How many $a$ are there like that?
SuperMaltese
2016-03-05 20:02:44
89
89
andsun19
2016-03-05 20:02:44
-44 to 44, so 89
-44 to 44, so 89
fz2012
2016-03-05 20:02:44
89
89
nukelauncher
2016-03-05 20:02:44
89
89
ninjataco
2016-03-05 20:02:44
89
89
nosaj
2016-03-05 20:02:44
-44 to 44
-44 to 44
Locust
2016-03-05 20:02:44
For integers, -44 through 44
For integers, -44 through 44
nosaj
2016-03-05 20:02:44
89
89
copeland
2016-03-05 20:02:45
$a^2 \le 2016$ if $|a| \le 44$. So are there 89 solutions in this case?
$a^2 \le 2016$ if $|a| \le 44$. So are there 89 solutions in this case?
MikoTennisPro
2016-03-05 20:03:10
-44 to 44 => 89 but can't be -10 or 10 b/c then denominator is 0
-44 to 44 => 89 but can't be -10 or 10 b/c then denominator is 0
ninjataco
2016-03-05 20:03:10
89 but denominator needs to be nonzero so 87
89 but denominator needs to be nonzero so 87
MikoTennisPro
2016-03-05 20:03:10
87
87
ninjataco
2016-03-05 20:03:10
no, a is not +/- 10
no, a is not +/- 10
andsun19
2016-03-05 20:03:10
nope., ab is not equal to -100
nope., ab is not equal to -100
Locust
2016-03-05 20:03:10
no, a and b cannot be a combination of 10,-10
no, a and b cannot be a combination of 10,-10
fz2012
2016-03-05 20:03:10
no, a=+-10 does not work
no, a=+-10 does not work
ompatel99
2016-03-05 20:03:10
But remember that ab cannot equal -100. So a cannot be 10 or -10. 87 solutions
But remember that ab cannot equal -100. So a cannot be 10 or -10. 87 solutions
azmath333
2016-03-05 20:03:10
ab cant be -100
ab cant be -100
copeland
2016-03-05 20:03:12
Not quite! Some of those solutions make our denominator $ab + 100 = 0$.
Not quite! Some of those solutions make our denominator $ab + 100 = 0$.
copeland
2016-03-05 20:03:13
We have to skip $a = -10$ and $a = 10$. For a total of 87 solutions here.
We have to skip $a = -10$ and $a = 10$. For a total of 87 solutions here.
copeland
2016-03-05 20:03:17
Okay, so our answer is 87?
Okay, so our answer is 87?
ChickenOnRage
2016-03-05 20:03:42
no
no
blizzard10
2016-03-05 20:03:42
No!
No!
hliu70
2016-03-05 20:03:42
NOOOO
NOOOO
kikipet
2016-03-05 20:03:42
no
no
nukelauncher
2016-03-05 20:03:42
no one more case
no one more case
ChickenOnRage
2016-03-05 20:03:42
no; the case when the entire expression evaluates to 0
no; the case when the entire expression evaluates to 0
bestwillcui1
2016-03-05 20:03:42
NO!
NO!
blizzard10
2016-03-05 20:03:42
There's a second part to consider. Both could be imaginary and cancel out.
There's a second part to consider. Both could be imaginary and cancel out.
mssmath
2016-03-05 20:03:44
Apparently stuff can cancel?
Apparently stuff can cancel?
copeland
2016-03-05 20:03:47
No! We can also cancel the imaginary parts from both terms. That is, we can also have $\sqrt{ab+2016} = i\sqrt{|a+b|}$.
No! We can also cancel the imaginary parts from both terms. That is, we can also have $\sqrt{ab+2016} = i\sqrt{|a+b|}$.
copeland
2016-03-05 20:03:51
Squaring both sides, we need to have $ab + 2016 = a + b$ (with $a + b < 0$) or $ab + 2016 = -a - b$ (with $a + b > 0$).
Squaring both sides, we need to have $ab + 2016 = a + b$ (with $a + b < 0$) or $ab + 2016 = -a - b$ (with $a + b > 0$).
copeland
2016-03-05 20:03:53
What can we do with those?
What can we do with those?
atmchallenge
2016-03-05 20:04:22
SFFT
SFFT
ompatel99
2016-03-05 20:04:22
SFFT
SFFT
jfurf
2016-03-05 20:04:22
SFFT!!!
SFFT!!!
MikoTennisPro
2016-03-05 20:04:22
SFFT?
SFFT?
brian6liu
2016-03-05 20:04:22
sfft
sfft
Math1331Math
2016-03-05 20:04:22
SFFT
SFFT
SimonSun
2016-03-05 20:04:22
SFFT
SFFT
qwerty733
2016-03-05 20:04:22
SFFT
SFFT
blue8931
2016-03-05 20:04:22
SFFT
SFFT
andsun19
2016-03-05 20:04:22
sfft
sfft
nosaj
2016-03-05 20:04:22
I smell sfft
I smell sfft
fz2012
2016-03-05 20:04:22
use SFFT and solve?
use SFFT and solve?
copeland
2016-03-05 20:04:27
SFFT! This is a classic set-up for Simon's favorite factoring trick: If you see terms like $xy, x,$ and $y$ in your equation, you probably want to think about using Simon's.
SFFT! This is a classic set-up for Simon's favorite factoring trick: If you see terms like $xy, x,$ and $y$ in your equation, you probably want to think about using Simon's.
copeland
2016-03-05 20:04:31
Here's a link: http://www.artofproblemsolving.com/wiki/index.php/Simon's_Favorite_Factoring_Trick
Here's a link: http://www.artofproblemsolving.com/wiki/index.php/Simon's_Favorite_Factoring_Trick
copeland
2016-03-05 20:04:34
Let's deal with $ab + 2016 = a + b$ first. How does this factor?
Let's deal with $ab + 2016 = a + b$ first. How does this factor?
brian6liu
2016-03-05 20:05:15
(a-1)(b-1)=-2015
(a-1)(b-1)=-2015
algebra_star1234
2016-03-05 20:05:15
(a-1)(b-1) = -2015
(a-1)(b-1) = -2015
andsun19
2016-03-05 20:05:15
(a-1)(b-1)=2015
(a-1)(b-1)=2015
blizzard10
2016-03-05 20:05:15
$(a-1)(b-1) + 2015 = 0$
$(a-1)(b-1) + 2015 = 0$
mathguy5041
2016-03-05 20:05:15
$(a-1)(b-1)=5*13*31$
$(a-1)(b-1)=5*13*31$
kikipet
2016-03-05 20:05:15
(a-1)(b-1)=-2015
(a-1)(b-1)=-2015
memc38123
2016-03-05 20:05:15
(A-1)(b-1)=2015
(A-1)(b-1)=2015
brian22
2016-03-05 20:05:15
$(a-1)(b-1)=-2015$
$(a-1)(b-1)=-2015$
SuperMaltese
2016-03-05 20:05:15
(a-1)(b-1) = -2015
(a-1)(b-1) = -2015
copeland
2016-03-05 20:05:18
This factors as $(a-1)(b-1) = - 2015 = - 5 \cdot 13 \cdot 31$.
This factors as $(a-1)(b-1) = - 2015 = - 5 \cdot 13 \cdot 31$.
copeland
2016-03-05 20:05:19
How many such pairs are there?
How many such pairs are there?
tennis1729
2016-03-05 20:06:13
8
8
ninjataco
2016-03-05 20:06:13
8
8
andsun19
2016-03-05 20:06:13
8
8
idomath12345
2016-03-05 20:06:13
8.
8.
person754
2016-03-05 20:06:13
8
8
ChickenOnRage
2016-03-05 20:06:13
8
8
memc38123
2016-03-05 20:06:13
8
8
idomath12345
2016-03-05 20:06:13
But 1/2 don't work.
But 1/2 don't work.
qwerty733
2016-03-05 20:06:13
8
8
Locust
2016-03-05 20:06:13
8, but some of them don't satisfy a+b > 0
8, but some of them don't satisfy a+b > 0
atmchallenge
2016-03-05 20:06:13
$8$ since $a+b<0$
$8$ since $a+b<0$
copeland
2016-03-05 20:06:18
Since we must have $a + b < 0$, the signs are already determined (the negative must go on the bigger factor). So we simply need to assign 5, 13, and 31 to $a$ and $b$.
Since we must have $a + b < 0$, the signs are already determined (the negative must go on the bigger factor). So we simply need to assign 5, 13, and 31 to $a$ and $b$.
copeland
2016-03-05 20:06:20
There are 2 options for where we put the 5, 2 options for the 13, and again 2 options for the 31. That gives us $2\cdot 2 \cdot 2 = 8$ options.
There are 2 options for where we put the 5, 2 options for the 13, and again 2 options for the 31. That gives us $2\cdot 2 \cdot 2 = 8$ options.
copeland
2016-03-05 20:06:22
Okay, now let's deal with the other case: $ab + 2016 = -a - b$. How does this factor?
Okay, now let's deal with the other case: $ab + 2016 = -a - b$. How does this factor?
brian22
2016-03-05 20:06:53
$(a+1)(b+1)=-2015$
$(a+1)(b+1)=-2015$
atmchallenge
2016-03-05 20:06:53
$(a+1)(b+1)=-2015$
$(a+1)(b+1)=-2015$
brian6liu
2016-03-05 20:06:53
(a+1)(b+1)=-2015
(a+1)(b+1)=-2015
memc38123
2016-03-05 20:06:53
(a+1)(b+1)=-2015
(a+1)(b+1)=-2015
ninjataco
2016-03-05 20:06:53
(a+1)(b+1) = -2015
(a+1)(b+1) = -2015
algebra_star1234
2016-03-05 20:06:53
(a+1)(b+1)=-2015
(a+1)(b+1)=-2015
kikipet
2016-03-05 20:06:53
(a+1)(b+1)=-2015
(a+1)(b+1)=-2015
copeland
2016-03-05 20:06:57
This factors as $(a + 1)(b+1) = -2015 = -5 \cdot 13 \cdot 31$.
This factors as $(a + 1)(b+1) = -2015 = -5 \cdot 13 \cdot 31$.
copeland
2016-03-05 20:06:57
How many such pairs are there?
How many such pairs are there?
epiphany
2016-03-05 20:07:34
8
8
idomath12345
2016-03-05 20:07:34
8 again.
8 again.
popcorn1
2016-03-05 20:07:34
8
8
nukelauncher
2016-03-05 20:07:34
8
8
brian22
2016-03-05 20:07:34
Same, $8$
Same, $8$
fz2012
2016-03-05 20:07:34
8
8
atmchallenge
2016-03-05 20:07:34
$8$ again, since $a+b>0$
$8$ again, since $a+b>0$
brian6liu
2016-03-05 20:07:34
8
8
jfurf
2016-03-05 20:07:34
$8$ again! Still have to watch out for the conditions, though :)
$8$ again! Still have to watch out for the conditions, though :)
andsun19
2016-03-05 20:07:34
8
8
copeland
2016-03-05 20:07:37
Since we must have $a + b > 0$, the signs are already determined (the negative must go on the smaller factor). So we simply need to assign 5, 13, and 31 to $a$ and $b$.
Since we must have $a + b > 0$, the signs are already determined (the negative must go on the smaller factor). So we simply need to assign 5, 13, and 31 to $a$ and $b$.
copeland
2016-03-05 20:07:39
Just like before, there are 8 ways to do that.
Just like before, there are 8 ways to do that.
copeland
2016-03-05 20:07:41
So, how many pairs are there in total?
So, how many pairs are there in total?
fz2012
2016-03-05 20:08:16
so there are a total of 87+8+8=103 solutions
so there are a total of 87+8+8=103 solutions
memc38123
2016-03-05 20:08:16
103
103
algebra_star1234
2016-03-05 20:08:16
103
103
mathguy5041
2016-03-05 20:08:16
87+8+8=$103$
87+8+8=$103$
nosaj
2016-03-05 20:08:16
103
103
andsun19
2016-03-05 20:08:16
16+87=103?
16+87=103?
kikipet
2016-03-05 20:08:16
$\boxed{103}$
$\boxed{103}$
blue8931
2016-03-05 20:08:16
103
103
person754
2016-03-05 20:08:16
103
103
atmchallenge
2016-03-05 20:08:16
$87+8+8=\boxed{103}$.
$87+8+8=\boxed{103}$.
walnutwaldo20
2016-03-05 20:08:16
16+87 = 103
16+87 = 103
copeland
2016-03-05 20:08:18
In total, we have $87 + 8 + 8 = \boxed{103}$ pairs.
In total, we have $87 + 8 + 8 = \boxed{103}$ pairs.
copeland
2016-03-05 20:08:23
And as a public service announcement: Please do not learn anything about square roots from this problem. You should never, ever take the square root of a negative number and then just assume something about its sign: Any time you write $\sqrt{-x}=i\sqrt x$, you are very likely to make a huge mess. This is because you quickly get into situations like: \[\sqrt{x}=\sqrt{-(-x)}=i\sqrt{-x}=i^2\sqrt{x}=-\sqrt{x}.\]
And as a public service announcement: Please do not learn anything about square roots from this problem. You should never, ever take the square root of a negative number and then just assume something about its sign: Any time you write $\sqrt{-x}=i\sqrt x$, you are very likely to make a huge mess. This is because you quickly get into situations like: \[\sqrt{x}=\sqrt{-(-x)}=i\sqrt{-x}=i^2\sqrt{x}=-\sqrt{x}.\]
copeland
2016-03-05 20:08:35
In fact, you are typically better off thinking about such an expression as having two values (like in the quadratic formula), and in that case, this problem deserves a footnote that we're looking for $a$ and $b$ where one of the values is zero.
In fact, you are typically better off thinking about such an expression as having two values (like in the quadratic formula), and in that case, this problem deserves a footnote that we're looking for $a$ and $b$ where one of the values is zero.
fishy15
2016-03-05 20:09:08
the problems are getting longer
the problems are getting longer
copeland
2016-03-05 20:09:10
I'm more worried about the solutions.
I'm more worried about the solutions.
copeland
2016-03-05 20:09:17
8. For a permutation $p = (a_1, a_2, \ldots, a_9)$ of the digits $1,2,\ldots, 9$, let $s(p)$ denote the sum of the three 3-digit numbers $a_1a_2a_3, a_4a_5a_6,$ and $a_7a_8a_9$. Let $m$ be the minimum value of $s(p)$ subject to the condition that the units digit of $s(p)$ is 0. Let $n$ denote the number of permutations $p$ with $s(p) = m$. Find $|m - n|$.
8. For a permutation $p = (a_1, a_2, \ldots, a_9)$ of the digits $1,2,\ldots, 9$, let $s(p)$ denote the sum of the three 3-digit numbers $a_1a_2a_3, a_4a_5a_6,$ and $a_7a_8a_9$. Let $m$ be the minimum value of $s(p)$ subject to the condition that the units digit of $s(p)$ is 0. Let $n$ denote the number of permutations $p$ with $s(p) = m$. Find $|m - n|$.
copeland
2016-03-05 20:09:35
Case in point: this is a long problem.
Case in point: this is a long problem.
copeland
2016-03-05 20:09:36
It's usually a good plan to deal with restrictive conditions first. What's a restrictive condition here?
It's usually a good plan to deal with restrictive conditions first. What's a restrictive condition here?
idomath12345
2016-03-05 20:10:25
Go for the smallest.
Go for the smallest.
blue8931
2016-03-05 20:10:25
units digit is 0
units digit is 0
sxu
2016-03-05 20:10:25
units digit is0
units digit is0
janabel
2016-03-05 20:10:25
the units digit is 0
the units digit is 0
qwerty733
2016-03-05 20:10:25
units digit is 0
units digit is 0
MikoTennisPro
2016-03-05 20:10:25
units digit is zero
units digit is zero
algebra_star1234
2016-03-05 20:10:25
the units digit is 0
the units digit is 0
simon1221
2016-03-05 20:10:25
units digit = 0?
units digit = 0?
snowumbrella
2016-03-05 20:10:25
units digit of s(p) must be 0
units digit of s(p) must be 0
illogical_21
2016-03-05 20:10:25
units digit is 0
units digit is 0
copeland
2016-03-05 20:10:28
We probably want to deal with the fact that the units digit of $s(p)$ is 0 first. What does that tell us about $p$?
We probably want to deal with the fact that the units digit of $s(p)$ is 0 first. What does that tell us about $p$?
brian22
2016-03-05 20:11:17
$s(p) \pmod{10} \equiv 0$
$s(p) \pmod{10} \equiv 0$
hamup1
2016-03-05 20:11:17
$a_3+a_6+a_9=10$ or $a_3+a_6+a_9=20$
$a_3+a_6+a_9=10$ or $a_3+a_6+a_9=20$
blue8931
2016-03-05 20:11:17
so you know that $a_3 + a_6 + a_9 = 10$ or $20$
so you know that $a_3 + a_6 + a_9 = 10$ or $20$
brian22
2016-03-05 20:11:17
$a_3+a_6+a_9=10$ or $20$
$a_3+a_6+a_9=10$ or $20$
memc38123
2016-03-05 20:11:17
a(3)+a(6)+a(9) = 10k
a(3)+a(6)+a(9) = 10k
copeland
2016-03-05 20:11:20
Since \[a_1a_2a_3 + a_4a_5a_6 + a_7a_8a_9 = 100(a_1 + a_4 + a_7) + 10(a_2 + a_5 + a_8) + (a_3 + a_6 + a_9),\] this tells us that $a_3 + a_6 + a_9$ must be a multiple of 10.
Since \[a_1a_2a_3 + a_4a_5a_6 + a_7a_8a_9 = 100(a_1 + a_4 + a_7) + 10(a_2 + a_5 + a_8) + (a_3 + a_6 + a_9),\] this tells us that $a_3 + a_6 + a_9$ must be a multiple of 10.
copeland
2016-03-05 20:11:21
Can we get $a_3 + a_6 + a_9 = 10$?
Can we get $a_3 + a_6 + a_9 = 10$?
blue8931
2016-03-05 20:12:39
$(7, 2, 1), (6, 3, 1), (5, 4, 1),$ or $(5, 3, 2)$
$(7, 2, 1), (6, 3, 1), (5, 4, 1),$ or $(5, 3, 2)$
memc38123
2016-03-05 20:12:39
2+3+5
2+3+5
Aragorn66
2016-03-05 20:12:39
4,5,1
4,5,1
atmchallenge
2016-03-05 20:12:39
$1+2+7=1+3+6=1+4+5=2+3+5=10$
$1+2+7=1+3+6=1+4+5=2+3+5=10$
copeland
2016-03-05 20:12:42
Of course. For example, 2+3+5=10.
Of course. For example, 2+3+5=10.
copeland
2016-03-05 20:12:46
Can we get $a_3 + a_6 + a_9 = 20$?
Can we get $a_3 + a_6 + a_9 = 20$?
brian22
2016-03-05 20:13:39
Yes: $(3,8,9)$, $(4,7,9)$, $(5,6,9)$, $(5,7,8)$
Yes: $(3,8,9)$, $(4,7,9)$, $(5,6,9)$, $(5,7,8)$
epiphany
2016-03-05 20:13:39
yes, 9+8+3, 9+7+4, and 8+7+5
yes, 9+8+3, 9+7+4, and 8+7+5
qwerty733
2016-03-05 20:13:39
Yes, e.g. 9+8+3
Yes, e.g. 9+8+3
hamup1
2016-03-05 20:13:39
9+8+3
9+8+3
atmchallenge
2016-03-05 20:13:39
$9+8+3=9+7+4=9+6+5=8+7+5=20$
$9+8+3=9+7+4=9+6+5=8+7+5=20$
brian6liu
2016-03-05 20:13:39
389, 479, 569, 578
389, 479, 569, 578
mssmath
2016-03-05 20:13:39
9+7+4
9+7+4
snowumbrella
2016-03-05 20:13:39
(9, 8, 3) (9, 7, 4) (9, 6, 5) (8, 7, 5)
(9, 8, 3) (9, 7, 4) (9, 6, 5) (8, 7, 5)
copeland
2016-03-05 20:13:43
Yes. There are several ways.
Yes. There are several ways.
copeland
2016-03-05 20:13:45
Up to permutations, the four ways to get 20 are for $(a_3,a_6,a_9)$ to be:
\begin{align*}
&(9,8,3) &(8,7,5) \\
&(9,7,4) \\
&(9,6,5)
\end{align*}
Up to permutations, the four ways to get 20 are for $(a_3,a_6,a_9)$ to be:
\begin{align*}
&(9,8,3) &(8,7,5) \\
&(9,7,4) \\
&(9,6,5)
\end{align*}
copeland
2016-03-05 20:13:47
Can we get $a_3 + a_6 + a_9 = 30$?
Can we get $a_3 + a_6 + a_9 = 30$?
tennis1729
2016-03-05 20:14:03
no
no
Locust
2016-03-05 20:14:03
No
No
calculus_riju
2016-03-05 20:14:03
no
no
weilunsun28
2016-03-05 20:14:03
nope
nope
jfurf
2016-03-05 20:14:03
No
No
kikipet
2016-03-05 20:14:03
no, it's too big
no, it's too big
andsun19
2016-03-05 20:14:03
nope.
nope.
Ericaops
2016-03-05 20:14:03
no
no
thequantumguy
2016-03-05 20:14:03
no
no
GoldenPhi1618033
2016-03-05 20:14:03
No.
No.
19bobhu
2016-03-05 20:14:03
no
no
SimonSun
2016-03-05 20:14:03
no
no
blizzard10
2016-03-05 20:14:03
No!
No!
copeland
2016-03-05 20:14:05
No, that's too big. The biggest $a_3 + a_6 + a_9$ can be is 9+8+7 =24.
No, that's too big. The biggest $a_3 + a_6 + a_9$ can be is 9+8+7 =24.
copeland
2016-03-05 20:14:07
Okay, what's the next restrictive condition we care about?
Okay, what's the next restrictive condition we care about?
andsun19
2016-03-05 20:14:26
MINIMUM
MINIMUM
snowumbrella
2016-03-05 20:14:26
m must be minimum
m must be minimum
person754
2016-03-05 20:14:26
minimum value
minimum value
weilunsun28
2016-03-05 20:14:26
minimize m
minimize m
janabel
2016-03-05 20:14:26
minimum
minimum
blue8931
2016-03-05 20:14:26
we want the minimum value
we want the minimum value
copeland
2016-03-05 20:14:29
The next interesting restriction is that we need our permutation to achieve the minimum sum.
The next interesting restriction is that we need our permutation to achieve the minimum sum.
copeland
2016-03-05 20:14:33
How are we going to minimize \[a_1a_2a_3 + a_4a_5a_6 + a_7a_8a_9 = 100(a_1 + a_4 + a_7) + 10(a_2 + a_5 + a_8) + (a_3 + a_6 + a_9)?\]
How are we going to minimize \[a_1a_2a_3 + a_4a_5a_6 + a_7a_8a_9 = 100(a_1 + a_4 + a_7) + 10(a_2 + a_5 + a_8) + (a_3 + a_6 + a_9)?\]
mathguy5041
2016-03-05 20:15:08
Make hundreds digits $1, 2, 3$
Make hundreds digits $1, 2, 3$
qwerty733
2016-03-05 20:15:08
Minimize hundreds digits
Minimize hundreds digits
andsun19
2016-03-05 20:15:08
well, a1 + a4 + a7, should be minimized
well, a1 + a4 + a7, should be minimized
19bobhu
2016-03-05 20:15:08
make a1 a4 and a7 as small as possible?
make a1 a4 and a7 as small as possible?
jfurf
2016-03-05 20:15:08
Minimize $a_1+a_4+a_7$, first.
Minimize $a_1+a_4+a_7$, first.
brian6liu
2016-03-05 20:15:08
minimize a_1+a_4+a_7
minimize a_1+a_4+a_7
kikipet
2016-03-05 20:15:08
make $a_1, a_2, a_3$ be 1, 2, and 3
make $a_1, a_2, a_3$ be 1, 2, and 3
Locust
2016-03-05 20:15:08
first minimize $a_1,a_4,a_7$
first minimize $a_1,a_4,a_7$
fishy15
2016-03-05 20:15:08
minimize a_1, a_4, and a_7
minimize a_1, a_4, and a_7
nosaj
2016-03-05 20:15:08
hundreds should be 1,2,3
hundreds should be 1,2,3
math1012
2016-03-05 20:15:08
make a_1, a_4, a_7 all 1,2,3
make a_1, a_4, a_7 all 1,2,3
nukelauncher
2016-03-05 20:15:08
a(1) + a(4) +a(7) has to be 1, 2, and 3
a(1) + a(4) +a(7) has to be 1, 2, and 3
copeland
2016-03-05 20:15:14
We'll need to make $a_1+a_4+a_7$ as small as possible. Up to permutations, we must have $(a_1,a_4,a_7) = (1,2,3)$.
We'll need to make $a_1+a_4+a_7$ as small as possible. Up to permutations, we must have $(a_1,a_4,a_7) = (1,2,3)$.
copeland
2016-03-05 20:15:17
Note that replacing any of 1,2,3 with a number $k > 3$ will swap more 100's for fewer 10's or 1's, thus increasing the sum.
Note that replacing any of 1,2,3 with a number $k > 3$ will swap more 100's for fewer 10's or 1's, thus increasing the sum.
copeland
2016-03-05 20:16:03
Now if we use 1, 2, and 3 in the hundreds digit, we can't make 10 from the ones digit since we would need a or or a 2 for that. So we'll have to have $a_3+a_6+a_9 = 20$.
Now if we use 1, 2, and 3 in the hundreds digit, we can't make 10 from the ones digit since we would need a or or a 2 for that. So we'll have to have $a_3+a_6+a_9 = 20$.
copeland
2016-03-05 20:16:05
So what will our minimum be?
So what will our minimum be?
weilunsun28
2016-03-05 20:16:39
810
810
MikoTennisPro
2016-03-05 20:16:39
810
810
jrl_ct
2016-03-05 20:16:39
810
810
qwerty733
2016-03-05 20:16:39
810
810
SimonSun
2016-03-05 20:16:39
810
810
nukelauncher
2016-03-05 20:16:39
810!
810!
brian6liu
2016-03-05 20:16:39
810
810
algebra_star1234
2016-03-05 20:16:39
810
810
calculus_riju
2016-03-05 20:16:39
810
810
ninjataco
2016-03-05 20:16:39
810
810
copeland
2016-03-05 20:16:42
Since $a_1 + \cdots + a_9 = 45$ and $a_1 + a_4 + a_7 = 6$, $a_3 + a_6 + a_9 = 20$, we must have $a_2 + a_5 + a_8 = 19$. So our minimum sum is \[ 100 \cdot 6 + 10 \cdot 19 + 20 = 600 + 190 + 20 = 810.\]
Since $a_1 + \cdots + a_9 = 45$ and $a_1 + a_4 + a_7 = 6$, $a_3 + a_6 + a_9 = 20$, we must have $a_2 + a_5 + a_8 = 19$. So our minimum sum is \[ 100 \cdot 6 + 10 \cdot 19 + 20 = 600 + 190 + 20 = 810.\]
copeland
2016-03-05 20:16:43
How many permutations achieve this sum?
How many permutations achieve this sum?
Aragorn66
2016-03-05 20:17:31
a lot
a lot
copeland
2016-03-05 20:17:35
Indeed. Let's work it out.
Indeed. Let's work it out.
copeland
2016-03-05 20:17:43
How many choices do we have for $(a_1,a_4,a_7)$?
How many choices do we have for $(a_1,a_4,a_7)$?
fz2012
2016-03-05 20:18:08
6
6
MikoTennisPro
2016-03-05 20:18:08
3!
3!
nukelauncher
2016-03-05 20:18:08
3!=6
3!=6
andsun19
2016-03-05 20:18:08
3! = 6
3! = 6
brian6liu
2016-03-05 20:18:08
6
6
blue8931
2016-03-05 20:18:08
3! ways to order them
3! ways to order them
PurplePancakes
2016-03-05 20:18:08
6
6
simon1221
2016-03-05 20:18:08
3! = 6
3! = 6
copeland
2016-03-05 20:18:10
We have 3! choices for $(a_1, a_4, a_7)$ because it must be a permutation of $(1,2,3)$.
We have 3! choices for $(a_1, a_4, a_7)$ because it must be a permutation of $(1,2,3)$.
copeland
2016-03-05 20:18:11
How many choices do we have for $(a_3,a_6,a_9)$?
How many choices do we have for $(a_3,a_6,a_9)$?
PurplePancakes
2016-03-05 20:18:52
6*3
6*3
hamup1
2016-03-05 20:18:52
three different choices, times 6 for permutations
three different choices, times 6 for permutations
nosaj
2016-03-05 20:18:52
3*6 = 18
3*6 = 18
MikoTennisPro
2016-03-05 20:18:52
6*3
6*3
person754
2016-03-05 20:18:52
18
18
calculus_riju
2016-03-05 20:18:52
3.3!
3.3!
andsun19
2016-03-05 20:18:52
(9,7,4)(8,7,5)(5,6,9). So 3*3!
(9,7,4)(8,7,5)(5,6,9). So 3*3!
jwlw2014
2016-03-05 20:18:52
3*3!
3*3!
copeland
2016-03-05 20:18:56
We can use permutations of any of $(9,7,4), (9,6,5), (8,7,5)$. For a total of 3*3!.
We can use permutations of any of $(9,7,4), (9,6,5), (8,7,5)$. For a total of 3*3!.
copeland
2016-03-05 20:18:57
How many choices for $(a_2,a_5,a_8)$?
How many choices for $(a_2,a_5,a_8)$?
mcmcphie
2016-03-05 20:19:32
6
6
MikoTennisPro
2016-03-05 20:19:32
3!, the remaining 3 #s
3!, the remaining 3 #s
mathguy5041
2016-03-05 20:19:32
3!
3!
nukelauncher
2016-03-05 20:19:32
3!=6
3!=6
sxu
2016-03-05 20:19:32
3!'
3!'
idomath12345
2016-03-05 20:19:32
3!
3!
copeland
2016-03-05 20:19:34
At this point, the three numbers available are determined. So we only get to choose their order. We can do that in 3! ways.
At this point, the three numbers available are determined. So we only get to choose their order. We can do that in 3! ways.
copeland
2016-03-05 20:19:35
So in total, we have 3*3!*3!*3! ways to achieve the minimum value.
So in total, we have 3*3!*3!*3! ways to achieve the minimum value.
copeland
2016-03-05 20:19:36
So what is our answer?
So what is our answer?
mewtwomew
2016-03-05 20:20:31
162
162
brian6liu
2016-03-05 20:20:31
162
162
idomath12345
2016-03-05 20:20:31
810-648
810-648
andsun19
2016-03-05 20:20:31
810-3*216=162
810-3*216=162
kikipet
2016-03-05 20:20:31
162
162
PurplePancakes
2016-03-05 20:20:31
810-3*6*6*6
810-3*6*6*6
blizzard10
2016-03-05 20:20:31
162
162
blue8931
2016-03-05 20:20:31
$810 - 648 = \boxed{162}$
$810 - 648 = \boxed{162}$
algebra_star1234
2016-03-05 20:20:31
162
162
nukelauncher
2016-03-05 20:20:31
810-648=162!
810-648=162!
jfurf
2016-03-05 20:20:31
$648$ ways so $810-648=\boxed{162}$
$648$ ways so $810-648=\boxed{162}$
Aragorn66
2016-03-05 20:20:31
162
162
lleB_ocaT
2016-03-05 20:20:31
162
162
walnutwaldo20
2016-03-05 20:20:31
810-648= 162
810-648= 162
lleB_ocaT
2016-03-05 20:20:31
162
162
copeland
2016-03-05 20:20:33
Multiplying this out, we have 3*3!*3!*3! = 648. So our answer is $810-648=\boxed{162}.$
Multiplying this out, we have 3*3!*3!*3! = 648. So our answer is $810-648=\boxed{162}.$
copeland
2016-03-05 20:20:54
Okey-dokey!
Okey-dokey!
copeland
2016-03-05 20:20:56
What now?
What now?
idomath12345
2016-03-05 20:21:13
9.
9.
mathguy5041
2016-03-05 20:21:13
Onto 9?
Onto 9?
hliu70
2016-03-05 20:21:13
#9!
#9!
Locust
2016-03-05 20:21:13
Problem 1+8
Problem 1+8
SuperMaltese
2016-03-05 20:21:13
number 8!!
number 8!!
brian22
2016-03-05 20:21:13
I think 9 comes after 8
I think 9 comes after 8
copeland
2016-03-05 20:21:15
9. Triangle $ABC$ has $AB=40,$ $AC=31,$ and $\sin A=\dfrac15.$ This triangle is inscribed in rectangle $AQRS$ with $B$ on $\overline {QR}$ and $C$ on $\overline{RS}.$ Find the maximum possible area of $AQRS.$
9. Triangle $ABC$ has $AB=40,$ $AC=31,$ and $\sin A=\dfrac15.$ This triangle is inscribed in rectangle $AQRS$ with $B$ on $\overline {QR}$ and $C$ on $\overline{RS}.$ Find the maximum possible area of $AQRS.$
copeland
2016-03-05 20:21:16
So what's the first step in a geometry problem?
So what's the first step in a geometry problem?
Peggy
2016-03-05 20:21:36
diagram
diagram
SimonSun
2016-03-05 20:21:36
diagram
diagram
MikoTennisPro
2016-03-05 20:21:36
diagram
diagram
jrl_ct
2016-03-05 20:21:36
Diagram
Diagram
nukelauncher
2016-03-05 20:21:36
diagram!!!
diagram!!!
algebra_star1234
2016-03-05 20:21:36
DIAGRAM
DIAGRAM
blizzard10
2016-03-05 20:21:36
Diagram, of course!
Diagram, of course!
blue8931
2016-03-05 20:21:36
draw a picture
draw a picture
tennis1729
2016-03-05 20:21:36
picture
picture
ompatel99
2016-03-05 20:21:36
A diagram
A diagram
hamup1
2016-03-05 20:21:36
diagram
diagram
Geode359
2016-03-05 20:21:36
draw a diagram
draw a diagram
andsun19
2016-03-05 20:21:36
Draw a diagram.
Draw a diagram.
copeland
2016-03-05 20:21:39
In most geometry problems, the first step is to draw a nice, accurate diagram.
In most geometry problems, the first step is to draw a nice, accurate diagram.
copeland
2016-03-05 20:21:41
Here is a nice diagram:
Here is a nice diagram:
copeland
2016-03-05 20:21:42
copeland
2016-03-05 20:21:45
. . . and here's a nice accurate diagram:
. . . and here's a nice accurate diagram:
copeland
2016-03-05 20:21:46
copeland
2016-03-05 20:21:53
Unfortunately, this triangle is obtuse. It's impossible for $A$ and $B$ to lie on the same side of line $SR$.
Unfortunately, this triangle is obtuse. It's impossible for $A$ and $B$ to lie on the same side of line $SR$.
copeland
2016-03-05 20:21:59
Hopefully, if you did draw an accurate diagram, it didn't throw you off your game too much.
Hopefully, if you did draw an accurate diagram, it didn't throw you off your game too much.
blue8931
2016-03-05 20:22:04
ugh MAA...
ugh MAA...
andsun19
2016-03-05 20:22:10
So this problem is defective?
So this problem is defective?
copeland
2016-03-05 20:22:14
Incidentally, the problem is perfectly legit and I believe it is what the AMC intended:
Incidentally, the problem is perfectly legit and I believe it is what the AMC intended:
copeland
2016-03-05 20:22:18
9.b Triangle $ABC$ has $AB=40,$ $AC=31,$ and $\sin A=\dfrac15.$ Draw rectangle $AQRS$ with $B$ on line $QR$ and $C$ on line $RS.$ Find the maximum possible area of $AQRS.$
9.b Triangle $ABC$ has $AB=40,$ $AC=31,$ and $\sin A=\dfrac15.$ Draw rectangle $AQRS$ with $B$ on line $QR$ and $C$ on line $RS.$ Find the maximum possible area of $AQRS.$
copeland
2016-03-05 20:22:29
Let's solve this problem instead.
Let's solve this problem instead.
copeland
2016-03-05 20:22:36
What should we focus on in the diagram?
What should we focus on in the diagram?
brian6liu
2016-03-05 20:23:18
finding AQ*AS
finding AQ*AS
nosaj
2016-03-05 20:23:18
lengths AQ, and AS, since their product gives the area
lengths AQ, and AS, since their product gives the area
fishy15
2016-03-05 20:23:18
side lengtha
side lengtha
copeland
2016-03-05 20:23:20
That's a noble goal. What do we want to use for that?
That's a noble goal. What do we want to use for that?
andsun19
2016-03-05 20:23:55
trig
trig
brian6liu
2016-03-05 20:23:55
trig
trig
nosaj
2016-03-05 20:23:55
trig!
trig!
blue8931
2016-03-05 20:23:55
trig
trig
PurplePancakes
2016-03-05 20:23:55
trig bash
trig bash
copeland
2016-03-05 20:23:56
Trig needs angles, yo.
Trig needs angles, yo.
copeland
2016-03-05 20:24:04
What are the good angles?
What are the good angles?
hliu70
2016-03-05 20:24:51
sin A = 1/5 ?
sin A = 1/5 ?
ChickenOnRage
2016-03-05 20:24:51
A
A
hamup1
2016-03-05 20:24:51
$\angle{SAC},\angle{BAQ}$
$\angle{SAC},\angle{BAQ}$
brian6liu
2016-03-05 20:24:51
<SAC and <BAQ
<SAC and <BAQ
PurplePancakes
2016-03-05 20:24:51
<CAB is basically given
<CAB is basically given
mathguy5041
2016-03-05 20:24:51
Angle BAC=arcsin (0.2)
Angle BAC=arcsin (0.2)
ninjataco
2016-03-05 20:24:51
the ones adjacent to angle A?
the ones adjacent to angle A?
nosaj
2016-03-05 20:24:51
BAQ, CAS
BAQ, CAS
copeland
2016-03-05 20:24:54
Let's think about the angles at $A$ since those determine the rest of the diagram. I've marked $\angle SAC$ and $\angle QAB$, as well as $\angle BAC$:
Let's think about the angles at $A$ since those determine the rest of the diagram. I've marked $\angle SAC$ and $\angle QAB$, as well as $\angle BAC$:
copeland
2016-03-05 20:24:56
copeland
2016-03-05 20:24:58
We know that $\sin z=\dfrac15$. We also know that $x+y+z=90^\circ.$
We know that $\sin z=\dfrac15$. We also know that $x+y+z=90^\circ.$
copeland
2016-03-05 20:25:17
To find the area we need the side length. What is $AS$ in terms of these angles?
To find the area we need the side length. What is $AS$ in terms of these angles?
Locust
2016-03-05 20:26:07
31 * cos x
31 * cos x
snowumbrella
2016-03-05 20:26:07
31cosx
31cosx
algebra_star1234
2016-03-05 20:26:07
AC cos x
AC cos x
SuperMaltese
2016-03-05 20:26:07
AS = ACcosx ?
AS = ACcosx ?
brian6liu
2016-03-05 20:26:07
31cosx
31cosx
memc38123
2016-03-05 20:26:07
cosx*AC
cosx*AC
TheRealDeal
2016-03-05 20:26:07
cos(x)*31
cos(x)*31
fz2012
2016-03-05 20:26:07
31cosx
31cosx
qwerty733
2016-03-05 20:26:07
31cosx
31cosx
copeland
2016-03-05 20:26:09
$AS=AC\cos x=31\cos x$
$AS=AC\cos x=31\cos x$
copeland
2016-03-05 20:26:10
What is $AQ?$
What is $AQ?$
snowumbrella
2016-03-05 20:26:52
40cosy
40cosy
qwerty733
2016-03-05 20:26:52
40cosy
40cosy
PurplePancakes
2016-03-05 20:26:52
40cos(y)
40cos(y)
sxu
2016-03-05 20:26:52
40cosy
40cosy
fz2012
2016-03-05 20:26:52
40cosy
40cosy
algebra_star1234
2016-03-05 20:26:52
40cosy
40cosy
andsun19
2016-03-05 20:26:52
40*cosy?
40*cosy?
copeland
2016-03-05 20:26:57
$AQ=AB\cos y=40\cos y$.
$AQ=AB\cos y=40\cos y$.
copeland
2016-03-05 20:27:00
And we want the area which is
\[\text{Area}=AS\cdot AQ=(31\cos x)(40\cos y)=1240\cos x\cos y.\]
And we want the area which is
\[\text{Area}=AS\cdot AQ=(31\cos x)(40\cos y)=1240\cos x\cos y.\]
copeland
2016-03-05 20:27:06
That product is scary. There's no obvious way to minimize it right now. Can we write it any differently?
That product is scary. There's no obvious way to minimize it right now. Can we write it any differently?
Ericaops
2016-03-05 20:27:46
product to sum
product to sum
andsun19
2016-03-05 20:27:46
product of cosines
product of cosines
jfurf
2016-03-05 20:27:46
Yes! Use product to sum formula!
Yes! Use product to sum formula!
memc38123
2016-03-05 20:27:46
product to sum identity
product to sum identity
mathguy5041
2016-03-05 20:27:46
Sum to product formulas!!!!
Sum to product formulas!!!!
brian6liu
2016-03-05 20:27:46
product to sum
product to sum
copeland
2016-03-05 20:27:49
We have the product-to-sum formula. What is $\cos x\cos y$ equal to?
We have the product-to-sum formula. What is $\cos x\cos y$ equal to?
PurplePancakes
2016-03-05 20:28:11
1/2(something I don't memorize because it's easy to derive)
1/2(something I don't memorize because it's easy to derive)
copeland
2016-03-05 20:28:14
agreed.
agreed.
andsun19
2016-03-05 20:29:13
use cosine sums to derive
use cosine sums to derive
weilunsun28
2016-03-05 20:29:13
lets derive it!
lets derive it!
copeland
2016-03-05 20:29:15
That's better done alone. In a quiet, dark room. Thanks, though.
That's better done alone. In a quiet, dark room. Thanks, though.
qwerty733
2016-03-05 20:29:28
1/2 (cos(x+y)+cos(x-y))
1/2 (cos(x+y)+cos(x-y))
brian6liu
2016-03-05 20:29:28
(cos(x+y)+cos(x-y))/2
(cos(x+y)+cos(x-y))/2
nosaj
2016-03-05 20:29:28
$\frac{1}{2}\left[\cos(x+y) + \cos(x-y)\right]$
$\frac{1}{2}\left[\cos(x+y) + \cos(x-y)\right]$
Ericaops
2016-03-05 20:29:28
1/2(cos(x+y)+cos(x-y))
1/2(cos(x+y)+cos(x-y))
algebra_star1234
2016-03-05 20:29:28
1/2 (cos(x+y) + cos(x-y))
1/2 (cos(x+y) + cos(x-y))
copeland
2016-03-05 20:29:31
\[\cos x\cos y=\frac12(\cos(x+y)+\cos(x-y)).\]
\[\cos x\cos y=\frac12(\cos(x+y)+\cos(x-y)).\]
copeland
2016-03-05 20:29:33
In our case, \[\text{Area}=620(\cos (x+y)+\cos(x-y)).\]
In our case, \[\text{Area}=620(\cos (x+y)+\cos(x-y)).\]
copeland
2016-03-05 20:29:34
Do we know either of those?
Do we know either of those?
PurplePancakes
2016-03-05 20:30:08
x+y=90-z
x+y=90-z
memc38123
2016-03-05 20:30:08
x+Y = 90-Z
x+Y = 90-Z
qwerty733
2016-03-05 20:30:08
yeah cos(x+y)
yeah cos(x+y)
fishy15
2016-03-05 20:30:08
x+y=90-z
x+y=90-z
brian6liu
2016-03-05 20:30:17
cos(x+y)=sin(A)=1/5
cos(x+y)=sin(A)=1/5
fz2012
2016-03-05 20:30:17
cos (x+y)=1/5
cos (x+y)=1/5
nukelauncher
2016-03-05 20:30:17
yes! cos(x+y) = sin z = 1/5!
yes! cos(x+y) = sin z = 1/5!
bestwillcui1
2016-03-05 20:30:17
cos(x+y) = sin(z) = 1/5
cos(x+y) = sin(z) = 1/5
PurplePancakes
2016-03-05 20:30:17
cos(x+y)=cos(90-z)=sin(z)
cos(x+y)=cos(90-z)=sin(z)
copeland
2016-03-05 20:30:20
We know $\cos(x+y)=\cos(90^\circ-z)=\sin(z)=\dfrac15,$ so\[\text{Area}=620\left(\frac15+\cos(x-y)\right).\]
We know $\cos(x+y)=\cos(90^\circ-z)=\sin(z)=\dfrac15,$ so\[\text{Area}=620\left(\frac15+\cos(x-y)\right).\]
copeland
2016-03-05 20:30:21
And how can we control $\cos(x-y)?$
And how can we control $\cos(x-y)?$
fz2012
2016-03-05 20:31:31
Make x and y equal to maximize
Make x and y equal to maximize
brian6liu
2016-03-05 20:31:31
it's maximized when x=y and cos(x-y)=cos(0)=1
it's maximized when x=y and cos(x-y)=cos(0)=1
PurplePancakes
2016-03-05 20:31:31
maximized when x=y
maximized when x=y
algebra_star1234
2016-03-05 20:31:31
we can maximize by letting x=y
we can maximize by letting x=y
sxu
2016-03-05 20:31:31
make it 1
make it 1
bestwillcui1
2016-03-05 20:31:31
x=y maximizes it
x=y maximizes it
andsun19
2016-03-05 20:31:31
well, max = 1
well, max = 1
nosaj
2016-03-05 20:31:31
it's maximized at cos(0) = 1
it's maximized at cos(0) = 1
snowumbrella
2016-03-05 20:31:31
max possible is 1?
max possible is 1?
qwerty733
2016-03-05 20:31:31
Well is x=y then its 1
Well is x=y then its 1
brian22
2016-03-05 20:31:31
Well it can be up to $1$
Well it can be up to $1$
copeland
2016-03-05 20:31:34
If we make $x=y$ then this is equal to 1. That's definitely the maximum value. What is the final answer?
If we make $x=y$ then this is equal to 1. That's definitely the maximum value. What is the final answer?
brian6liu
2016-03-05 20:32:23
744
744
andsun19
2016-03-05 20:32:23
744
744
mssmath
2016-03-05 20:32:23
744
744
nosaj
2016-03-05 20:32:23
744
744
blue8931
2016-03-05 20:32:23
744
744
blizzard10
2016-03-05 20:32:23
744
744
Axolotl
2016-03-05 20:32:23
744
744
MikoTennisPro
2016-03-05 20:32:23
744
744
copeland
2016-03-05 20:32:26
When $x=y,$\[\text{Area}=620\left(\frac15+1\right)=\frac{620\cdot6}5=\frac{3720}5=\boxed{744}.\]
When $x=y,$\[\text{Area}=620\left(\frac15+1\right)=\frac{620\cdot6}5=\frac{3720}5=\boxed{744}.\]
LighteningAB
2016-03-05 20:34:19
NUMBER TENNNN
NUMBER TENNNN
ThorJames
2016-03-05 20:34:19
10.
10.
mathguy5041
2016-03-05 20:34:19
10
10
blizzard10
2016-03-05 20:34:19
Problem 10!
Problem 10!
memc38123
2016-03-05 20:34:22
onward
onward
copeland
2016-03-05 20:34:28
10. A strictly increasing sequence of positive integers $a_1, a_2, a_3, \ldots$ has the property that for every positive integer $k$, the subsequence $a_{2k-1}, a_{2k}, a_{2k+1}$ is geometric and the subsequence $a_{2k}, a_{2k+1},a_{2k+2}$ is arithmetic. Suppose that $a_{13} = 2016$. Find $a_1$.
10. A strictly increasing sequence of positive integers $a_1, a_2, a_3, \ldots$ has the property that for every positive integer $k$, the subsequence $a_{2k-1}, a_{2k}, a_{2k+1}$ is geometric and the subsequence $a_{2k}, a_{2k+1},a_{2k+2}$ is arithmetic. Suppose that $a_{13} = 2016$. Find $a_1$.
copeland
2016-03-05 20:34:38
Hey, I've seen this problem before...
Hey, I've seen this problem before...
copeland
2016-03-05 20:34:40
2004II/9: A sequence of positive integers with $a_1=1$ and $a_9+a_{10}=646$ is formed so that the first three terms are in geometric progression, the second, third, and fourth terms are in arithmetic progression, and, in general, for all $n\ge1$, the terms $a_{2n-1}$, $a_{2n}$, $a_{2n+1}$ are in geometric progression, and the terms $a_{2n}$, $a_{2n+1}$, and $a_{2n+2}$ are in arithmetic progression. Let $a_n$ be the greatest term in this sequence that is less than 1000. Find $n+a_n$.
2004II/9: A sequence of positive integers with $a_1=1$ and $a_9+a_{10}=646$ is formed so that the first three terms are in geometric progression, the second, third, and fourth terms are in arithmetic progression, and, in general, for all $n\ge1$, the terms $a_{2n-1}$, $a_{2n}$, $a_{2n+1}$ are in geometric progression, and the terms $a_{2n}$, $a_{2n+1}$, and $a_{2n+2}$ are in arithmetic progression. Let $a_n$ be the greatest term in this sequence that is less than 1000. Find $n+a_n$.
copeland
2016-03-05 20:34:47
Well, we have information about the end, so let's start there. What type of sequence is $a_{11}, a_{12}, a_{13}$?
Well, we have information about the end, so let's start there. What type of sequence is $a_{11}, a_{12}, a_{13}$?
ryanyz10
2016-03-05 20:35:12
geo
geo
kikipet
2016-03-05 20:35:12
geometric
geometric
blue8931
2016-03-05 20:35:12
geometric
geometric
MikoTennisPro
2016-03-05 20:35:12
geometric
geometric
brian6liu
2016-03-05 20:35:12
geometric
geometric
atmchallenge
2016-03-05 20:35:12
Geometric
Geometric
Axolotl
2016-03-05 20:35:12
geometric
geometric
PurplePancakes
2016-03-05 20:35:12
geometric
geometric
copeland
2016-03-05 20:35:14
That's a geometric sequence, so let's write $a_{12} = 2016r$ and $a_{11} = 2016 r^2$ for some $r = \dfrac mn$.
That's a geometric sequence, so let's write $a_{12} = 2016r$ and $a_{11} = 2016 r^2$ for some $r = \dfrac mn$.
copeland
2016-03-05 20:35:19
Is there anything interesting we can say about $m$ or $n$?
Is there anything interesting we can say about $m$ or $n$?
copeland
2016-03-05 20:35:24
(Assume that $m$ and $n$ are relatively prime since this is likely to be about divisibility.)
(Assume that $m$ and $n$ are relatively prime since this is likely to be about divisibility.)
mewtwomew
2016-03-05 20:36:04
n^2 is a factor of 2016
n^2 is a factor of 2016
ryanyz10
2016-03-05 20:36:04
n is a factor of 2016 since all numbers are ints
n is a factor of 2016 since all numbers are ints
Ericaops
2016-03-05 20:36:04
n^2 is a factor of 2016
n^2 is a factor of 2016
qwerty733
2016-03-05 20:36:04
n^2 is a factor of 2016
n^2 is a factor of 2016
brian6liu
2016-03-05 20:36:04
n^2 divides 2016
n^2 divides 2016
copeland
2016-03-05 20:36:09
Since $a_{11}$ is an integer, $n^2$ divides 2016. What is the prime factorization of 2016?
Since $a_{11}$ is an integer, $n^2$ divides 2016. What is the prime factorization of 2016?
atmchallenge
2016-03-05 20:37:06
$2016=2^5\cdot 3^2 \cdot 7$
$2016=2^5\cdot 3^2 \cdot 7$
nukelauncher
2016-03-05 20:37:06
2^5*3^2*7
2^5*3^2*7
qwerty733
2016-03-05 20:37:06
2^5 * 3^2 *7
2^5 * 3^2 *7
andsun19
2016-03-05 20:37:06
2^5*3^2*7
2^5*3^2*7
brian6liu
2016-03-05 20:37:06
2^5*3^2*7
2^5*3^2*7
sxu
2016-03-05 20:37:06
2^5 3^2 7 or something
2^5 3^2 7 or something
Locust
2016-03-05 20:37:06
AlphaPi17
2016-03-05 20:37:06
2^5 * 3^2 * 7
2^5 * 3^2 * 7
fishy15
2016-03-05 20:37:06
2^5*3^2*7
2^5*3^2*7
copeland
2016-03-05 20:37:08
We have $2016 = 2^5 \cdot 3^2 \cdot 7$. So $n$ must be a divisor of $2^2 \cdot 3 =12$.
We have $2016 = 2^5 \cdot 3^2 \cdot 7$. So $n$ must be a divisor of $2^2 \cdot 3 =12$.
copeland
2016-03-05 20:37:18
And what else do we know about $m$ and $n$?
And what else do we know about $m$ and $n$?
ryanyz10
2016-03-05 20:37:38
m < n
m < n
ChickenOnRage
2016-03-05 20:37:38
m/n < 1
m/n < 1
Locust
2016-03-05 20:37:38
m<n
m<n
nosaj
2016-03-05 20:37:38
m<n
m<n
qwerty733
2016-03-05 20:37:38
m<n
m<n
copeland
2016-03-05 20:38:08
We need the sequence to decrease from right to left, so $r = \frac m{12}$ where $m$ is one of 1,2,3,...,11.
We need the sequence to decrease from right to left, so $r = \frac m{12}$ where $m$ is one of 1,2,3,...,11.
copeland
2016-03-05 20:38:11
Let's try our next three-term sequence: $a_{10}, a_{11}, a_{12}$ is arithmetic. What equation does that give us?
Let's try our next three-term sequence: $a_{10}, a_{11}, a_{12}$ is arithmetic. What equation does that give us?
blue8931
2016-03-05 20:38:36
how do we know that n=12 for sure
how do we know that n=12 for sure
copeland
2016-03-05 20:38:45
Sorry, we don't know $n=12$.
Sorry, we don't know $n=12$.
copeland
2016-03-05 20:38:48
We know $n$ divides 12.
We know $n$ divides 12.
copeland
2016-03-05 20:39:09
However, we know that the FRACTION $\frac mn$ is equal to something over 12.
However, we know that the FRACTION $\frac mn$ is equal to something over 12.
copeland
2016-03-05 20:39:41
So if $m=2$ and $n=3$, we still have $\dfrac mn=\dfrac8{12}.$
So if $m=2$ and $n=3$, we still have $\dfrac mn=\dfrac8{12}.$
copeland
2016-03-05 20:40:00
What equation do we get for $a_{10}$?
What equation do we get for $a_{10}$?
copeland
2016-03-05 20:41:01
We have $a_{12} = 2016r$ and $a_{11} = 2016 r^2.$
We have $a_{12} = 2016r$ and $a_{11} = 2016 r^2.$
qwerty733
2016-03-05 20:42:06
a_10=2(2016(m/12)^2-2016(m/12)
a_10=2(2016(m/12)^2-2016(m/12)
atmchallenge
2016-03-05 20:42:06
$a_{10}=4032r^2-2016r$.
$a_{10}=4032r^2-2016r$.
PurplePancakes
2016-03-05 20:42:06
2016r-2(2016r-2016r^2)=2*2016r^2-2016r
2016r-2(2016r-2016r^2)=2*2016r^2-2016r
calculus_riju
2016-03-05 20:42:06
then $a_{10}= 4032r^2 - 2016r
then $a_{10}= 4032r^2 - 2016r
ninjataco
2016-03-05 20:42:06
2*2016r^2 - 2016r
2*2016r^2 - 2016r
walnutwaldo20
2016-03-05 20:42:06
2016r(2r-1)
2016r(2r-1)
nukelauncher
2016-03-05 20:42:06
a(10)=4032r^2-2016r
a(10)=4032r^2-2016r
copeland
2016-03-05 20:42:10
We get \[a_{10} = 2a_{11} - a_{12} = 2 \cdot 2016 r^2 - 2016r = 2016r(2r - 1).\]
We get \[a_{10} = 2a_{11} - a_{12} = 2 \cdot 2016 r^2 - 2016r = 2016r(2r - 1).\]
copeland
2016-03-05 20:42:13
What does that tell us about $r$?
What does that tell us about $r$?
brian6liu
2016-03-05 20:42:46
r>1/2
r>1/2
mssmath
2016-03-05 20:42:46
r>1/2
r>1/2
hamup1
2016-03-05 20:42:46
greater than 1/2
greater than 1/2
calculus_riju
2016-03-05 20:42:46
r>.5
r>.5
sophiazhi
2016-03-05 20:42:46
>1/2
>1/2
copeland
2016-03-05 20:42:51
Since $a_{10}$ is a positive integer, this tells us that $r > \dfrac 12$. So $r = \dfrac m{12}$ where $m$ is one of 7,8,9,10,11.
Since $a_{10}$ is a positive integer, this tells us that $r > \dfrac 12$. So $r = \dfrac m{12}$ where $m$ is one of 7,8,9,10,11.
copeland
2016-03-05 20:42:55
Next up, $a_9, a_{10}, a_{11}$ is geometric. What's its common ratio (again going downward, so the ratio of $a_{10}$ to $a_{11}$)?
Next up, $a_9, a_{10}, a_{11}$ is geometric. What's its common ratio (again going downward, so the ratio of $a_{10}$ to $a_{11}$)?
atmchallenge
2016-03-05 20:44:32
$\frac{2r-1}{r}$.
$\frac{2r-1}{r}$.
sxu
2016-03-05 20:44:32
(2r-1)/r
(2r-1)/r
xwang1
2016-03-05 20:44:32
$\frac{2r-1}{r}$
$\frac{2r-1}{r}$
calculus_riju
2016-03-05 20:44:32
$\frac{2r-1}{r}$
$\frac{2r-1}{r}$
blizzard10
2016-03-05 20:44:32
$\dfrac{2r-1}{r}$
$\dfrac{2r-1}{r}$
nosaj
2016-03-05 20:44:34
(2r-1)/r
(2r-1)/r
copeland
2016-03-05 20:44:36
Its common ratio is \[\frac{a_{10}}{a_{11}} = \frac{2016r(2r-1)}{2016r^2} = \frac{2r-1}{r}.\]
Its common ratio is \[\frac{a_{10}}{a_{11}} = \frac{2016r(2r-1)}{2016r^2} = \frac{2r-1}{r}.\]
copeland
2016-03-05 20:44:38
In particular, $a_9 = 2016(2r - 1)^2$.
In particular, $a_9 = 2016(2r - 1)^2$.
copeland
2016-03-05 20:44:44
Then $a_8, a_9, a_{10}$ is arithmetic. So we get
\begin{align*}
a_8 &= 2a_{9} - a_{10} \\
&= 2\cdot 2016 (2r-1)^2 - 2016 r(2r - 1) \\
&= 2016\cdot(2r-1) \cdot(4r - 2 - r) \\
&= 2016 \cdot (2r - 1) \cdot (3r - 2).
\end{align*}
Then $a_8, a_9, a_{10}$ is arithmetic. So we get
\begin{align*}
a_8 &= 2a_{9} - a_{10} \\
&= 2\cdot 2016 (2r-1)^2 - 2016 r(2r - 1) \\
&= 2016\cdot(2r-1) \cdot(4r - 2 - r) \\
&= 2016 \cdot (2r - 1) \cdot (3r - 2).
\end{align*}
copeland
2016-03-05 20:44:50
Does that tell us anything interesting?
Does that tell us anything interesting?
brian6liu
2016-03-05 20:45:18
r>2/3
r>2/3
nosaj
2016-03-05 20:45:18
r>2/3
r>2/3
mssmath
2016-03-05 20:45:18
r>2/3
r>2/3
copeland
2016-03-05 20:45:22
Again, $a_8$ is a positive integer, so $r > \dfrac 23$.
Again, $a_8$ is a positive integer, so $r > \dfrac 23$.
copeland
2016-03-05 20:45:23
Now we have $m$ is one of 9, 10, 11.
Now we have $m$ is one of 9, 10, 11.
deltaepsilon6
2016-03-05 20:45:33
pattern!
pattern!
ryanyz10
2016-03-05 20:45:33
looks like a pattern kindof
looks like a pattern kindof
SuperMaltese
2016-03-05 20:45:33
looks like a pattern
looks like a pattern
deltaepsilon6
2016-03-05 20:45:33
pattern recognition!
pattern recognition!
copeland
2016-03-05 20:45:39
It does look kind of like a pattern.
It does look kind of like a pattern.
copeland
2016-03-05 20:45:43
Lt's try one mor.
Lt's try one mor.
copeland
2016-03-05 20:45:45
What is the common ratio of $a_7, a_8, a_9$? (Again, going downward.)
What is the common ratio of $a_7, a_8, a_9$? (Again, going downward.)
blizzard10
2016-03-05 20:47:24
$\dfrac{3r-2}{2r-1}$
$\dfrac{3r-2}{2r-1}$
SuperMaltese
2016-03-05 20:47:24
(3r-2)/(2r-1)
(3r-2)/(2r-1)
atmchallenge
2016-03-05 20:47:24
$\frac{3r-2}{2r-1}$.
$\frac{3r-2}{2r-1}$.
memc38123
2016-03-05 20:47:24
(3r-2)/(2r-1)
(3r-2)/(2r-1)
nosaj
2016-03-05 20:47:24
(3r-2)/(2r-1)
(3r-2)/(2r-1)
copeland
2016-03-05 20:47:26
Its common ratio is \[ \frac{a_8}{a_9} = \frac{3r - 2}{2r - 1}. \]
Its common ratio is \[ \frac{a_8}{a_9} = \frac{3r - 2}{2r - 1}. \]
copeland
2016-03-05 20:47:28
So $a_7 = 2016 (3r - 2)^2$.
So $a_7 = 2016 (3r - 2)^2$.
copeland
2016-03-05 20:47:35
Anyone starting to notice a pattern?
Anyone starting to notice a pattern?
blue8931
2016-03-05 20:48:09
yes
yes
nukelauncher
2016-03-05 20:48:09
YES!
YES!
jfurf
2016-03-05 20:48:09
ME!
ME!
fishy15
2016-03-05 20:48:09
me!
me!
Prof.iHen
2016-03-05 20:48:09
ja
ja
sxu
2016-03-05 20:48:09
sort of
sort of
walnutwaldo20
2016-03-05 20:48:09
Me!
Me!
Aragorn66
2016-03-05 20:48:09
yup
yup
copeland
2016-03-05 20:48:17
Let's compute a few more terms and see if we notice anything.
Let's compute a few more terms and see if we notice anything.
copeland
2016-03-05 20:48:21
\begin{align*}
a_6 &= 2a_7 - a_8 \\
&= 2\cdot 2016 \cdot (3r - 2)^2 - 2016 \cdot (2r - 1) \cdot (3r - 2) \\
& = 2016(3r - 2)(6r - 4 - 2r + 1) \\
&= 2016(3r - 2)(4r - 3)
\end{align*}
\begin{align*}
a_6 &= 2a_7 - a_8 \\
&= 2\cdot 2016 \cdot (3r - 2)^2 - 2016 \cdot (2r - 1) \cdot (3r - 2) \\
& = 2016(3r - 2)(6r - 4 - 2r + 1) \\
&= 2016(3r - 2)(4r - 3)
\end{align*}
copeland
2016-03-05 20:48:27
The common ratio of $a_5, a_6, a_7$ is \[ \frac{a_6}{a_7} = \frac{4r -3}{3r - 2}.\]
The common ratio of $a_5, a_6, a_7$ is \[ \frac{a_6}{a_7} = \frac{4r -3}{3r - 2}.\]
copeland
2016-03-05 20:48:27
So $a_5 = 2016(4r - 3)^2$.
So $a_5 = 2016(4r - 3)^2$.
copeland
2016-03-05 20:48:31
What will $a_1$ be?
What will $a_1$ be?
deltaepsilon6
2016-03-05 20:49:07
a1= 2016(6r-5)^2
a1= 2016(6r-5)^2
deltaepsilon6
2016-03-05 20:49:07
a1=2016(6r-5)^2
a1=2016(6r-5)^2
nosaj
2016-03-05 20:49:07
2016(6r-5)^2
2016(6r-5)^2
azmath333
2016-03-05 20:49:07
$2016(6r-5)^2$
$2016(6r-5)^2$
blue8931
2016-03-05 20:49:07
$2016(6r-5)^2$
$2016(6r-5)^2$
atmchallenge
2016-03-05 20:49:07
$a_1=2016(6r-5)^2
$a_1=2016(6r-5)^2
copeland
2016-03-05 20:49:09
We'll have $a_5 = 2016 (4r - 3)^2$, then $a_3 = 2016(5r - 4)^2$ and $a_1 = 2016(6r - 5)^2$.
We'll have $a_5 = 2016 (4r - 3)^2$, then $a_3 = 2016(5r - 4)^2$ and $a_1 = 2016(6r - 5)^2$.
copeland
2016-03-05 20:49:15
So what does this tell us about $r$?
So what does this tell us about $r$?
brian6liu
2016-03-05 20:49:53
r>5/6
r>5/6
algebra_star1234
2016-03-05 20:49:53
r=11/12
r=11/12
walnutwaldo20
2016-03-05 20:49:53
r > 5/6
r > 5/6
hamup1
2016-03-05 20:49:53
$r>\dfrac{5}{6}$
$r>\dfrac{5}{6}$
ryanyz10
2016-03-05 20:49:53
r > 5/6
r > 5/6
sxu
2016-03-05 20:49:53
its 11/12
its 11/12
andsun19
2016-03-05 20:49:53
11/12
11/12
PurplePancakes
2016-03-05 20:49:53
r>5/6
r>5/6
hamup1
2016-03-05 20:49:53
or $r=\dfrac{11}{12}$!!
or $r=\dfrac{11}{12}$!!
akaashp11
2016-03-05 20:49:53
11/12
11/12
Aragorn66
2016-03-05 20:49:53
r=11/12
r=11/12
copeland
2016-03-05 20:49:55
Since $a_1$ is a positive integer, we must have $r \ge \dfrac 56.$ That means $m$ can only be 11, and we have $r = \dfrac{11}{12}$.
Since $a_1$ is a positive integer, we must have $r \ge \dfrac 56.$ That means $m$ can only be 11, and we have $r = \dfrac{11}{12}$.
copeland
2016-03-05 20:49:57
So what is $a_1$?
So what is $a_1$?
SuperMaltese
2016-03-05 20:50:31
504
504
tennis1729
2016-03-05 20:50:31
504
504
kikipet
2016-03-05 20:50:31
504
504
illogical_21
2016-03-05 20:50:31
504
504
nukelauncher
2016-03-05 20:50:31
504!
504!
sxu
2016-03-05 20:50:31
$504$
$504$
bestwillcui1
2016-03-05 20:50:31
five hunnet 4
five hunnet 4
AlphaPi17
2016-03-05 20:50:31
504
504
brian6liu
2016-03-05 20:50:31
504
504
hamup1
2016-03-05 20:50:31
$\dfrac{2016}{4}=504$
$\dfrac{2016}{4}=504$
copeland
2016-03-05 20:50:33
We must have $a_1 = 2016 \cdot \dfrac{36}{144} = \boxed{504}$.
We must have $a_1 = 2016 \cdot \dfrac{36}{144} = \boxed{504}$.
copeland
2016-03-05 20:50:49
Incidentally, all the odd terms were squares of an arithmetic progression.
Incidentally, all the odd terms were squares of an arithmetic progression.
copeland
2016-03-05 20:51:05
The even terms were their geometric means (of course).
The even terms were their geometric means (of course).
ChickenOnRage
2016-03-05 20:51:16
was this the best solution?
was this the best solution?
copeland
2016-03-05 20:51:33
Interestingly, this was probably the most "guessable" problem on the test. . .
Interestingly, this was probably the most "guessable" problem on the test. . .
copeland
2016-03-05 20:52:20
Now Problem 11:
Now Problem 11:
copeland
2016-03-05 20:52:22
11. Let $P(x)$ be a nonzero polynomial such that $(x-1)P(x+1) = (x+2)P(x)$ for every real $x$, and $(P(2))^2 = P(3)$. Then $P(\frac 72) = \frac mn$, where $m$ and $n$ are relatively prime positive integers. Find $m + n$.
11. Let $P(x)$ be a nonzero polynomial such that $(x-1)P(x+1) = (x+2)P(x)$ for every real $x$, and $(P(2))^2 = P(3)$. Then $P(\frac 72) = \frac mn$, where $m$ and $n$ are relatively prime positive integers. Find $m + n$.
copeland
2016-03-05 20:52:36
What's a nice thing to do when you're given a functional equation?
What's a nice thing to do when you're given a functional equation?
themoocow
2016-03-05 20:52:52
plug values
plug values
sxu
2016-03-05 20:52:52
plug in values
plug in values
mewtwomew
2016-03-05 20:52:52
plug and chug
plug and chug
brian6liu
2016-03-05 20:52:52
plug in values
plug in values
blizzard10
2016-03-05 20:52:52
Plug in some numbers?
Plug in some numbers?
andsun19
2016-03-05 20:52:52
Plug in numbers that you have
Plug in numbers that you have
copeland
2016-03-05 20:52:58
It's always a good idea to plug in nice values for the free variable. What's something good to plug in here?
It's always a good idea to plug in nice values for the free variable. What's something good to plug in here?
atmchallenge
2016-03-05 20:53:12
$x=1$
$x=1$
AlphaPi17
2016-03-05 20:53:12
1
1
calculus_riju
2016-03-05 20:53:12
1
1
ChickenOnRage
2016-03-05 20:53:12
1
1
copeland
2016-03-05 20:53:15
Plugging in $x = 1$ gives us $0 = 3P(1)$. So?
Plugging in $x = 1$ gives us $0 = 3P(1)$. So?
atmchallenge
2016-03-05 20:53:32
$P(1)=0$.
$P(1)=0$.
themoocow
2016-03-05 20:53:32
1 is aroot of P
1 is aroot of P
brian6liu
2016-03-05 20:53:32
P(1)=0
P(1)=0
xwang1
2016-03-05 20:53:32
$P(1)=0$
$P(1)=0$
PurplePancakes
2016-03-05 20:53:32
1 is a root
1 is a root
Locust
2016-03-05 20:53:32
P(1)=0
P(1)=0
kikipet
2016-03-05 20:53:32
P(1) = 0
P(1) = 0
epgy8
2016-03-05 20:53:32
$P(1)=0$
$P(1)=0$
copeland
2016-03-05 20:53:35
So we conclude that 1 is a root of $P(x)$. Is there anything else we might try plugging in?
So we conclude that 1 is a root of $P(x)$. Is there anything else we might try plugging in?
PurplePancakes
2016-03-05 20:53:57
0
0
AlphaPi17
2016-03-05 20:53:57
0
0
epgy8
2016-03-05 20:53:57
0
0
hamup1
2016-03-05 20:53:57
or 0 while we're at it
or 0 while we're at it
Aragorn66
2016-03-05 20:53:57
0
0
copeland
2016-03-05 20:54:03
If we plug in $x = 0$, we get $-P(1) = 2P(0)$, or $0 = 2P(0)$. So 0 is also a root of $P(x)$.
If we plug in $x = 0$, we get $-P(1) = 2P(0)$, or $0 = 2P(0)$. So 0 is also a root of $P(x)$.
copeland
2016-03-05 20:54:04
Anything else we might want to plug in?
Anything else we might want to plug in?
Aragorn66
2016-03-05 20:54:26
-1
-1
Aragorn66
2016-03-05 20:54:26
-1
-1
hamup1
2016-03-05 20:54:26
-1...
-1...
illogical_21
2016-03-05 20:54:26
-1
-1
epgy8
2016-03-05 20:54:26
-1
-1
AlphaPi17
2016-03-05 20:54:26
-1
-1
copeland
2016-03-05 20:54:28
Now that we know 0 is a root, it makes sense to plug in $x = -1$. Then we get $-2P(0) = P(-1)$, or $0 = P(-1)$. So -1 is also a root of $P(x)$.
Now that we know 0 is a root, it makes sense to plug in $x = -1$. Then we get $-2P(0) = P(-1)$, or $0 = P(-1)$. So -1 is also a root of $P(x)$.
copeland
2016-03-05 20:54:49
Anything else?
Anything else?
calculus_riju
2016-03-05 20:55:38
-2
-2
Aragorn66
2016-03-05 20:55:38
-2
-2
algebra_star1234
2016-03-05 20:55:38
-2
-2
ninjataco
2016-03-05 20:55:38
-2
-2
PurplePancakes
2016-03-05 20:55:38
-2
-2
atmchallenge
2016-03-05 20:55:38
$x=-2$
$x=-2$
ChickenOnRage
2016-03-05 20:55:38
-2
-2
azmath333
2016-03-05 20:55:38
-2
-2
AlphaPi17
2016-03-05 20:55:38
-2
-2
algebra_star1234
2016-03-05 20:55:38
-2?
-2?
walnutwaldo20
2016-03-05 20:55:38
-2
-2
epgy8
2016-03-05 20:55:38
-2
-2
19bobhu
2016-03-05 20:55:38
-2?
-2?
copeland
2016-03-05 20:55:40
If we substitute $-2$ we get $-3P(-1)=0P(-2).$
If we substitute $-2$ we get $-3P(-1)=0P(-2).$
copeland
2016-03-05 20:55:44
What do we learn?
What do we learn?
walnutwaldo20
2016-03-05 20:56:13
NOTHING!!!!!!!!!!
NOTHING!!!!!!!!!!
brian6liu
2016-03-05 20:56:13
nothing
nothing
epiphany
2016-03-05 20:56:13
0=0
0=0
sxu
2016-03-05 20:56:13
NOTHING!
NOTHING!
hamup1
2016-03-05 20:56:13
nothing!
nothing!
bestwillcui1
2016-03-05 20:56:13
nothing
nothing
copeland
2016-03-05 20:56:16
This tells us 0=0. Reassuring, guys.
This tells us 0=0. Reassuring, guys.
copeland
2016-03-05 20:56:20
I don't see anything else that helps right away. But we do have $P(x) = (x-1)x(x+1)Q(x)$ for some polynomial $Q(x)$.
I don't see anything else that helps right away. But we do have $P(x) = (x-1)x(x+1)Q(x)$ for some polynomial $Q(x)$.
copeland
2016-03-05 20:56:27
What would be really nice?
What would be really nice?
xwang1
2016-03-05 20:56:58
P is a cubic
P is a cubic
blue8931
2016-03-05 20:56:58
P(x) is cubic
P(x) is cubic
xwang1
2016-03-05 20:56:58
If $Q(x)$ was a constant.
If $Q(x)$ was a constant.
hamup1
2016-03-05 20:56:58
if $Q(x)$ was constant ;)
if $Q(x)$ was constant ;)
mewtwomew
2016-03-05 20:56:58
if it was a constant
if it was a constant
blue8931
2016-03-05 20:56:58
Q(x) is constant
Q(x) is constant
atmchallenge
2016-03-05 20:56:58
If $P(x)$ was cubic, so $Q(x)$ was a constant!
If $P(x)$ was cubic, so $Q(x)$ was a constant!
bestwillcui1
2016-03-05 20:56:58
if Q(x) is constant!!
if Q(x) is constant!!
deltaepsilon6
2016-03-05 20:56:58
if Q(x)=k
if Q(x)=k
copeland
2016-03-05 20:56:59
Is it? That'd be cool. . .
Is it? That'd be cool. . .
copeland
2016-03-05 20:57:03
Since we're struggling to find more roots, it would be really nice if $Q(x)$ was just a constant. How can we figure out the degree of $P(x)$?
Since we're struggling to find more roots, it would be really nice if $Q(x)$ was just a constant. How can we figure out the degree of $P(x)$?
ninjataco
2016-03-05 20:57:42
finite differences
finite differences
xwang1
2016-03-05 20:57:42
Finite differences!!!
Finite differences!!!
nukelauncher
2016-03-05 20:57:42
use differences
use differences
SimonSun
2016-03-05 20:57:42
we have delta
we have delta
andsun19
2016-03-05 20:57:42
differences... then differences of differences... etc etc until we get constants
differences... then differences of differences... etc etc until we get constants
deltaepsilon6
2016-03-05 20:57:42
list values of P(x), and find the difference of each term
list values of P(x), and find the difference of each term
SimonSun
2016-03-05 20:57:42
DELTA ALL THE WAY
DELTA ALL THE WAY
copeland
2016-03-05 20:57:44
The given equation rearranges to look a lot like finite differences: \[x\left(P(x+1) - P(x) \right) = P(x+1) + 2P(x).\]
The given equation rearranges to look a lot like finite differences: \[x\left(P(x+1) - P(x) \right) = P(x+1) + 2P(x).\]
copeland
2016-03-05 20:57:52
What is interesting there?
What is interesting there?
copeland
2016-03-05 20:58:28
Hmm. . .
Hmm. . .
copeland
2016-03-05 20:58:41
The left side has the first finite difference.
The left side has the first finite difference.
copeland
2016-03-05 20:59:00
(The right doesn't.)
(The right doesn't.)
copeland
2016-03-05 20:59:14
What exactly is going to happen when we take the first finite difference?
What exactly is going to happen when we take the first finite difference?
copeland
2016-03-05 21:00:02
Say, for example, that $P(x)=5x^{13}+\cdots.$ Then what does $x(P(x+1)-P(x))$ look like?
Say, for example, that $P(x)=5x^{13}+\cdots.$ Then what does $x(P(x+1)-P(x))$ look like?
brian6liu
2016-03-05 21:01:08
65x^13+...
65x^13+...
ninjataco
2016-03-05 21:01:08
65x^13 + ...
65x^13 + ...
copeland
2016-03-05 21:01:12
If $P(x)=5x^{13}+\cdots,$ then $$x(P(x+1)-P(x))=13\cdot 5x^{13}+\cdots$$
If $P(x)=5x^{13}+\cdots,$ then $$x(P(x+1)-P(x))=13\cdot 5x^{13}+\cdots$$
copeland
2016-03-05 21:01:25
Hmm.
Hmm.
copeland
2016-03-05 21:01:33
Maybe this finite differences thing isn't working so well.
Maybe this finite differences thing isn't working so well.
copeland
2016-03-05 21:01:36
Let's come back to it in a bit.
Let's come back to it in a bit.
copeland
2016-03-05 21:01:57
Our functional equation is\[(x-1)P(x+1) = (x+2)P(x).\]
Our functional equation is\[(x-1)P(x+1) = (x+2)P(x).\]
copeland
2016-03-05 21:02:10
What does it "say"?
What does it "say"?
brian22
2016-03-05 21:03:10
copeland, I will NOT be putting a polynomial FE into words
copeland, I will NOT be putting a polynomial FE into words
copeland
2016-03-05 21:03:13
copeland
2016-03-05 21:03:15
Let me:
Let me:
copeland
2016-03-05 21:03:16
The left side says, "I take $P$ and shift it to the right by one and then add a root at $1$."
The left side says, "I take $P$ and shift it to the right by one and then add a root at $1$."
copeland
2016-03-05 21:03:21
What does the right side say?
What does the right side say?
atmchallenge
2016-03-05 21:04:09
I take $P$ and add a root at $-2$.
I take $P$ and add a root at $-2$.
TheRealDeal
2016-03-05 21:04:09
add a root at -2
add a root at -2
walnutwaldo20
2016-03-05 21:04:09
add a root at -2
add a root at -2
andsun19
2016-03-05 21:04:09
I take P, and add a root at -2?
I take P, and add a root at -2?
ychen
2016-03-05 21:04:09
add a root at -2
add a root at -2
nukelauncher
2016-03-05 21:04:09
add a root of -2
add a root of -2
SuperMaltese
2016-03-05 21:04:09
i take P and add a root at -2
i take P and add a root at -2
blizzard10
2016-03-05 21:04:09
I will take P and add a root at -2
I will take P and add a root at -2
kikipet
2016-03-05 21:04:09
I take P and add a root at -2
I take P and add a root at -2
copeland
2016-03-05 21:04:11
The right side says, "I take $P$ and add a root at $-2$."
The right side says, "I take $P$ and add a root at $-2$."
copeland
2016-03-05 21:04:46
So shifting right and adding a root at 1 is the same as doing nothing and adding a root at -2. What does that say about the roots of $P?$
So shifting right and adding a root at 1 is the same as doing nothing and adding a root at -2. What does that say about the roots of $P?$
brian6liu
2016-03-05 21:06:12
they're 0, 1, and -1
they're 0, 1, and -1
andsun19
2016-03-05 21:06:12
uh... 3 roots?
uh... 3 roots?
nosaj
2016-03-05 21:06:12
between 1 and -2
between 1 and -2
Aragorn66
2016-03-05 21:06:12
there are 3 of them
there are 3 of them
nukelauncher
2016-03-05 21:06:12
it has 3 root?
it has 3 root?
copeland
2016-03-05 21:06:14
The functional equation just says that the roots of $P$ are a sequence of integers starting at 1 and ending at -1.
The functional equation just says that the roots of $P$ are a sequence of integers starting at 1 and ending at -1.
copeland
2016-03-05 21:06:35
Here's a third approach:
Here's a third approach:
calculus_riju
2016-03-05 21:06:42
substituting Q(x) in the mother equation we have Q(x+1)=Q(x). Thus its a constant function. So P(x) is cubic
substituting Q(x) in the mother equation we have Q(x+1)=Q(x). Thus its a constant function. So P(x) is cubic
copeland
2016-03-05 21:06:59
Is that right? What happens when we substitute our expression with $Q$ into the mother equation?
Is that right? What happens when we substitute our expression with $Q$ into the mother equation?
brian6liu
2016-03-05 21:08:45
x(x-1)(x+1)(x+2)Q(x+1)=x(x-1)(x+1)(x+2)Q(x)
x(x-1)(x+1)(x+2)Q(x+1)=x(x-1)(x+1)(x+2)Q(x)
PurplePancakes
2016-03-05 21:08:45
(x-1)(x)(x+1)(x+2)Q(x+1)=(x+2)(x-1)(x)(x+1)Q(x)
(x-1)(x)(x+1)(x+2)Q(x+1)=(x+2)(x-1)(x)(x+1)Q(x)
ninjataco
2016-03-05 21:08:45
Q(x+1) = Q(x) so Q is constant
Q(x+1) = Q(x) so Q is constant
copeland
2016-03-05 21:08:47
Indeed, Substituting $P(x) = (x-1)x(x+1)Q(x)$ into our original equation simplifies to $Q(x+1)=Q(x),$ so $Q$ is indeed constant (as it is a polynomial).
Indeed, Substituting $P(x) = (x-1)x(x+1)Q(x)$ into our original equation simplifies to $Q(x+1)=Q(x),$ so $Q$ is indeed constant (as it is a polynomial).
copeland
2016-03-05 21:08:56
So $P(x) = c(x-1)x(x+1)$. How can we find $c$?
So $P(x) = c(x-1)x(x+1)$. How can we find $c$?
SimonSun
2016-03-05 21:09:37
2/3
2/3
ninjataco
2016-03-05 21:09:37
solve for P(2)
solve for P(2)
epiphany
2016-03-05 21:09:37
we know that P(2)=4
we know that P(2)=4
walnutwaldo20
2016-03-05 21:09:37
Use P(2)^2 = P(3)
Use P(2)^2 = P(3)
epiphany
2016-03-05 21:09:37
and P(3)=16
and P(3)=16
SimonSun
2016-03-05 21:09:37
plug in 2
plug in 2
AlcumusGuy
2016-03-05 21:09:37
(P(2))^2 = P(3)
(P(2))^2 = P(3)
PurplePancakes
2016-03-05 21:09:37
the other condition
the other condition
ryanyz10
2016-03-05 21:09:37
use the other thing we're given
use the other thing we're given
copeland
2016-03-05 21:09:40
We have another condition: $((P(2))^2 = P(3)$. Plugging into that we have \[ (6c)^2 = c\cdot 2 \cdot 3 \cdot 4 = 24c.\] So what is $c$?
We have another condition: $((P(2))^2 = P(3)$. Plugging into that we have \[ (6c)^2 = c\cdot 2 \cdot 3 \cdot 4 = 24c.\] So what is $c$?
bestwillcui1
2016-03-05 21:10:08
2/3
2/3
SimonSun
2016-03-05 21:10:08
2/3
2/3
ninjataco
2016-03-05 21:10:08
2/3
2/3
PurplePancakes
2016-03-05 21:10:08
2/3
2/3
brian6liu
2016-03-05 21:10:08
2/3
2/3
nosaj
2016-03-05 21:10:08
2/3
2/3
AlcumusGuy
2016-03-05 21:10:08
$c = \frac{2}{3}$
$c = \frac{2}{3}$
blue8931
2016-03-05 21:10:08
2/3
2/3
copeland
2016-03-05 21:10:10
We have $c = \dfrac 23$.
We have $c = \dfrac 23$.
copeland
2016-03-05 21:10:48
So what is our answer?
So what is our answer?
blue8931
2016-03-05 21:11:23
just plug in 7/2 to get the answer of 105/4 ----> $\boxed{109}$
just plug in 7/2 to get the answer of 105/4 ----> $\boxed{109}$
blizzard10
2016-03-05 21:11:23
Now plug in at $\frac{7}{2}$
Now plug in at $\frac{7}{2}$
illogical_21
2016-03-05 21:11:23
Plug in 7/2,
Plug in 7/2,
nosaj
2016-03-05 21:11:23
Finally, we have $P\left(\frac 72 \right) = \frac 23 \cdot \frac 52 \cdot \frac 72 \cdot \frac 92 = \frac{105}{4}$, so our answer is $\boxed{109}$.
Finally, we have $P\left(\frac 72 \right) = \frac 23 \cdot \frac 52 \cdot \frac 72 \cdot \frac 92 = \frac{105}{4}$, so our answer is $\boxed{109}$.
mewtwomew
2016-03-05 21:11:23
109
109
andsun19
2016-03-05 21:11:23
109
109
algebra_star1234
2016-03-05 21:11:23
105/4 --> 109
105/4 --> 109
SimonSun
2016-03-05 21:11:23
105/4=109
105/4=109
blizzard10
2016-03-05 21:11:23
109
109
copeland
2016-03-05 21:11:26
We can just substitute $\dfrac 72$ into $P(x) = \dfrac 23 (x-1)x(x+1)$ to get \[P\left(\frac 72\right) = \frac 23 \cdot \frac 52 \cdot \frac 72 \cdot \frac 92 = \frac{105}{4}.\] So our answer is $105+4=\boxed{109}.$
We can just substitute $\dfrac 72$ into $P(x) = \dfrac 23 (x-1)x(x+1)$ to get \[P\left(\frac 72\right) = \frac 23 \cdot \frac 52 \cdot \frac 72 \cdot \frac 92 = \frac{105}{4}.\] So our answer is $105+4=\boxed{109}.$
copeland
2016-03-05 21:11:40
Great.
Great.
copeland
2016-03-05 21:11:47
Also, to tie up that finite differences thing.
Also, to tie up that finite differences thing.
copeland
2016-03-05 21:12:21
When you take a polynomial of the form $P(x)=a_nx^n$ and you compute $\Delta P(x)=P(x+1)-P(x),$ you get a new polynomial.
When you take a polynomial of the form $P(x)=a_nx^n$ and you compute $\Delta P(x)=P(x+1)-P(x),$ you get a new polynomial.
copeland
2016-03-05 21:12:38
This new polynomial has degree $n-1$ and the leading term is $na_n$.
This new polynomial has degree $n-1$ and the leading term is $na_n$.
copeland
2016-03-05 21:12:57
We were here:\[x\left(P(x+1) - P(x) \right) = P(x+1) + 2P(x).\]
We were here:\[x\left(P(x+1) - P(x) \right) = P(x+1) + 2P(x).\]
copeland
2016-03-05 21:13:11
The leading term on the left side gets a coefficient of 3.
The leading term on the left side gets a coefficient of 3.
copeland
2016-03-05 21:13:17
That means the degree of $P$ must be 3.
That means the degree of $P$ must be 3.
kikipet
2016-03-05 21:13:21
is finite differences like a calculus thing?
is finite differences like a calculus thing?
hamup1
2016-03-05 21:13:21
so basically the derivative
so basically the derivative
copeland
2016-03-05 21:13:27
More or less, yes.
More or less, yes.
copeland
2016-03-05 21:13:47
In fact, you can use calculus both to remember and to prove the leading term stuff I just mentioned.
In fact, you can use calculus both to remember and to prove the leading term stuff I just mentioned.
copeland
2016-03-05 21:13:53
Where were we again?
Where were we again?
blizzard10
2016-03-05 21:14:16
Problem 12
Problem 12
weilunsun28
2016-03-05 21:14:16
11+1=13...-1=12
11+1=13...-1=12
Aragorn66
2016-03-05 21:14:16
problem 12
problem 12
copeland
2016-03-05 21:14:22
12. Find the least positive integer $m$ such that $m^2-m+11$ is a product of at least four not necessarily distinct primes.
12. Find the least positive integer $m$ such that $m^2-m+11$ is a product of at least four not necessarily distinct primes.
copeland
2016-03-05 21:14:26
OK, we're trying to minimize $m$ even though we're looking at some quadratic in $m$. Is that a problem?
OK, we're trying to minimize $m$ even though we're looking at some quadratic in $m$. Is that a problem?
mewtwomew
2016-03-05 21:15:16
no
no
illogical_21
2016-03-05 21:15:16
no
no
leeandrew1029gmail.com
2016-03-05 21:15:16
no
no
idomath12345
2016-03-05 21:15:16
No?
No?
copeland
2016-03-05 21:15:22
Why not?
Why not?
hliu70
2016-03-05 21:15:26
no, a lower m for the most part means a lower result.
no, a lower m for the most part means a lower result.
copeland
2016-03-05 21:15:30
How can you tell?
How can you tell?
kikipet
2016-03-05 21:16:03
Graph it!
Graph it!
ninjataco
2016-03-05 21:16:03
because the quadratic is increasing on the positive integers
because the quadratic is increasing on the positive integers
illogical_21
2016-03-05 21:16:03
coefficient of m^2 is positive
coefficient of m^2 is positive
idomath12345
2016-03-05 21:16:03
cuz strictly increasing
cuz strictly increasing
PurplePancakes
2016-03-05 21:16:03
m^2>m for sufficiently larg m (in this case 2)
m^2>m for sufficiently larg m (in this case 2)
qwerty733
2016-03-05 21:16:03
Because it's m(m-1)+11, and for positives, this increases
Because it's m(m-1)+11, and for positives, this increases
copeland
2016-03-05 21:16:06
No, the vertex of this parabola is at $\dfrac12$ so the function is strictly increasing on the positive integers. Minimizing $m$ is exactly the same as minimizing $m^2-m+11.$
No, the vertex of this parabola is at $\dfrac12$ so the function is strictly increasing on the positive integers. Minimizing $m$ is exactly the same as minimizing $m^2-m+11.$
copeland
2016-03-05 21:16:08
OK, so the smallest this can be is $2^4$. Is it possible for $m^2-m+11$ to be equal to $2^4$?
OK, so the smallest this can be is $2^4$. Is it possible for $m^2-m+11$ to be equal to $2^4$?
nosaj
2016-03-05 21:16:40
nope cuz it's odd
nope cuz it's odd
WL0410
2016-03-05 21:16:40
No, it's never divisible by 2
No, it's never divisible by 2
ninjataco
2016-03-05 21:16:40
no because it's 1 mod 2
no because it's 1 mod 2
hamup1
2016-03-05 21:16:40
no way, even + odd cannot equal even
no way, even + odd cannot equal even
walnutwaldo20
2016-03-05 21:16:40
no
no
copeland
2016-03-05 21:16:42
No! In fact, if $m$ is even then $m^2-m+11$ is a sum of two evens and an odd so it is odd. If $m$ is odd, $m^2-m+11$ is a sum of three odds so it is odd.
No! In fact, if $m$ is even then $m^2-m+11$ is a sum of two evens and an odd so it is odd. If $m$ is odd, $m^2-m+11$ is a sum of three odds so it is odd.
copeland
2016-03-05 21:16:43
Let's make that more formal since it sounds useful.
Let's make that more formal since it sounds useful.
copeland
2016-03-05 21:17:02
Modulo 2, we have the polynomial congruence:\[m^2-m+11\equiv0\pmod{2}.\]This congruence has no solutions. That's the same as saying that we can't factor the polynomial (since it's quadratic).
Modulo 2, we have the polynomial congruence:\[m^2-m+11\equiv0\pmod{2}.\]This congruence has no solutions. That's the same as saying that we can't factor the polynomial (since it's quadratic).
copeland
2016-03-05 21:17:12
This gives us a model for looking for the primes in question. What do we check next?
This gives us a model for looking for the primes in question. What do we check next?
blue8931
2016-03-05 21:17:41
3
3
AlcumusGuy
2016-03-05 21:17:41
Mod 3
Mod 3
19bobhu
2016-03-05 21:17:41
3^4
3^4
hamup1
2016-03-05 21:17:41
mod 3!
mod 3!
ninjataco
2016-03-05 21:17:41
mod 3
mod 3
epiphany
2016-03-05 21:17:41
mod 3
mod 3
copeland
2016-03-05 21:17:44
Let's look at \[m^2-m+11\equiv0\pmod3.\]That's the same as \[m^2+2m+2\equiv0\pmod3.\]
Let's look at \[m^2-m+11\equiv0\pmod3.\]That's the same as \[m^2+2m+2\equiv0\pmod3.\]
copeland
2016-03-05 21:17:47
Can you prove that doesn't factor?
Can you prove that doesn't factor?
algebra_star1234
2016-03-05 21:18:10
yes
yes
jfurf
2016-03-05 21:18:10
Yep...
Yep...
Aragorn66
2016-03-05 21:18:10
yes
yes
PurplePancakes
2016-03-05 21:18:10
check the residues, and it doesn't work
check the residues, and it doesn't work
copeland
2016-03-05 21:18:17
We can just substitute stuff, sure.
We can just substitute stuff, sure.
brian22
2016-03-05 21:18:24
$(m+1)^2+1$
$(m+1)^2+1$
WL0410
2016-03-05 21:18:24
$(m+1)^2\equiv -1 \mod 3$ has no solutions
$(m+1)^2\equiv -1 \mod 3$ has no solutions
copeland
2016-03-05 21:18:30
We can also complete the square. This is the same as \[(m+1)^2\equiv-1\pmod3.\]
We can also complete the square. This is the same as \[(m+1)^2\equiv-1\pmod3.\]
copeland
2016-03-05 21:18:34
Are there any solutions?
Are there any solutions?
kikipet
2016-03-05 21:18:54
no
no
ninjataco
2016-03-05 21:18:54
no, 2 is not a quadratic residue mod 3
no, 2 is not a quadratic residue mod 3
blue8931
2016-03-05 21:18:54
not for mod 3
not for mod 3
AlcumusGuy
2016-03-05 21:18:54
no since squares are never are only congruent to 0 or 1 mod 3
no since squares are never are only congruent to 0 or 1 mod 3
MikoTennisPro
2016-03-05 21:18:54
nope
nope
copeland
2016-03-05 21:18:57
No! 0 and 1 are the only squares modulo 3 so that's out.
No! 0 and 1 are the only squares modulo 3 so that's out.
copeland
2016-03-05 21:18:58
Next?
Next?
mssmath
2016-03-05 21:19:22
5
5
hamup1
2016-03-05 21:19:22
mod 5
mod 5
azmath333
2016-03-05 21:19:22
5
5
Locust
2016-03-05 21:19:22
5
5
epgy8
2016-03-05 21:19:22
Mod 5?
Mod 5?
brian6liu
2016-03-05 21:19:22
mod 5
mod 5
copeland
2016-03-05 21:19:24
Don't mind if I do!
Don't mind if I do!
copeland
2016-03-05 21:19:25
Next is 5:\[m^2-m+11\equiv0\pmod5\] is the same as \[m^2+4m+1\equiv0\pmod5.\]
Next is 5:\[m^2-m+11\equiv0\pmod5\] is the same as \[m^2+4m+1\equiv0\pmod5.\]
copeland
2016-03-05 21:19:28
What can we do with that?
What can we do with that?
AlcumusGuy
2016-03-05 21:20:08
complete the square again
complete the square again
ninjataco
2016-03-05 21:20:08
add 3 to both sides to complete the square
add 3 to both sides to complete the square
rkm0959
2016-03-05 21:20:08
$(m+2)^2 \equiv 3 \pmod{5}$
$(m+2)^2 \equiv 3 \pmod{5}$
brian6liu
2016-03-05 21:20:08
(m+2)^2=3 mod 5
(m+2)^2=3 mod 5
hamup1
2016-03-05 21:20:08
add 3 and complete the square
add 3 and complete the square
WL0410
2016-03-05 21:20:08
$(m+2)^2\equiv 3 \mod 5$, no solutions
$(m+2)^2\equiv 3 \mod 5$, no solutions
copeland
2016-03-05 21:20:10
To complete the square we write \[m^2+4m+1\equiv (m^2+4m+4)+2.\] Completing the square again gives\[(m+2)^2\equiv3\pmod5.\]
To complete the square we write \[m^2+4m+1\equiv (m^2+4m+4)+2.\] Completing the square again gives\[(m+2)^2\equiv3\pmod5.\]
copeland
2016-03-05 21:20:11
Is 3 a square?
Is 3 a square?
sxu
2016-03-05 21:20:26
NO
NO
PurplePancakes
2016-03-05 21:20:26
no
no
Aragorn66
2016-03-05 21:20:26
no
no
TheRealDeal
2016-03-05 21:20:26
no
no
kikipet
2016-03-05 21:20:26
no
no
copeland
2016-03-05 21:20:30
No. The squares are 0, 1, and 4.
No. The squares are 0, 1, and 4.
copeland
2016-03-05 21:20:30
On to 7!
On to 7!
rkm0959
2016-03-05 21:21:10
$m^2+6m+4 \equiv 0 \pmod{7}$ so $(m+3)^2 \equiv 5 \pmod{7}$, impossible as the squares are $0,1,2,4$
$m^2+6m+4 \equiv 0 \pmod{7}$ so $(m+3)^2 \equiv 5 \pmod{7}$, impossible as the squares are $0,1,2,4$
copeland
2016-03-05 21:21:12
We get \[m^2-m+11\equiv m^2+6m+4\equiv (m+3)^2-5\pmod7.\] This means we want to solve \[(m-3)^2\equiv 5\pmod7.\]
We get \[m^2-m+11\equiv m^2+6m+4\equiv (m+3)^2-5\pmod7.\] This means we want to solve \[(m-3)^2\equiv 5\pmod7.\]
copeland
2016-03-05 21:21:55
The squares modulo 7 are {0,1,4,2}, so 7 does not divide our number either.
The squares modulo 7 are {0,1,4,2}, so 7 does not divide our number either.
copeland
2016-03-05 21:21:56
On to 11. Do you see any solutions to $m^2-m+11\equiv0\pmod{11}?$
On to 11. Do you see any solutions to $m^2-m+11\equiv0\pmod{11}?$
ninjataco
2016-03-05 21:22:13
m = 0 or 1 (mod 11)
m = 0 or 1 (mod 11)
PurplePancakes
2016-03-05 21:22:13
m is a multiple of 11
m is a multiple of 11
MikoTennisPro
2016-03-05 21:22:13
m=11
m=11
andsun19
2016-03-05 21:22:13
11, 12
11, 12
blue8931
2016-03-05 21:22:13
yes! when m is a multiple of 11
yes! when m is a multiple of 11
andsun19
2016-03-05 21:22:13
anything divisible by 11 or one more than a multiple of 11
anything divisible by 11 or one more than a multiple of 11
eswa2000
2016-03-05 21:22:13
m is divisible by 11
m is divisible by 11
Ericaops
2016-03-05 21:22:13
11
11
copeland
2016-03-05 21:22:19
This simplifies to $m(m-1) \equiv 0 \pmod {11}$. This is clearly possible if $m$ or $m-1$ is divisible by 11, so $m^2 - m + 11$ can be divisible by 11. (And further, every prime dividing $m^2 - m + 11$ must be at least 11.)
This simplifies to $m(m-1) \equiv 0 \pmod {11}$. This is clearly possible if $m$ or $m-1$ is divisible by 11, so $m^2 - m + 11$ can be divisible by 11. (And further, every prime dividing $m^2 - m + 11$ must be at least 11.)
copeland
2016-03-05 21:22:25
What should we check now?
What should we check now?
ninjataco
2016-03-05 21:22:50
whether m^2 - m + 11 can be 11^4
whether m^2 - m + 11 can be 11^4
lucylou
2016-03-05 21:22:50
11^4
11^4
blue8931
2016-03-05 21:22:50
if $11^4$ works
if $11^4$ works
brian6liu
2016-03-05 21:22:50
if 11^4 works
if 11^4 works
AlcumusGuy
2016-03-05 21:22:50
$11^4$
$11^4$
copeland
2016-03-05 21:22:52
The smallest possible number is $11^4$. Now we should check whether we can solve $11^4=m^2-m+11$.
The smallest possible number is $11^4$. Now we should check whether we can solve $11^4=m^2-m+11$.
copeland
2016-03-05 21:22:54
This simplifies to $m(m-1)=11^4-11.$
This simplifies to $m(m-1)=11^4-11.$
copeland
2016-03-05 21:22:59
About how big is the $m$ that solves this equation?
About how big is the $m$ that solves this equation?
nosaj
2016-03-05 21:23:22
121 ish?
121 ish?
sxu
2016-03-05 21:23:22
121
121
swagger21
2016-03-05 21:23:22
~121
~121
andsun19
2016-03-05 21:23:22
11^2 or 121
11^2 or 121
Locust
2016-03-05 21:23:22
Somewhere around 11^2
Somewhere around 11^2
Pot
2016-03-05 21:23:22
121
121
Aragorn66
2016-03-05 21:23:22
121
121
copeland
2016-03-05 21:23:24
The left side is a little less than $m^2$ and the right side is a little less than $121^2.$ Therefore $m\approx121$.
The left side is a little less than $m^2$ and the right side is a little less than $121^2.$ Therefore $m\approx121$.
copeland
2016-03-05 21:23:25
Does $m=121$ solve the equation?
Does $m=121$ solve the equation?
algebra_star1234
2016-03-05 21:23:39
no
no
Aragorn66
2016-03-05 21:23:39
no
no
ninjataco
2016-03-05 21:23:39
no
no
Pot
2016-03-05 21:23:39
no
no
epiphany
2016-03-05 21:23:39
no!
no!
brian6liu
2016-03-05 21:23:39
no
no
nosaj
2016-03-05 21:23:39
nope
nope
copeland
2016-03-05 21:23:41
No. $m(m-1)=121^2-121$ and $11^4-11=121^2-11$. This $m$ is too small.
No. $m(m-1)=121^2-121$ and $11^4-11=121^2-11$. This $m$ is too small.
copeland
2016-03-05 21:23:42
Does $m=122$ solve this equation?
Does $m=122$ solve this equation?
Pot
2016-03-05 21:23:56
no
no
brian6liu
2016-03-05 21:23:56
no
no
ninjataco
2016-03-05 21:23:56
no
no
AlcumusGuy
2016-03-05 21:23:56
no
no
rkm0959
2016-03-05 21:23:56
Nope
Nope
leeandrew1029gmail.com
2016-03-05 21:23:56
no
no
copeland
2016-03-05 21:23:58
No. $m(m-1)=122\cdot121=121^2+121$ and this is too big. This shows that $11^4$ is not of this form.
No. $m(m-1)=122\cdot121=121^2+121$ and this is too big. This shows that $11^4$ is not of this form.
copeland
2016-03-05 21:24:00
What now?
What now?
Pot
2016-03-05 21:24:26
11^3 * 13
11^3 * 13
mssmath
2016-03-05 21:24:26
11^3*13
11^3*13
brian6liu
2016-03-05 21:24:26
11^3*13
11^3*13
blue8931
2016-03-05 21:24:26
then we have no check if $11^3 * 13$ works
then we have no check if $11^3 * 13$ works
epiphany
2016-03-05 21:24:26
11^3*13
11^3*13
AlcumusGuy
2016-03-05 21:24:26
$11^3 \cdot 13$ is next smallest
$11^3 \cdot 13$ is next smallest
azmath333
2016-03-05 21:24:26
11^3 * 13
11^3 * 13
Ericaops
2016-03-05 21:24:26
11^3*13
11^3*13
PurplePancakes
2016-03-05 21:24:26
11^3*13?
11^3*13?
rkm0959
2016-03-05 21:24:26
So we move on to $13 \cdot 11^3$ since it's the next smallest one
So we move on to $13 \cdot 11^3$ since it's the next smallest one
blue8931
2016-03-05 21:24:26
have to check if $11^3 * 13$ works
have to check if $11^3 * 13$ works
copeland
2016-03-05 21:24:47
We could go the optimistic route. The next smallest value we could attain is $11^3\cdot13.$
We could go the optimistic route. The next smallest value we could attain is $11^3\cdot13.$
copeland
2016-03-05 21:24:50
Let's try to solve $m^2-m+11=11^3\cdot13$.
Let's try to solve $m^2-m+11=11^3\cdot13$.
copeland
2016-03-05 21:24:55
What is this equation approximately?
What is this equation approximately?
jfurf
2016-03-05 21:25:36
121 times 143
121 times 143
sxu
2016-03-05 21:25:36
m squared-m=11 cubed times 12
m squared-m=11 cubed times 12
Aragorn66
2016-03-05 21:25:36
132 ish
132 ish
calculus_riju
2016-03-05 21:25:36
11.(k^2-k+1) and check k to get three primes
11.(k^2-k+1) and check k to get three primes
brian6liu
2016-03-05 21:25:40
m=132
m=132
Ericaops
2016-03-05 21:25:40
132
132
epiphany
2016-03-05 21:25:40
132
132
copeland
2016-03-05 21:25:44
This is approximately $m^2\approx11^2(13\cdot11)\approx11^2\cdot12^2.$
This is approximately $m^2\approx11^2(13\cdot11)\approx11^2\cdot12^2.$
copeland
2016-03-05 21:25:53
What happens if we plug $m=11\cdot12$ into our equation?
What happens if we plug $m=11\cdot12$ into our equation?
blue8931
2016-03-05 21:26:29
this actually works! m= $\boxed{132}$
this actually works! m= $\boxed{132}$
brian6liu
2016-03-05 21:26:29
it works
it works
SimonSun
2016-03-05 21:26:29
it works!!!!
it works!!!!
AlcumusGuy
2016-03-05 21:26:29
132 works!
132 works!
epiphany
2016-03-05 21:26:29
it works! we are done 132
it works! we are done 132
blue8931
2016-03-05 21:26:29
it works!
it works!
Aragorn66
2016-03-05 21:26:29
it works
it works
PurplePancakes
2016-03-05 21:26:29
it works
it works
copeland
2016-03-05 21:26:33
OK, let's try it:\begin{align*}
m^2-m+11&=(11\cdot12)^2-11\cdot12+11\\
&=(11\cdot12)^2-11^2\\
&=(11\cdot12+11)(11\cdot12-11)\\
&=(11\cdot13)(11\cdot11)\\
&=11^3\cdot13.
\end{align*}
OK, let's try it:\begin{align*}
m^2-m+11&=(11\cdot12)^2-11\cdot12+11\\
&=(11\cdot12)^2-11^2\\
&=(11\cdot12+11)(11\cdot12-11)\\
&=(11\cdot13)(11\cdot11)\\
&=11^3\cdot13.
\end{align*}
copeland
2016-03-05 21:26:41
And the answer?
And the answer?
illogical_21
2016-03-05 21:27:06
m=132
m=132
kikipet
2016-03-05 21:27:06
132
132
walnutwaldo20
2016-03-05 21:27:06
132!
132!
ychen
2016-03-05 21:27:06
132
132
algebra_star1234
2016-03-05 21:27:06
132
132
andsun19
2016-03-05 21:27:06
Optimism FTW. 132
Optimism FTW. 132
Ericaops
2016-03-05 21:27:06
132
132
SK200
2016-03-05 21:27:06
132
132
mssmath
2016-03-05 21:27:06
m=132
m=132
blue8931
2016-03-05 21:27:06
$\boxed{132}$
$\boxed{132}$
copeland
2016-03-05 21:27:08
The smallest value occurs at $m=11\cdot12=\boxed{132}.$
The smallest value occurs at $m=11\cdot12=\boxed{132}.$
copeland
2016-03-05 21:27:11
Good.
Good.
copeland
2016-03-05 21:27:14
Ready for more?
Ready for more?
calculus_riju
2016-03-05 21:27:36
#13
#13
Transcranial
2016-03-05 21:27:36
Yea
Yea
brian6liu
2016-03-05 21:27:36
yeah
yeah
itised
2016-03-05 21:27:36
yep
yep
Aragorn66
2016-03-05 21:27:36
yup
yup
SimonSun
2016-03-05 21:27:36
yessir
yessir
algebra_star1234
2016-03-05 21:27:36
yes
yes
andsun19
2016-03-05 21:27:36
yessir
yessir
ychen
2016-03-05 21:27:36
13 is my best question
13 is my best question
illogical_21
2016-03-05 21:27:36
on to 13!
on to 13!
PurplePancakes
2016-03-05 21:27:36
13 was my favorite problem on the test... LET'S DO IT
13 was my favorite problem on the test... LET'S DO IT
copeland
2016-03-05 21:27:42
13. Freddy the frog is is jumping aroudn the coordinate plane searching for a river, which lies on the horizontal line $y=24.$ A fence is located at the horizontal line $y=0.$ On each jump Freddy randomly chooses a dirction parallel to one of the coordinate axes and moves one unit in that direction. When he is at a point where $y=0,$ with equal likelihoods he chooses one of three directions where he either jumps parallel to the fence or jumps away from the fence, but he never chooses the direction that would have him cross over the fence to where $y<0.$ Freddy starts his search at the point $(0,21)$ and will stop once he reaches a point on the river. Find the expected number of jumps it will take Freddy to reach the river.
13. Freddy the frog is is jumping aroudn the coordinate plane searching for a river, which lies on the horizontal line $y=24.$ A fence is located at the horizontal line $y=0.$ On each jump Freddy randomly chooses a dirction parallel to one of the coordinate axes and moves one unit in that direction. When he is at a point where $y=0,$ with equal likelihoods he chooses one of three directions where he either jumps parallel to the fence or jumps away from the fence, but he never chooses the direction that would have him cross over the fence to where $y<0.$ Freddy starts his search at the point $(0,21)$ and will stop once he reaches a point on the river. Find the expected number of jumps it will take Freddy to reach the river.
copeland
2016-03-05 21:27:48
OK, this is a pretty standard set-up. What should we do?
OK, this is a pretty standard set-up. What should we do?
idomath12345
2016-03-05 21:28:33
RECURSION.
RECURSION.
ninjataco
2016-03-05 21:28:33
make one of those nifty state diagrams?
make one of those nifty state diagrams?
PurplePancakes
2016-03-05 21:28:33
I solved it with recursion on the actual test
I solved it with recursion on the actual test
AlcumusGuy
2016-03-05 21:28:33
Let $E(n)$ be the expected number of steps starting from y = n
Let $E(n)$ be the expected number of steps starting from y = n
brian6liu
2016-03-05 21:28:33
let E_i be the expected steps it takes freddy when he starts out i steps away from the river
let E_i be the expected steps it takes freddy when he starts out i steps away from the river
copeland
2016-03-05 21:28:36
We want the expected number of jumps Freddy will take to get to the river starting at $(0,21)$.
We want the expected number of jumps Freddy will take to get to the river starting at $(0,21)$.
copeland
2016-03-05 21:28:37
This value depends on the expected number of steps Freddy takes from $(1,21)$, $(-1,21)$, $(0,22)$ and $(0,20)$.
This value depends on the expected number of steps Freddy takes from $(1,21)$, $(-1,21)$, $(0,22)$ and $(0,20)$.
copeland
2016-03-05 21:28:39
And what simplification is immediate?
And what simplification is immediate?
PurplePancakes
2016-03-05 21:29:23
all expected values for (n,21) are the same
all expected values for (n,21) are the same
copeland
2016-03-05 21:29:26
The expected number of steps Freddy takes from $(a,n)$ is independent of $a$, since the river and the fence are both horizontal.
The expected number of steps Freddy takes from $(a,n)$ is independent of $a$, since the river and the fence are both horizontal.
copeland
2016-03-05 21:29:29
Specifically, the expected time to reach the river is the same for any point on the line $y=n$. Let's let $e_n$ be the expected number of jumps to the river from the line $y=n$.
Specifically, the expected time to reach the river is the same for any point on the line $y=n$. Let's let $e_n$ be the expected number of jumps to the river from the line $y=n$.
copeland
2016-03-05 21:29:31
Which value do we know immediately?
Which value do we know immediately?
idomath12345
2016-03-05 21:30:04
e24 is 0
e24 is 0
idomath12345
2016-03-05 21:30:04
e24
e24
Pot
2016-03-05 21:30:04
when n=24, en = 0
when n=24, en = 0
ninjataco
2016-03-05 21:30:04
e_24 = 0
e_24 = 0
sxu
2016-03-05 21:30:04
n=24
n=24
andsun19
2016-03-05 21:30:04
24
24
deltaepsilon6
2016-03-05 21:30:04
n=24
n=24
copeland
2016-03-05 21:30:07
Of course $e_{24}=0$ since if Freddy is at the river, he doesn't need to move at all to get to the river.
Of course $e_{24}=0$ since if Freddy is at the river, he doesn't need to move at all to get to the river.
copeland
2016-03-05 21:30:09
What can we say about $e_{23}?$
What can we say about $e_{23}?$
AlcumusGuy
2016-03-05 21:31:12
e(23) = 1/2 e(23) + 1/4 e(24) + 1/4 e(22) + 1
e(23) = 1/2 e(23) + 1/4 e(24) + 1/4 e(22) + 1
idomath12345
2016-03-05 21:31:12
1/2e23+1/4e22+1/4e24+1
1/2e23+1/4e22+1/4e24+1
ninjataco
2016-03-05 21:31:12
e_23 = 1/4 e_22 + 1/2 e_23 + 1
e_23 = 1/4 e_22 + 1/2 e_23 + 1
copeland
2016-03-05 21:31:15
When Freddy is on the line $y=23,$ he will hop one of each of the four directions with equal probability. The time it takes him to get to the river after the hop is the average of the times it takes from the four possible places he lands. That means the expected time from his current position is one more than the average of the expected times from the new positions.
When Freddy is on the line $y=23,$ he will hop one of each of the four directions with equal probability. The time it takes him to get to the river after the hop is the average of the times it takes from the four possible places he lands. That means the expected time from his current position is one more than the average of the expected times from the new positions.
copeland
2016-03-05 21:31:24
He hops to $y=24$ with probability $\dfrac14$, he hops to $y=22$ with probability $\dfrac14$ and he hops horizontally to $y=23$ with probability $\dfrac24$.
He hops to $y=24$ with probability $\dfrac14$, he hops to $y=22$ with probability $\dfrac14$ and he hops horizontally to $y=23$ with probability $\dfrac24$.
copeland
2016-03-05 21:31:27
Therefore
Therefore
copeland
2016-03-05 21:31:30
\[e_{23}=1+\frac14(e_{24}+e_{22}+2e_{23}).\]
\[e_{23}=1+\frac14(e_{24}+e_{22}+2e_{23}).\]
copeland
2016-03-05 21:31:34
What about $e_{22}$?
What about $e_{22}$?
PurplePancakes
2016-03-05 21:32:17
basically the same thing
basically the same thing
sxu
2016-03-05 21:32:17
same, excep with the #s switched
same, excep with the #s switched
MikoTennisPro
2016-03-05 21:32:17
same thing except with 23,21,22
same thing except with 23,21,22
brian6liu
2016-03-05 21:32:17
same thing with all the indices shifted
same thing with all the indices shifted
andsun19
2016-03-05 21:32:17
e22 can be same done the same way
e22 can be same done the same way
AlcumusGuy
2016-03-05 21:32:17
same structure: e22 = 1 + 1/4(e23 + e21 + 2e22)
same structure: e22 = 1 + 1/4(e23 + e21 + 2e22)
hamup1
2016-03-05 21:32:17
same thing
same thing
copeland
2016-03-05 21:32:21
\[e_{22}=1+\frac14(e_{23}+e_{21}+2e_{22}).\]
\[e_{22}=1+\frac14(e_{23}+e_{21}+2e_{22}).\]
copeland
2016-03-05 21:32:23
Also:\[e_{21}=1+\frac14(e_{22}+e_{20}+2e_{21}).\]
Also:\[e_{21}=1+\frac14(e_{22}+e_{20}+2e_{21}).\]
copeland
2016-03-05 21:32:28
This keeps going until. . .
This keeps going until. . .
sxu
2016-03-05 21:32:50
y=0!
y=0!
epiphany
2016-03-05 21:32:50
e0
e0
mewtwomew
2016-03-05 21:32:50
e0
e0
idomath12345
2016-03-05 21:32:50
Keep going until e0?
Keep going until e0?
Pot
2016-03-05 21:32:50
e_0
e_0
hamup1
2016-03-05 21:32:50
$e_{0}$
$e_{0}$
copeland
2016-03-05 21:32:52
At $y=1$ we still have\[e_{1}=1+\frac14(e_{2}+e_{0}+2e_{1}).\]
At $y=1$ we still have\[e_{1}=1+\frac14(e_{2}+e_{0}+2e_{1}).\]
copeland
2016-03-05 21:33:00
And what about at $y=0?$
And what about at $y=0?$
sxu
2016-03-05 21:33:51
1/3 this tiem
1/3 this tiem
walnutwaldo20
2016-03-05 21:33:51
e0 = 1 + 1/3*(e1+2*e0)
e0 = 1 + 1/3*(e1+2*e0)
AlcumusGuy
2016-03-05 21:33:51
e0 = 2/3 e0 + 1/3 e1 + 1
e0 = 2/3 e0 + 1/3 e1 + 1
ychen
2016-03-05 21:33:51
e0 = 1 + 1/3(2e0 + e1)
e0 = 1 + 1/3(2e0 + e1)
deltaepsilon6
2016-03-05 21:33:51
e0=1+1/3(e1+2e0)
e0=1+1/3(e1+2e0)
hamup1
2016-03-05 21:33:51
$e_0 = 1+ \frac{2}{3}e_0+\frac{1}{3}e_1$
$e_0 = 1+ \frac{2}{3}e_0+\frac{1}{3}e_1$
copeland
2016-03-05 21:33:54
At $y=0$, he can't hop downward. Therefore he hops left or right with probability $\dfrac13$ each and he hops up with probability $\dfrac13$. This gives\[e_0=1+\dfrac13(e_1+2e_0).\]
At $y=0$, he can't hop downward. Therefore he hops left or right with probability $\dfrac13$ each and he hops up with probability $\dfrac13$. This gives\[e_0=1+\dfrac13(e_1+2e_0).\]
copeland
2016-03-05 21:33:55
Let me write all of our equations and clear the denominators:
Let me write all of our equations and clear the denominators:
copeland
2016-03-05 21:33:56
\begin{align*}
e_{24}&=0\\
4e_{23}&=4+e_{24}+e_{22}+2e_{23}\\
4e_{22}&=4+e_{23}+e_{21}+2e_{22}\\
&\vdots\\
4e_{2}&=4+e_{3}+e_{1}+2e_{2}\\
4e_{1}&=4+e_{2}+e_{0}+2e_{1}\\
3e_0&=3+e_1+2e_0
\end{align*}
\begin{align*}
e_{24}&=0\\
4e_{23}&=4+e_{24}+e_{22}+2e_{23}\\
4e_{22}&=4+e_{23}+e_{21}+2e_{22}\\
&\vdots\\
4e_{2}&=4+e_{3}+e_{1}+2e_{2}\\
4e_{1}&=4+e_{2}+e_{0}+2e_{1}\\
3e_0&=3+e_1+2e_0
\end{align*}
copeland
2016-03-05 21:33:59
What should we do with those?
What should we do with those?
Ericaops
2016-03-05 21:34:26
add
add
fz2012
2016-03-05 21:34:26
Add them up
Add them up
Transcranial
2016-03-05 21:34:26
Add them
Add them
MikoTennisPro
2016-03-05 21:34:26
add?
add?
illogical_21
2016-03-05 21:34:26
add!
add!
mssmath
2016-03-05 21:34:26
add?
add?
deltaepsilon6
2016-03-05 21:34:26
add
add
copeland
2016-03-05 21:34:28
Since there's a lot of symmetry and the coefficients on the variables on both sides almost add to 4, something cool ought to happen if we add all of the equations.
Since there's a lot of symmetry and the coefficients on the variables on both sides almost add to 4, something cool ought to happen if we add all of the equations.
copeland
2016-03-05 21:34:32
However, everything doesn't quite line up correctly and I'm a little worried. Is there something we can do to make things look a little cleaner?
However, everything doesn't quite line up correctly and I'm a little worried. Is there something we can do to make things look a little cleaner?
blue8931
2016-03-05 21:35:44
shift the e0 equation over a bit
shift the e0 equation over a bit
azmath333
2016-03-05 21:35:44
add e_0 to both sides of the last equation
add e_0 to both sides of the last equation
brian22
2016-03-05 21:35:44
Add some e0 to the bottom-most equation
Add some e0 to the bottom-most equation
copeland
2016-03-05 21:35:47
We could add $e_0=e_0$ to the bottom equation to get another 4 on the left side and make the right side a little neater.
We could add $e_0=e_0$ to the bottom equation to get another 4 on the left side and make the right side a little neater.
copeland
2016-03-05 21:35:53
And what about the top equation?
And what about the top equation?
sxu
2016-03-05 21:36:26
x4
x4
walnutwaldo20
2016-03-05 21:36:26
multiply by 4
multiply by 4
kikipet
2016-03-05 21:36:26
*4
*4
azmath333
2016-03-05 21:36:26
multiply by 4
multiply by 4
SuperMaltese
2016-03-05 21:36:26
multiply by 4
multiply by 4
leeandrew1029gmail.com
2016-03-05 21:36:26
multiply by 4
multiply by 4
copeland
2016-03-05 21:36:29
We could multiply the top equation by 4 to get another 4 on the left side.
We could multiply the top equation by 4 to get another 4 on the left side.
copeland
2016-03-05 21:36:30
\begin{align*}
4e_{24}&=0\\
4e_{23}&=4+e_{24}+e_{22}+2e_{23}\\
4e_{22}&=4+e_{23}+e_{21}+2e_{22}\\
&\vdots\\
4e_{2}&=4+e_{3}+e_{1}+2e_{2}\\
4e_{1}&=4+e_{2}+e_{0}+2e_{1}\\
4e_0&=3+e_1+e_0+2e_0
\end{align*}
\begin{align*}
4e_{24}&=0\\
4e_{23}&=4+e_{24}+e_{22}+2e_{23}\\
4e_{22}&=4+e_{23}+e_{21}+2e_{22}\\
&\vdots\\
4e_{2}&=4+e_{3}+e_{1}+2e_{2}\\
4e_{1}&=4+e_{2}+e_{0}+2e_{1}\\
4e_0&=3+e_1+e_0+2e_0
\end{align*}
copeland
2016-03-05 21:36:34
Adding the left sides gives\[4(e_0+e_1+\cdots+e_n).\] This looks like an important quantity, so let's call that $4S$. What is the sum on the right side?
Adding the left sides gives\[4(e_0+e_1+\cdots+e_n).\] This looks like an important quantity, so let's call that $4S$. What is the sum on the right side?
fz2012
2016-03-05 21:38:45
4(e_0+e_1+...+e_22)+3e_23+95
4(e_0+e_1+...+e_22)+3e_23+95
TheRealDeal
2016-03-05 21:38:45
95+4(e0+e1+...)
95+4(e0+e1+...)
deltaepsilon6
2016-03-05 21:38:45
95+4S-e23
95+4S-e23
hamup1
2016-03-05 21:38:45
$4\cdot 23 + 3 + 2S -(e_{23}+e_{24}) + 2S.$
$4\cdot 23 + 3 + 2S -(e_{23}+e_{24}) + 2S.$
copeland
2016-03-05 21:38:48
Summing everything gives
\[\begin{array}{rcccccccc}
4e_{24}&=&0&&&&&&\\
4e_{23}&=&4&+&e_{24}&+&e_{22}&+&2e_{23}\\
4e_{22}&=&4&+&e_{23}&+&e_{21}&+&2e_{22}\\
\vdots&&\vdots&&\vdots&&\vdots&&\vdots\\
4e_{2}&=&4&+&e_{3}&+&e_{1}&+&2e_{2}\\
4e_{1}&=&4&+&e_{2}&+&e_{0}&+&2e_{1}\\
4e_0&=&3&+&e_1&+&e_0&+&2e_0\\
\hline
4S&=&95&+&(S-e_{0})&+&(S-e_{24}-e_{23}+e_0)&+&2(S-e_{24}).
\end{array}\]
Summing everything gives
\[\begin{array}{rcccccccc}
4e_{24}&=&0&&&&&&\\
4e_{23}&=&4&+&e_{24}&+&e_{22}&+&2e_{23}\\
4e_{22}&=&4&+&e_{23}&+&e_{21}&+&2e_{22}\\
\vdots&&\vdots&&\vdots&&\vdots&&\vdots\\
4e_{2}&=&4&+&e_{3}&+&e_{1}&+&2e_{2}\\
4e_{1}&=&4&+&e_{2}&+&e_{0}&+&2e_{1}\\
4e_0&=&3&+&e_1&+&e_0&+&2e_0\\
\hline
4S&=&95&+&(S-e_{0})&+&(S-e_{24}-e_{23}+e_0)&+&2(S-e_{24}).
\end{array}\]
swagger21
2016-03-05 21:38:55
95 + 4(e0 + e1 + .... + e23) - e23
95 + 4(e0 + e1 + .... + e23) - e23
copeland
2016-03-05 21:38:57
There's $4S$ on each side, so those cancel. We also have $e_{24}=0$, so this simplifies to $0=95-e_{23}$ so \[e_{23}=95.\]
There's $4S$ on each side, so those cancel. We also have $e_{24}=0$, so this simplifies to $0=95-e_{23}$ so \[e_{23}=95.\]
copeland
2016-03-05 21:39:02
What is $e_{22}?$
What is $e_{22}?$
brian6liu
2016-03-05 21:39:41
186
186
deltaepsilon6
2016-03-05 21:39:41
186
186
MikoTennisPro
2016-03-05 21:39:41
186
186
swagger21
2016-03-05 21:39:41
186
186
calculus_riju
2016-03-05 21:39:41
186
186
fz2012
2016-03-05 21:39:41
186
186
copeland
2016-03-05 21:39:43
The equation\[4e_{23}=4+e_{24}+e_{22}+2e_{23}\] becomes \[4\cdot95=4+0+e_{22}+2\cdot95.\]This makes \[e_{22}=2\cdot95-4=186.\]
The equation\[4e_{23}=4+e_{24}+e_{22}+2e_{23}\] becomes \[4\cdot95=4+0+e_{22}+2\cdot95.\]This makes \[e_{22}=2\cdot95-4=186.\]
copeland
2016-03-05 21:39:44
What is $e_{21}?$
What is $e_{21}?$
PurplePancakes
2016-03-05 21:40:19
273
273
brian6liu
2016-03-05 21:40:19
273, so our answer is 273
273, so our answer is 273
swagger21
2016-03-05 21:40:19
273
273
fz2012
2016-03-05 21:40:19
273
273
calculus_riju
2016-03-05 21:40:19
273
273
Pot
2016-03-05 21:40:19
\boxed{\text{273}}
\boxed{\text{273}}
blue8931
2016-03-05 21:40:19
273
273
nosaj
2016-03-05 21:40:19
273
273
deltaepsilon6
2016-03-05 21:40:19
273
273
copeland
2016-03-05 21:40:21
The equation\[4e_{22}=4+e_{23}+e_{21}+2e_{22}\] becomes \[4\cdot186=4+95+e_{22}+2\cdot186.\]This makes \[e_{21}=2\cdot186-4-95=\boxed{273}.\]
The equation\[4e_{22}=4+e_{23}+e_{21}+2e_{22}\] becomes \[4\cdot186=4+95+e_{22}+2\cdot186.\]This makes \[e_{21}=2\cdot186-4-95=\boxed{273}.\]
copeland
2016-03-05 21:40:24
Rad.
Rad.
copeland
2016-03-05 21:40:37
Two more. Dos moro.
Two more. Dos moro.
copeland
2016-03-05 21:40:42
14. Centered at each lattice point in the coordinate plane are a circle radius $\dfrac1{10}$ and a square with sides of length $\dfrac15$ whose sides are parallel to the coordinate axes. The line segment from $(0,0)$ to $(1001,429)$ intersects $m$ of the squaers and $n$ of the circles. Find $m+n.$
14. Centered at each lattice point in the coordinate plane are a circle radius $\dfrac1{10}$ and a square with sides of length $\dfrac15$ whose sides are parallel to the coordinate axes. The line segment from $(0,0)$ to $(1001,429)$ intersects $m$ of the squaers and $n$ of the circles. Find $m+n.$
copeland
2016-03-05 21:40:50
Here we have a line working its way up and to the right and we want to know how often it hits a bunch of circles and squares centered at lattice points.
Here we have a line working its way up and to the right and we want to know how often it hits a bunch of circles and squares centered at lattice points.
copeland
2016-03-05 21:40:54
OK, everybody knows about 1001. What is special about 1001?
OK, everybody knows about 1001. What is special about 1001?
Ericaops
2016-03-05 21:41:11
7*11*13
7*11*13
sxu
2016-03-05 21:41:11
7,11,13
7,11,13
MikoTennisPro
2016-03-05 21:41:11
11 * 7 * 13
11 * 7 * 13
TheRealDeal
2016-03-05 21:41:11
factors as 7*11*13
factors as 7*11*13
hamup1
2016-03-05 21:41:11
$1001 = 7\cdot 11\cdot 13$
$1001 = 7\cdot 11\cdot 13$
ompatel99
2016-03-05 21:41:14
7*11*13?
7*11*13?
ChickenOnRage
2016-03-05 21:41:14
7 * 11 * 13
7 * 11 * 13
copeland
2016-03-05 21:41:18
This number comes up on all sorts of math contests because it factors nicely as $1001=7\cdot11\cdot13.$ Is that useful here?
This number comes up on all sorts of math contests because it factors nicely as $1001=7\cdot11\cdot13.$ Is that useful here?
blue8931
2016-03-05 21:41:48
7 * 11 *13, which cancels with 429 to become 3/7
7 * 11 *13, which cancels with 429 to become 3/7
hamup1
2016-03-05 21:41:48
$429 = 11\cdot 13 \cdot 3\implies \gcd(1001,429) = 11\cdot 13$
$429 = 11\cdot 13 \cdot 3\implies \gcd(1001,429) = 11\cdot 13$
illogical_21
2016-03-05 21:41:48
yes common factors with 429
yes common factors with 429
deltaepsilon6
2016-03-05 21:41:48
yes because 429=11*13*3
yes because 429=11*13*3
Pot
2016-03-05 21:41:48
useful since 429 also has 11*13
useful since 429 also has 11*13
ChickenOnRage
2016-03-05 21:41:48
429 = 3 * 11 * 13
429 = 3 * 11 * 13
copeland
2016-03-05 21:41:50
I see that since $4+9-2=11,$ we have 11 as a factor of 429! Dividing gives $429=11\cdot39.$
I see that since $4+9-2=11,$ we have 11 as a factor of 429! Dividing gives $429=11\cdot39.$
copeland
2016-03-05 21:41:54
Ooh, also we have a 13 in there so $429=11\cdot13\cdot3.$ Both of those big numbers are divisible by $11\cdot13=143$.
Ooh, also we have a 13 in there so $429=11\cdot13\cdot3.$ Both of those big numbers are divisible by $11\cdot13=143$.
copeland
2016-03-05 21:41:55
Our top-right vertex is $(143\cdot7,143\cdot3).$
Our top-right vertex is $(143\cdot7,143\cdot3).$
copeland
2016-03-05 21:41:57
What's that tell us about our line?
What's that tell us about our line?
Axolotl
2016-03-05 21:42:19
slope=7/3
slope=7/3
kikipet
2016-03-05 21:42:19
slope of 3/7
slope of 3/7
deltaepsilon6
2016-03-05 21:42:19
slope of 3/7
slope of 3/7
algebra_star1234
2016-03-05 21:42:19
slope is 3/7
slope is 3/7
copeland
2016-03-05 21:42:22
First off, it has slope $\dfrac37$.
First off, it has slope $\dfrac37$.
hamup1
2016-03-05 21:42:47
therefore just consider a 7 vy 3 box
therefore just consider a 7 vy 3 box
illogical_21
2016-03-05 21:42:47
repeats in 3 by 7 rectangles
repeats in 3 by 7 rectangles
idomath12345
2016-03-05 21:42:47
We only need to go til 7,3
We only need to go til 7,3
blue8931
2016-03-05 21:42:47
so just examine one rectangle and find a pattern
so just examine one rectangle and find a pattern
TheRealDeal
2016-03-05 21:42:47
3*7 box 143 times
3*7 box 143 times
copeland
2016-03-05 21:42:55
The line goes through $(7,3),$ and in fact it goes through every lattice point of the form $(7a,3b)$.
The line goes through $(7,3),$ and in fact it goes through every lattice point of the form $(7a,3b)$.
copeland
2016-03-05 21:42:56
copeland
2016-03-05 21:42:59
This rectangle repeats:
This rectangle repeats:
copeland
2016-03-05 21:43:00
copeland
2016-03-05 21:43:01
So what?
So what?
Ericaops
2016-03-05 21:43:36
just check one of the rectangles
just check one of the rectangles
Peggy
2016-03-05 21:43:36
just focus on one rectangle
just focus on one rectangle
TheRealDeal
2016-03-05 21:43:36
do a 7*3 box and then multiply by 143
do a 7*3 box and then multiply by 143
brian6liu
2016-03-05 21:43:36
draw in squares and circles
draw in squares and circles
andsun19
2016-03-05 21:43:36
think about one box!
think about one box!
hamup1
2016-03-05 21:43:36
rejoice because i brought graph paper
rejoice because i brought graph paper
blue8931
2016-03-05 21:43:36
focus on one rectangle and then mutiply
focus on one rectangle and then mutiply
deltaepsilon6
2016-03-05 21:43:36
consider one rectangle
consider one rectangle
Pot
2016-03-05 21:43:36
draw a very accurate diagram on graph paper XD
draw a very accurate diagram on graph paper XD
AlcumusGuy
2016-03-05 21:43:36
just focus on (0, 0) to (7, 3) and the multiply by 143
just focus on (0, 0) to (7, 3) and the multiply by 143
PurplePancakes
2016-03-05 21:43:36
count the ones that apply in one rectangle and multiply it by 143
count the ones that apply in one rectangle and multiply it by 143
copeland
2016-03-05 21:43:39
So all we need to do is solve the problem on the rectangle with corners $(0,0)$ and $(7,3)$ and multiply by the number of rectangles!
So all we need to do is solve the problem on the rectangle with corners $(0,0)$ and $(7,3)$ and multiply by the number of rectangles!
copeland
2016-03-05 21:43:40
copeland
2016-03-05 21:43:55
OK, so let's figure out if any of the points on the line $y=\dfrac37x$ falls into a box or circle around any of these points.
OK, so let's figure out if any of the points on the line $y=\dfrac37x$ falls into a box or circle around any of these points.
copeland
2016-03-05 21:44:08
Are any of them obvious?
Are any of them obvious?
TheRealDeal
2016-03-05 21:44:35
(0,0) has square and circle of course
(0,0) has square and circle of course
andsun19
2016-03-05 21:44:35
well, the one at (0,0)
well, the one at (0,0)
Pot
2016-03-05 21:44:35
only falls on circles at (0, 0) and (7,3)
only falls on circles at (0, 0) and (7,3)
Ericaops
2016-03-05 21:44:35
endpoints
endpoints
kikipet
2016-03-05 21:44:35
the 2 corners
the 2 corners
TheRealDeal
2016-03-05 21:44:35
(0,0) and (7,3)
(0,0) and (7,3)
brian6liu
2016-03-05 21:44:35
the 2 corners
the 2 corners
walnutwaldo20
2016-03-05 21:44:35
(0,0) and (7,3)
(0,0) and (7,3)
blizzard10
2016-03-05 21:44:35
(0,0), (7,3)
(0,0), (7,3)
copeland
2016-03-05 21:44:39
The line definitely falls in the both the square and circle centered at $(0,0)$ and $(7,3)$ since it passes through these two points.
The line definitely falls in the both the square and circle centered at $(0,0)$ and $(7,3)$ since it passes through these two points.
copeland
2016-03-05 21:44:46
How should we organize checking all these points?
How should we organize checking all these points?
andsun19
2016-03-05 21:45:17
Think about x=something lines and when they interesect
Think about x=something lines and when they interesect
SuperMaltese
2016-03-05 21:45:17
pair by column?
pair by column?
copeland
2016-03-05 21:45:20
That works.
That works.
copeland
2016-03-05 21:45:24
Any better ideas?
Any better ideas?
TheRealDeal
2016-03-05 21:45:58
(2,1) (3,1) (4,2) and (5,2) are the only point that need to be checked
(2,1) (3,1) (4,2) and (5,2) are the only point that need to be checked
AlcumusGuy
2016-03-05 21:45:58
look at y = blah
look at y = blah
copeland
2016-03-05 21:46:01
There are fewer rows than columns. . .
There are fewer rows than columns. . .
copeland
2016-03-05 21:46:02
Why don't we use casework on the $y$-coordinate of the points.
Why don't we use casework on the $y$-coordinate of the points.
copeland
2016-03-05 21:46:03
Start with $y=0$:
Start with $y=0$:
copeland
2016-03-05 21:46:06
We know we're in the circle and square at $(0,0).$ Can we pass through the square at $(1,0)$?
We know we're in the circle and square at $(0,0).$ Can we pass through the square at $(1,0)$?
illogical_21
2016-03-05 21:46:30
no
no
kikipet
2016-03-05 21:46:30
no
no
fishy15
2016-03-05 21:46:30
no . . .
no . . .
brian6liu
2016-03-05 21:46:30
no
no
nosaj
2016-03-05 21:46:30
nope
nope
copeland
2016-03-05 21:46:33
No! The slope of the line is $\dfrac37$. However, look at two neighboring squares:
No! The slope of the line is $\dfrac37$. However, look at two neighboring squares:
copeland
2016-03-05 21:46:36
copeland
2016-03-05 21:46:36
What's the maximum slope of a line containing a point in the left square and a point in the right square?
What's the maximum slope of a line containing a point in the left square and a point in the right square?
brian6liu
2016-03-05 21:47:09
1/4
1/4
Pot
2016-03-05 21:47:09
0.2/0.8 = 1/4
0.2/0.8 = 1/4
deltaepsilon6
2016-03-05 21:47:09
1/4
1/4
kikipet
2016-03-05 21:47:09
1/4?
1/4?
copeland
2016-03-05 21:47:12
The maximum slope is between these two points (I've put the centers at $(0,0)$ and $(1,0)$ in this example):
The maximum slope is between these two points (I've put the centers at $(0,0)$ and $(1,0)$ in this example):
copeland
2016-03-05 21:47:13
copeland
2016-03-05 21:47:15
This maximum slope is $\dfrac{0.2}{0.8}=\dfrac14$. This is less than $\dfrac37$ so two horizontally neighboring squares can't both contain a point on the line.
This maximum slope is $\dfrac{0.2}{0.8}=\dfrac14$. This is less than $\dfrac37$ so two horizontally neighboring squares can't both contain a point on the line.
copeland
2016-03-05 21:47:19
So, we can hit at most one square per row.
So, we can hit at most one square per row.
copeland
2016-03-05 21:47:22
Since the circles are inscribed in the squares, we can't hit a circle without hitting its square.
Since the circles are inscribed in the squares, we can't hit a circle without hitting its square.
copeland
2016-03-05 21:47:23
Thus we hit one square at height $y=0$ and we hit one circle at height $y=0$.
Thus we hit one square at height $y=0$ and we hit one circle at height $y=0$.
copeland
2016-03-05 21:47:25
Next for $y=1$:
Next for $y=1$:
copeland
2016-03-05 21:47:30
As before, we can either hit the square containing $(2,1)$ or the square containing $(3,1)$. Which should we check first?
As before, we can either hit the square containing $(2,1)$ or the square containing $(3,1)$. Which should we check first?
nosaj
2016-03-05 21:47:56
(2,1)
(2,1)
brian6liu
2016-03-05 21:47:56
(2, 1)
(2, 1)
blue8931
2016-03-05 21:47:56
(2,1)
(2,1)
illogical_21
2016-03-05 21:47:56
(2,1)
(2,1)
Ericaops
2016-03-05 21:47:56
(2,1)
(2,1)
TheRealDeal
2016-03-05 21:47:56
2,1
2,1
kikipet
2016-03-05 21:47:56
(2,1)
(2,1)
copeland
2016-03-05 21:47:59
Well, it looks like the line gets closer to $(2,1)$ than $(3,1)$ in our picture. If you want to be more algebraic, then for $y=1$ the line contains the point $\left(\dfrac73,1\right),$ and $\dfrac73$ is indeed closer to 2 than to 3.
Well, it looks like the line gets closer to $(2,1)$ than $(3,1)$ in our picture. If you want to be more algebraic, then for $y=1$ the line contains the point $\left(\dfrac73,1\right),$ and $\dfrac73$ is indeed closer to 2 than to 3.
copeland
2016-03-05 21:48:04
What is the bottom-right point of the square in question?
What is the bottom-right point of the square in question?
Pot
2016-03-05 21:49:03
2.1, 0.9
2.1, 0.9
swagger21
2016-03-05 21:49:03
21/10, 9/10
21/10, 9/10
brian6liu
2016-03-05 21:49:03
(2.1, 0.9)
(2.1, 0.9)
copeland
2016-03-05 21:49:05
The bottom-right point is $(2.1,0.9).$ What do you notice about this point?
The bottom-right point is $(2.1,0.9).$ What do you notice about this point?
leeandrew1029gmail.com
2016-03-05 21:49:44
also on the line
also on the line
sxu
2016-03-05 21:49:44
it works!
it works!
brian6liu
2016-03-05 21:49:44
it lies on the line
it lies on the line
Pot
2016-03-05 21:49:44
wow it has slope 3/7 if you extend through origin!1!!!
wow it has slope 3/7 if you extend through origin!1!!!
andsun19
2016-03-05 21:49:44
it exactly matches?
it exactly matches?
SimonSun
2016-03-05 21:49:44
it matches
it matches
SuperMaltese
2016-03-05 21:49:44
7/3 ratio!
7/3 ratio!
Ericaops
2016-03-05 21:49:44
it is on the line
it is on the line
copeland
2016-03-05 21:49:46
This point is on the line! $\dfrac{0.9}{2.1}=\dfrac37.$ Therefore we hit this square.
This point is on the line! $\dfrac{0.9}{2.1}=\dfrac37.$ Therefore we hit this square.
copeland
2016-03-05 21:49:48
Do we hit the circle inside this square?
Do we hit the circle inside this square?
kikipet
2016-03-05 21:50:03
no
no
Pot
2016-03-05 21:50:03
nope
nope
memc38123
2016-03-05 21:50:03
no
no
eswa2000
2016-03-05 21:50:03
no
no
sxu
2016-03-05 21:50:03
NOPE
NOPE
PurplePancakes
2016-03-05 21:50:03
no
no
leeandrew1029gmail.com
2016-03-05 21:50:03
no
no
brian6liu
2016-03-05 21:50:03
no, it barely touches the square
no, it barely touches the square
copeland
2016-03-05 21:50:05
No. ONLY the corner of the square is on the line. Since the circle we've defined is the incircle of the square, it does not contain the corner.
No. ONLY the corner of the square is on the line. Since the circle we've defined is the incircle of the square, it does not contain the corner.
copeland
2016-03-05 21:50:09
Therefore we hit one square at height $y=1$ and zero circles at height $y=1$.
Therefore we hit one square at height $y=1$ and zero circles at height $y=1$.
copeland
2016-03-05 21:50:10
Next for $y=2$:
Next for $y=2$:
copeland
2016-03-05 21:50:11
How do we handle the $y=2$ case?
How do we handle the $y=2$ case?
copeland
2016-03-05 21:51:07
We could do the same thing, but stare at the diagram a moment. Stare deep into its soul. . .
We could do the same thing, but stare at the diagram a moment. Stare deep into its soul. . .
brian6liu
2016-03-05 21:51:35
it's symmetric to y=1, just turn the rectangle
it's symmetric to y=1, just turn the rectangle
PurplePancakes
2016-03-05 21:51:35
It's symmetric
It's symmetric
simon1221
2016-03-05 21:51:35
symmetry!
symmetry!
hamup1
2016-03-05 21:51:35
symmetric
symmetric
copeland
2016-03-05 21:51:36
The diagram is rotationally symmetric. So?
The diagram is rotationally symmetric. So?
Transcranial
2016-03-05 21:52:05
Its same
Its same
AlcumusGuy
2016-03-05 21:52:05
it will pass through corner of a square
it will pass through corner of a square
kikipet
2016-03-05 21:52:05
it touches the top left corner of (5,2)'s square
it touches the top left corner of (5,2)'s square
hamup1
2016-03-05 21:52:05
exact same amount as y=1
exact same amount as y=1
sxu
2016-03-05 21:52:05
just the square
just the square
simon1221
2016-03-05 21:52:05
it intersects the square at (5,2) but not the circle
it intersects the square at (5,2) but not the circle
leeandrew1029gmail.com
2016-03-05 21:52:05
square on 5,2 touches the line
square on 5,2 touches the line
Ericaops
2016-03-05 21:52:05
(5,2) works for the square but not the circle
(5,2) works for the square but not the circle
andsun19
2016-03-05 21:52:05
so 1
so 1
brian6liu
2016-03-05 21:52:05
(2, 1) corresponds to (5, 2), so (5, 2) works, and the corner is barely on the libne
(2, 1) corresponds to (5, 2), so (5, 2) works, and the corner is barely on the libne
copeland
2016-03-05 21:52:08
So we also hit one square at height $y=2$ and zero circles at height $y=2$.
So we also hit one square at height $y=2$ and zero circles at height $y=2$.
copeland
2016-03-05 21:52:09
Finally we hit one circle at $y=3$ and one square at $y=3$.
Finally we hit one circle at $y=3$ and one square at $y=3$.
copeland
2016-03-05 21:52:10
In all, we hit 4 squares and 2 circles in this rectangle.
In all, we hit 4 squares and 2 circles in this rectangle.
copeland
2016-03-05 21:52:16
Now we multiply! Since there are 143 of these rectangles, we hit $4\cdot143$ squares and $2\cdot143$ circles.
Now we multiply! Since there are 143 of these rectangles, we hit $4\cdot143$ squares and $2\cdot143$ circles.
brian6liu
2016-03-05 21:52:43
no
no
sxu
2016-03-05 21:52:43
Wait, won't you overcount?
Wait, won't you overcount?
PurplePancakes
2016-03-05 21:52:43
WAIT we overcounted
WAIT we overcounted
blue8931
2016-03-05 21:52:43
no no no
no no no
copeland
2016-03-05 21:52:48
No?
No?
kikipet
2016-03-05 21:52:54
Doesn't that overcount?
Doesn't that overcount?
hamup1
2016-03-05 21:52:54
corners!
corners!
copeland
2016-03-05 21:52:55
Oh, I counted the corners a lot of times. How many intersections are there actually?
Oh, I counted the corners a lot of times. How many intersections are there actually?
brian6liu
2016-03-05 21:53:55
430 squares and 144 circles
430 squares and 144 circles
PurplePancakes
2016-03-05 21:53:55
4*143+2
4*143+2
hamup1
2016-03-05 21:53:55
3*143 + 1*143 + 1 + 1
3*143 + 1*143 + 1 + 1
Stephenpiano
2016-03-05 21:53:55
We could not take the upper square and circle and then multiply by 143 and finally add 1 circle and 1 square for an easier counting method.
We could not take the upper square and circle and then multiply by 143 and finally add 1 circle and 1 square for an easier counting method.
simon1221
2016-03-05 21:53:55
3*143 squares, 1*143 circles, +2 for the ones at the end
3*143 squares, 1*143 circles, +2 for the ones at the end
kikipet
2016-03-05 21:53:55
143* (3 squares, 1 circle) + 1 each
143* (3 squares, 1 circle) + 1 each
Pot
2016-03-05 21:53:55
3 squares and 1 circle, so 4*143 then add origin = 574
3 squares and 1 circle, so 4*143 then add origin = 574
copeland
2016-03-05 21:53:58
We hit the interior squares $2\cdot143$ times and we hit the interior circles 0 times.
We hit the interior squares $2\cdot143$ times and we hit the interior circles 0 times.
copeland
2016-03-05 21:53:58
We hit the corner squares 144 times and we hit the corner circles 144 times.
We hit the corner squares 144 times and we hit the corner circles 144 times.
copeland
2016-03-05 21:53:59
There are a total of \[2\cdot143+2\cdot144=2\cdot287=\boxed{574}\] intersections.
There are a total of \[2\cdot143+2\cdot144=2\cdot287=\boxed{574}\] intersections.
copeland
2016-03-05 21:54:03
OK, now what?
OK, now what?
kikipet
2016-03-05 21:54:46
Problem 15!
Problem 15!
Pot
2016-03-05 21:54:46
on to number 15!
on to number 15!
copeland
2016-03-05 21:54:51
Yup.
Yup.
copeland
2016-03-05 21:54:53
15. Circles $\omega_1$ and $\omega_2$ intersect at points $X$ and $Y.$ Line $\ell$ is tangent to $\omega_1$ and $\omega_2$ at $A$ and $B,$ respectively, with line $AB$ closer to point $X$ than to $Y.$ Circle $\omega$ passes through $A$ and$B$ intersecting $\omega_1$ again at $D\neq A$ and intersecting $\omega_2$ again at $C\neq B.$ The three points $C,$ $Y,$ and $D$ are collinear, $XC=67,$ $XY=47,$ and $XD=37.$ Find $AB.$
15. Circles $\omega_1$ and $\omega_2$ intersect at points $X$ and $Y.$ Line $\ell$ is tangent to $\omega_1$ and $\omega_2$ at $A$ and $B,$ respectively, with line $AB$ closer to point $X$ than to $Y.$ Circle $\omega$ passes through $A$ and$B$ intersecting $\omega_1$ again at $D\neq A$ and intersecting $\omega_2$ again at $C\neq B.$ The three points $C,$ $Y,$ and $D$ are collinear, $XC=67,$ $XY=47,$ and $XD=37.$ Find $AB.$
copeland
2016-03-05 21:55:00
Wut?
Wut?
copeland
2016-03-05 21:55:02
So. . .
So. . .
brian22
2016-03-05 21:55:08
Well, Problem 15 is trivial, so we're done guys!
Well, Problem 15 is trivial, so we're done guys!
copeland
2016-03-05 21:55:12
Almost.
Almost.
copeland
2016-03-05 21:56:53
And our diagram:
And our diagram:
copeland
2016-03-05 21:56:55
copeland
2016-03-05 21:56:58
Obviously this diagram isn't so great: $XY$ is supposd to be bigger than $XD$.
Obviously this diagram isn't so great: $XY$ is supposd to be bigger than $XD$.
copeland
2016-03-05 21:57:03
Let's start over. The real diagram looks a bit more like this:
Let's start over. The real diagram looks a bit more like this:
copeland
2016-03-05 21:57:05
copeland
2016-03-05 21:57:20
The lengths we know are the three edges emanating from $X.$ I've left those off because we're clearly going to need to know a lot more before we can us them. Don't forget, though, that those are the edges we know about. We're trying to find $AB.$
The lengths we know are the three edges emanating from $X.$ I've left those off because we're clearly going to need to know a lot more before we can us them. Don't forget, though, that those are the edges we know about. We're trying to find $AB.$
copeland
2016-03-05 21:57:25
First off, what do we get from those three circles?
First off, what do we get from those three circles?
copeland
2016-03-05 21:58:49
What is the line $XY?$
What is the line $XY?$
copeland
2016-03-05 21:59:07
Oh, oops, there's also a typo in the problem. We want:
Oh, oops, there's also a typo in the problem. We want:
copeland
2016-03-05 21:59:14
15. Circles $\omega_1$ and $\omega_2$ intersect at points $X$ and $Y.$ Line $\ell$ is tangent to $\omega_1$ and $\omega_2$ at $A$ and $B,$ respectively, with line $AB$ closer to point $X$ than to $Y.$ Circle $\omega$ passes through $A$ and$B$ intersecting $\omega_1$ again at $D\neq A$ and intersecting $\omega_2$ again at $C\neq B.$ The three points $C,$ $Y,$ and $D$ are collinear, $XC=67,$ $XY=47,$ and $XD=37.$ Find $AB^2.$
15. Circles $\omega_1$ and $\omega_2$ intersect at points $X$ and $Y.$ Line $\ell$ is tangent to $\omega_1$ and $\omega_2$ at $A$ and $B,$ respectively, with line $AB$ closer to point $X$ than to $Y.$ Circle $\omega$ passes through $A$ and$B$ intersecting $\omega_1$ again at $D\neq A$ and intersecting $\omega_2$ again at $C\neq B.$ The three points $C,$ $Y,$ and $D$ are collinear, $XC=67,$ $XY=47,$ and $XD=37.$ Find $AB^2.$
copeland
2016-03-05 21:59:39
Note that we want to find $AB^2,$ not $AB$.
Note that we want to find $AB^2,$ not $AB$.
hamup1
2016-03-05 22:00:16
by radical axis lemma we have that da, xy, and bc intersect at a point p
by radical axis lemma we have that da, xy, and bc intersect at a point p
PurplePancakes
2016-03-05 22:00:16
radial axis
radial axis
brian22
2016-03-05 22:00:16
Radical axis
Radical axis
sxu
2016-03-05 22:00:16
radical axis, wait, wait
radical axis, wait, wait
AlcumusGuy
2016-03-05 22:00:16
radical axis
radical axis
ninjataco
2016-03-05 22:00:16
XY is the radical axis of w1 and w2
XY is the radical axis of w1 and w2
copeland
2016-03-05 22:00:21
The line $XY$ is the radical axis of $\omega_1$ and $\omega_2$. This is the set of points for which the power with respect to $\omega_1$ is equal to the power with respect to $\omega_2$.
The line $XY$ is the radical axis of $\omega_1$ and $\omega_2$. This is the set of points for which the power with respect to $\omega_1$ is equal to the power with respect to $\omega_2$.
copeland
2016-03-05 22:00:26
Likewise, $DA$ is the radical axis of $\omega_1$ and $\omega$ and $BC$ is the radical axis of $\omega_2$ and $\omega$.
Likewise, $DA$ is the radical axis of $\omega_1$ and $\omega$ and $BC$ is the radical axis of $\omega_2$ and $\omega$.
copeland
2016-03-05 22:00:29
What do we know about those?
What do we know about those?
Ericaops
2016-03-05 22:01:15
concurrent
concurrent
brian6liu
2016-03-05 22:01:15
they're concurrent
they're concurrent
hamup1
2016-03-05 22:01:15
intersect at a point! call it $P$ :)
intersect at a point! call it $P$ :)
AlcumusGuy
2016-03-05 22:01:15
concurrent at radical center
concurrent at radical center
sxu
2016-03-05 22:01:15
concurrent?
concurrent?
copeland
2016-03-05 22:01:17
They're concurrent. Specifically, call $Z$ the intersection of $AD$ and $BC$ (clearly they intersect). Since $Z$ lies on both lines, $\text{Pow}_{\omega_1}(Z)=\text{Pow}_{\omega}(Z)$ and $\text{Pow}_{\omega_2}(Z)=\text{Pow}_{\omega}(Z).$ Putting those together gives $\text{Pow}_{\omega_1}(Z)=\text{Pow}_{\omega_2}(Z),$ so $Z$ also lies on $XY$.
They're concurrent. Specifically, call $Z$ the intersection of $AD$ and $BC$ (clearly they intersect). Since $Z$ lies on both lines, $\text{Pow}_{\omega_1}(Z)=\text{Pow}_{\omega}(Z)$ and $\text{Pow}_{\omega_2}(Z)=\text{Pow}_{\omega}(Z).$ Putting those together gives $\text{Pow}_{\omega_1}(Z)=\text{Pow}_{\omega_2}(Z),$ so $Z$ also lies on $XY$.
copeland
2016-03-05 22:01:23
But it's obviously $Z$ and not $P.$
But it's obviously $Z$ and not $P.$
copeland
2016-03-05 22:01:25
copeland
2016-03-05 22:01:27
What else do you see here?
What else do you see here?
copeland
2016-03-05 22:02:12
Here, check this out:
Here, check this out:
copeland
2016-03-05 22:02:14
deltaepsilon6
2016-03-05 22:02:47
cyclic quadrilateral!
cyclic quadrilateral!
hamup1
2016-03-05 22:02:47
cyclic quads!
cyclic quads!
PurplePancakes
2016-03-05 22:02:47
cyclic quads
cyclic quads
deltaepsilon6
2016-03-05 22:02:47
2 cyclic quadrilaterals
2 cyclic quadrilaterals
copeland
2016-03-05 22:02:49
Two cyclic quadrilaterals!
Two cyclic quadrilaterals!
hamup1
2016-03-05 22:03:00
xazb might be cyclic
xazb might be cyclic
copeland
2016-03-05 22:03:05
Huh, might be.
Huh, might be.
SohamSchwarz119
2016-03-05 22:03:27
Is AXBZ cyclic?
Is AXBZ cyclic?
copeland
2016-03-05 22:03:29
Is it?
Is it?
swagger21
2016-03-05 22:04:06
miquel!
miquel!
fishy15
2016-03-05 22:04:06
lets check
lets check
PurplePancakes
2016-03-05 22:04:06
YES IT IS angle chase for a bit
YES IT IS angle chase for a bit
Ericaops
2016-03-05 22:04:06
yes
yes
hamup1
2016-03-05 22:04:06
yes!
yes!
calculus_riju
2016-03-05 22:04:06
we can find out by angle chasing
we can find out by angle chasing
copeland
2016-03-05 22:04:18
$X$ is actually the Miquel point of $CDZ$ with respect to $A,$ $B,$ and $Y.$
$X$ is actually the Miquel point of $CDZ$ with respect to $A,$ $B,$ and $Y.$
copeland
2016-03-05 22:04:21
Specifically, $AXYD$ and $BXYC$ are cyclic. Let me prove really quickly that $AXBZ$ is also cyclic for those of you who haven't seen it.
Specifically, $AXYD$ and $BXYC$ are cyclic. Let me prove really quickly that $AXBZ$ is also cyclic for those of you who haven't seen it.
copeland
2016-03-05 22:04:27
Since the other two quadrilaterals are cyclic, we get
\begin{align*}
\angle D+\angle YXA=180^\circ\\
\angle C+\angle YXB=180^\circ\\
\end{align*}
Since the other two quadrilaterals are cyclic, we get
\begin{align*}
\angle D+\angle YXA=180^\circ\\
\angle C+\angle YXB=180^\circ\\
\end{align*}
copeland
2016-03-05 22:04:32
We would like to compute\[\angle Z+\angle AXB=\theta\] and show that $\theta=180^\circ$.
We would like to compute\[\angle Z+\angle AXB=\theta\] and show that $\theta=180^\circ$.
copeland
2016-03-05 22:04:37
If we add the three equations we get \[(\angle D+\angle C+\angle Z)+(\angle YXA+\angle YXB+\angle AXB)=180^\circ+180^\circ+\theta.\]
If we add the three equations we get \[(\angle D+\angle C+\angle Z)+(\angle YXA+\angle YXB+\angle AXB)=180^\circ+180^\circ+\theta.\]
copeland
2016-03-05 22:04:40
This simplifies to \[180^\circ+360^\circ=180^\circ+180^\circ+\theta,\] so $\theta=180^\circ,$ as promised.
This simplifies to \[180^\circ+360^\circ=180^\circ+180^\circ+\theta,\] so $\theta=180^\circ,$ as promised.
hamup1
2016-03-05 22:04:56
$180=\angle{DYX} + \angle{XYC} = 180-\angle{DAX} + 180-\angle{CXB} = \angle{XAZ} + \angle{XBZ} = 180.$
$180=\angle{DYX} + \angle{XYC} = 180-\angle{DAX} + 180-\angle{CXB} = \angle{XAZ} + \angle{XBZ} = 180.$
deltaepsilon6
2016-03-05 22:04:56
xazb is cyclic
xazb is cyclic
copeland
2016-03-05 22:05:02
deltaepsilon6
2016-03-05 22:05:05
how does that help
how does that help
copeland
2016-03-05 22:05:23
No idea, but the principle of "More circles makes you look smart" definitely applies here.
No idea, but the principle of "More circles makes you look smart" definitely applies here.
copeland
2016-03-05 22:05:28
Now, we have a lot of angles and circles. We should probably be looking for similar inscribed angles that will help. Now is the time to remember that we know $XD,$ $XY,$ and $XC.$
Now, we have a lot of angles and circles. We should probably be looking for similar inscribed angles that will help. Now is the time to remember that we know $XD,$ $XY,$ and $XC.$
copeland
2016-03-05 22:05:37
What do you got?
What do you got?
copeland
2016-03-05 22:07:13
Not a lot yet, huh? I'm sure it's time to look for similar triangles now.
Not a lot yet, huh? I'm sure it's time to look for similar triangles now.
copeland
2016-03-05 22:07:28
Where can we get some equal angles that might help? Which trianglees are similar?
Where can we get some equal angles that might help? Which trianglees are similar?
copeland
2016-03-05 22:09:08
This is definitely the hard part.
This is definitely the hard part.
copeland
2016-03-05 22:09:40
We have lengths $DX,$ $YX,$ and $CX.$ Where can we find similar triangles that us some of those lengths?
We have lengths $DX,$ $YX,$ and $CX.$ Where can we find similar triangles that us some of those lengths?
copeland
2016-03-05 22:12:37
A lot of people are excited about $\triangle CDX.$ This is not a bad triangle, except that it's hard to move either that angle at $D$ or that angle at $C$ around nicely.
A lot of people are excited about $\triangle CDX.$ This is not a bad triangle, except that it's hard to move either that angle at $D$ or that angle at $C$ around nicely.
copeland
2016-03-05 22:13:05
However, we do have a lot of things we can do with $\angle ADX.$
However, we do have a lot of things we can do with $\angle ADX.$
calculus_riju
2016-03-05 22:13:45
can u join XA and XB?
can u join XA and XB?
calculus_riju
2016-03-05 22:13:45
DXZ and CXZ
DXZ and CXZ
deltaepsilon6
2016-03-05 22:13:45
XDA and DZX
XDA and DZX
copeland
2016-03-05 22:14:18
Here are a couple of good ideas.
Here are a couple of good ideas.
copeland
2016-03-05 22:14:42
Considering $\omega_1,$ we have equal inscribed angles $\angle XDA=\angle XAB.$ Similarly, $\omega_2$ tells us $\angle XCB=\angle XBA.$
Considering $\omega_1,$ we have equal inscribed angles $\angle XDA=\angle XAB.$ Similarly, $\omega_2$ tells us $\angle XCB=\angle XBA.$
copeland
2016-03-05 22:14:46
copeland
2016-03-05 22:15:14
It would be nice if we had all three of these triangles similar, $\triangle DXZ\sim\triangle AXB\sim\triangle ZXC,$ and we're close with a handful of matching angles.
It would be nice if we had all three of these triangles similar, $\triangle DXZ\sim\triangle AXB\sim\triangle ZXC,$ and we're close with a handful of matching angles.
copeland
2016-03-05 22:15:16
Let's try to show the rest. Can you argue that $\angle BZX=\alpha?$
Let's try to show the rest. Can you argue that $\angle BZX=\alpha?$
PurplePancakes
2016-03-05 22:17:52
yes, same arc
yes, same arc
hamup1
2016-03-05 22:17:52
$\angle{BAX}$ and $\angle{BZX}$ both intercept arc $BX$, thus equal to $\alpha$
$\angle{BAX}$ and $\angle{BZX}$ both intercept arc $BX$, thus equal to $\alpha$
deltaepsilon6
2016-03-05 22:17:52
properties of cyclic quadrilaterals
properties of cyclic quadrilaterals
danzhi
2016-03-05 22:17:52
[asy]\angle BZX = \angle XAB = \alpha[\asy]
[asy]\angle BZX = \angle XAB = \alpha[\asy]
deltaepsilon6
2016-03-05 22:17:52
if XAB=alpha, then cyclic quadrilaterals state that BZX is also alpha
if XAB=alpha, then cyclic quadrilaterals state that BZX is also alpha
copeland
2016-03-05 22:17:56
These angles are also inside the lower circle: $\angle XAB$ and $\angle XZB$ subtend the same arc, so $\angle XZB=\alpha$ as well. Symmetrically, $\angle XZA=\beta.$
These angles are also inside the lower circle: $\angle XAB$ and $\angle XZB$ subtend the same arc, so $\angle XZB=\alpha$ as well. Symmetrically, $\angle XZA=\beta.$
copeland
2016-03-05 22:17:58
copeland
2016-03-05 22:18:05
Now we have similar triangles! How can we use that?
Now we have similar triangles! How can we use that?
ninjataco
2016-03-05 22:18:42
length ratios?
length ratios?
fishy15
2016-03-05 22:18:42
side lengths
side lengths
copeland
2016-03-05 22:19:03
Which ratios? What length do we get instantly, knowing $DX,$ $YX,$ and $XC?$
Which ratios? What length do we get instantly, knowing $DX,$ $YX,$ and $XC?$
deltaepsilon6
2016-03-05 22:19:58
XD/XZ=XZ/XC
XD/XZ=XZ/XC
PurplePancakes
2016-03-05 22:19:58
ZX is the geometric mean of DX and CX
ZX is the geometric mean of DX and CX
hamup1
2016-03-05 22:19:58
XZ?
XZ?
copeland
2016-03-05 22:20:01
We know $DX$ and $CX.$ Since $\triangle DXZ\sim\triangle ZXC,$ we get\[\frac{DX}{XZ}=\frac{ZX}{XC}.\]
We know $DX$ and $CX.$ Since $\triangle DXZ\sim\triangle ZXC,$ we get\[\frac{DX}{XZ}=\frac{ZX}{XC}.\]
copeland
2016-03-05 22:20:03
This tells us
This tells us
copeland
2016-03-05 22:20:04
\[ZX=\sqrt{DX\cdot XC}=\sqrt{37\cdot67}.\]
\[ZX=\sqrt{DX\cdot XC}=\sqrt{37\cdot67}.\]
copeland
2016-03-05 22:20:11
Can you see any more triangles that are similar to these three?
Can you see any more triangles that are similar to these three?
copeland
2016-03-05 22:20:24
deltaepsilon6
2016-03-05 22:20:33
XAB
XAB
copeland
2016-03-05 22:20:38
Yup, that's on our list.
Yup, that's on our list.
copeland
2016-03-05 22:21:55
Do you see $\alpha$ anywhere else in the diagram?
Do you see $\alpha$ anywhere else in the diagram?
brian22
2016-03-05 22:22:30
AYZ
AYZ
hamup1
2016-03-05 22:22:36
YAZ?
YAZ?
hamup1
2016-03-05 22:22:36
YAZ and YBZ
YAZ and YBZ
brian22
2016-03-05 22:22:36
AYZ
AYZ
andsun19
2016-03-05 22:22:36
yes, ayz
yes, ayz
copeland
2016-03-05 22:22:50
Great (some of those are angles and some are triangles, note).
Great (some of those are angles and some are triangles, note).
copeland
2016-03-05 22:23:06
I see another triangle! In $\omega_1,$ the angles $\angle ADX$ and $\angle AYX$ subtend the same arc. Therefore $\angle AYX=\alpha.$ Symmetrically, $\angle BYX=\beta.$
I see another triangle! In $\omega_1,$ the angles $\angle ADX$ and $\angle AYX$ subtend the same arc. Therefore $\angle AYX=\alpha.$ Symmetrically, $\angle BYX=\beta.$
copeland
2016-03-05 22:23:23
Now, we could try to construct more ratios, but it looks a lot like we'd end up reproving Power of a Point six different ways if we did. I'd rather not.
Now, we could try to construct more ratios, but it looks a lot like we'd end up reproving Power of a Point six different ways if we did. I'd rather not.
copeland
2016-03-05 22:23:28
Let me drop those new angles on the diagram, ignoring $C$ and $D$ for now since we just used them.
Let me drop those new angles on the diagram, ignoring $C$ and $D$ for now since we just used them.
copeland
2016-03-05 22:23:29
copeland
2016-03-05 22:23:35
What do you see?
What do you see?
Ericaops
2016-03-05 22:24:17
parallelogram
parallelogram
danzhi
2016-03-05 22:24:17
parallelgram AZBY
parallelgram AZBY
calculus_riju
2016-03-05 22:24:17
parallelogram
parallelogram
brian22
2016-03-05 22:24:17
Parallelogram!
Parallelogram!
swagger21
2016-03-05 22:24:17
parallelogram?
parallelogram?
copeland
2016-03-05 22:24:19
I see a parallelogram!
I see a parallelogram!
copeland
2016-03-05 22:24:19
copeland
2016-03-05 22:24:21
Now we know $ZX$ and we know $XY,$ so we seem to know quite a bit about the segment $ZY.$
Now we know $ZX$ and we know $XY,$ so we seem to know quite a bit about the segment $ZY.$
copeland
2016-03-05 22:24:31
What else should we draw?
What else should we draw?
hamup1
2016-03-05 22:25:36
midpoint of $AB$!!
midpoint of $AB$!!
copeland
2016-03-05 22:25:46
Let's label the intersection of $AB$ and $YZ$ as $M.$ (It's the midpoint of both $AB$ and $YZ$, of course. A couple of you noticed that it is the midpoint of $AB$ a long time ago since it's on the radical axis of $\omega_1$ and $\omega_2.$)
Let's label the intersection of $AB$ and $YZ$ as $M.$ (It's the midpoint of both $AB$ and $YZ$, of course. A couple of you noticed that it is the midpoint of $AB$ a long time ago since it's on the radical axis of $\omega_1$ and $\omega_2.$)
copeland
2016-03-05 22:25:48
copeland
2016-03-05 22:26:04
Now, what do we have?
Now, what do we have?
deltaepsilon6
2016-03-05 22:26:39
XM
XM
danzhi
2016-03-05 22:26:39
XM
XM
fishy15
2016-03-05 22:26:39
similar triangles?
similar triangles?
Ericaops
2016-03-05 22:26:39
similar triangles
similar triangles
copeland
2016-03-05 22:26:41
Similar triangles WITH MEDIANS. That's a little more information.
Similar triangles WITH MEDIANS. That's a little more information.
copeland
2016-03-05 22:26:49
Now we have medians of similar triangles so we get a new equation.
Now we have medians of similar triangles so we get a new equation.
copeland
2016-03-05 22:27:26
Specifcially, $\triangle AYZ$ with median $AM$ is similar to $\triangle XAB$ with median $XM.$
Specifcially, $\triangle AYZ$ with median $AM$ is similar to $\triangle XAB$ with median $XM.$
copeland
2016-03-05 22:27:31
What can we do?
What can we do?
sxu
2016-03-05 22:28:29
ratios to find lengths
ratios to find lengths
hamup1
2016-03-05 22:29:43
$\dfrac{YZ}{AX} = \dfrac{AB}{XM}$?
$\dfrac{YZ}{AX} = \dfrac{AB}{XM}$?
danzhi
2016-03-05 22:29:43
AB : YZ = XM : AM
AB : YZ = XM : AM
copeland
2016-03-05 22:29:47
Here's the one I see:\[\frac{AM}{XM}=\frac{YZ}{AB}.\]
Here's the one I see:\[\frac{AM}{XM}=\frac{YZ}{AB}.\]
copeland
2016-03-05 22:30:41
Let's think about all these ideas.
Let's think about all these ideas.
hamup1
2016-03-05 22:31:02
wait this is good
wait this is good
copeland
2016-03-05 22:31:06
Yeah, this is good.
Yeah, this is good.
copeland
2016-03-05 22:31:22
Why did I pick the ratios I picked?
Why did I pick the ratios I picked?
copeland
2016-03-05 22:31:33
I wanted to get $AB$ (or $AM$) from both triangles.
I wanted to get $AB$ (or $AM$) from both triangles.
copeland
2016-03-05 22:31:49
It's the base in one and the median in the other, so we had to take that ratio.
It's the base in one and the median in the other, so we had to take that ratio.
copeland
2016-03-05 22:31:53
Can we write $YZ$ in terms of the known values $XY$ and $XZ?$
Can we write $YZ$ in terms of the known values $XY$ and $XZ?$
hamup1
2016-03-05 22:32:32
XY + XZ
XY + XZ
sxu
2016-03-05 22:32:32
XY+XZ
XY+XZ
brian6liu
2016-03-05 22:32:32
XY+XZ
XY+XZ
MathLearner01
2016-03-05 22:32:32
YZ=XY+XZ
YZ=XY+XZ
copeland
2016-03-05 22:32:34
Simple, it's the sum: $YZ=YX+XZ$.
Simple, it's the sum: $YZ=YX+XZ$.
copeland
2016-03-05 22:32:35
Can we write $XM$ in terms of the known values $XY$ and $XZ?$
Can we write $XM$ in terms of the known values $XY$ and $XZ?$
deltaepsilon6
2016-03-05 22:33:45
XM=.5*(XZ-XY)
XM=.5*(XZ-XY)
MathLearner01
2016-03-05 22:33:45
(XZ-XY)/2
(XZ-XY)/2
copeland
2016-03-05 22:33:47
Since $YM=ZM,$ we have $XM+YX=XZ-XM$ so $XM=(XZ-YX)/2$.
Since $YM=ZM,$ we have $XM+YX=XZ-XM$ so $XM=(XZ-YX)/2$.
copeland
2016-03-05 22:33:50
Now we can rewrite that equation, \[\frac{AM}{XM}=\frac{YZ}{AB}.\] It becomes, \[\frac{AM}{(XZ-YX)/2}=\frac{YX+XZ}{AB}.\]
Now we can rewrite that equation, \[\frac{AM}{XM}=\frac{YZ}{AB}.\] It becomes, \[\frac{AM}{(XZ-YX)/2}=\frac{YX+XZ}{AB}.\]
copeland
2016-03-05 22:33:57
Cross-multiplying gives \[2AM\cdot AB=(XZ-YX)(XZ+YX)=XZ^2-YX^2.\]
Cross-multiplying gives \[2AM\cdot AB=(XZ-YX)(XZ+YX)=XZ^2-YX^2.\]
copeland
2016-03-05 22:33:58
Now what?
Now what?
deltaepsilon6
2016-03-05 22:35:10
let AM=.5AB and substitute
let AM=.5AB and substitute
hamup1
2016-03-05 22:35:10
$AB^2 = XZ^2-YX^2$!
$AB^2 = XZ^2-YX^2$!
brian6liu
2016-03-05 22:35:10
our answer is just XZ^2-YX^2=37*67-47^2=270
our answer is just XZ^2-YX^2=37*67-47^2=270
calculus_riju
2016-03-05 22:35:10
2AM=AB^2
2AM=AB^2
calculus_riju
2016-03-05 22:35:10
we know RHS
we know RHS
calculus_riju
2016-03-05 22:35:10
jst compute
jst compute
calculus_riju
2016-03-05 22:35:10
tada!
tada!
nosaj
2016-03-05 22:35:10
plug in stuff!
plug in stuff!
copeland
2016-03-05 22:35:12
Well, $2AM=AB,$ so the left side is what we want, and we know the right side, too:
Well, $2AM=AB,$ so the left side is what we want, and we know the right side, too:
copeland
2016-03-05 22:35:14
\[AB^2=\left(\sqrt{37\cdot67}\right)^2-\left(47\right)^2.\]
\[AB^2=\left(\sqrt{37\cdot67}\right)^2-\left(47\right)^2.\]
copeland
2016-03-05 22:35:18
What does that simplify to?
What does that simplify to?
brian6liu
2016-03-05 22:35:44
270
270
brian22
2016-03-05 22:35:44
270 I suppose
270 I suppose
swagger21
2016-03-05 22:35:44
270
270
fclvbfm934
2016-03-05 22:35:44
270
270
fz2012
2016-03-05 22:35:49
270
270
copeland
2016-03-05 22:35:51
\begin{align*}
AB^2&=37\cdot67-47^2\\
&=(47-10)(47+20)-47^2\\
&=47\cdot10-200\\
&=\boxed{270}.
\end{align*}
\begin{align*}
AB^2&=37\cdot67-47^2\\
&=(47-10)(47+20)-47^2\\
&=47\cdot10-200\\
&=\boxed{270}.
\end{align*}
copeland
2016-03-05 22:35:55
Great!
Great!
copeland
2016-03-05 22:35:59
That's all. That's it.
That's all. That's it.
copeland
2016-03-05 22:36:05
Time to go have dinner?
Time to go have dinner?
blizzard10
2016-03-05 22:36:45
That was long.
That was long.
copeland
2016-03-05 22:36:51
Possibly a Math Jam record, yes.
Possibly a Math Jam record, yes.
copeland
2016-03-05 22:37:00
Thanks, Problem 15 geo writers.
Thanks, Problem 15 geo writers.
copeland
2016-03-05 22:37:07
Don't forget to come back for the AIME II Math Jam, tentatively planned for March 18.
Don't forget to come back for the AIME II Math Jam, tentatively planned for March 18.
calculus_riju
2016-03-05 22:37:12
see u on the nect jam i.e. AIME II
see u on the nect jam i.e. AIME II
copeland
2016-03-05 22:37:17
See you around.
See you around.
violu
2016-03-05 22:38:00
Is the math jam finished?
Is the math jam finished?
copeland
2016-03-05 22:38:06
Yes, all done. Nothing else.
Yes, all done. Nothing else.
copeland
2016-03-05 22:38:10
Go away!
Go away!
Copyright © 2024 AoPS Incorporated. This page is copyrighted material. You can view and print this page for your own use, but you cannot share the contents of this file with others.